Mental health study questions: Townsend

Lakukan tugas rumah & ujian kamu dengan baik sekarang menggunakan Quizwiz!

23. A client has the following symptoms: preoccupation with imagined defect, verbalizations that are out of proportion to actual physical abnormalities, and numerous visits to plastic surgeons to seek relief. Which nursing diagnosis would best describe the problems evidenced by these symptoms? A. Ineffective coping B. Disturbed body image C. Complicated grieving D. Panic anxiety

ANS: B The symptoms presented describe the DSM-5 diagnosis of body dysmorphic disorder, and the related nursing diagnosis is disturbed body image.

15. A patient being treated for symptoms of PTSD following a shooting incident at a local elementary school reports I feel like theres no reason to go on living when so many others died. Which of these is the most appropriate response by the nurse at this juncture? A. Youve got lots of reasons to go on living B. Are you having thoughts of hurting or killing yourself? C. Youre just experiencing survivor guilt. D. There must be something that gives you hope.

ANS: B This patient is expressing hopelessness, and it is a priority to assess for suicide ideation in these circumstances. Items A and D minimize the patients experience of feeling hopelessness. Item C may be a useful strategy to encourage the patient that this is a common experience of trauma survivors, but the immediate priority is determining patient safety.

29. While improving, a client demands to have a phone installed in the intensive care unit (ICU) room. When a nurse states, "This is not allowed; it is a unit rule," the client angrily demands to see the doctor. Which approach should the nurse use in this situation? A. Provide an explanation for the necessity of the unit rule. B. Assist the client to discuss anger and frustrations. C. Call the physician and relay the request. D. Arrange for a phone to be installed in the client's unit room.

ANS: B Clients who demand special privileges may be diagnosed with narcissistic personality disorder. The best approach in this situation is for the nurse to identify the function that anger, frustration, and rage serve for the client. The verbalization of feelings may help the client to gain insight into his or her behavior.

Which client statement would the nurse recognize as indicating that the client understands dietary teaching related to lithium carbonate (Lithobid) treatment? A. "I will limit my intake of fluids daily." B. "I will maintain normal salt intake." C. "I will take Lithobid on an empty stomach." D. "I will increase my caloric intake to prevent weight loss."

ANS: B A client taking Lithobid should be taught not to skimp on dietary sodium intake. He or she should take Lithobid on a full stomach to avoid gastrointestinal upset and choose lower-calorie foods to prevent weight gain. KEY: Cognitive Level: Application | Integrated Processes: Nursing Process: Evaluation | Client Need: Physiological Integrity: Pharmacological and Parenteral Therapies

7. An employee of the National Institute for Mental Health would most likely be involved in the: a. Expansion of psychiatric units in general hospitals b. Development of education programs for community mental health treatment c. Legislation and advocacy for the rights of people with mental disorders d. Deinstitutionalization of patients

ANS: B After the National Mental Health Act was passed in 1946, the National Institute of Mental Health (NIMH) administered its programs. Objectives included development of education and research programs for community mental health treatment approaches.

A client presents in the emergency department with complaints of overwhelming anxiety. Which of the following is a priority for the nurse to assess? A. Risk for suicide B. Cardiac status C. Current stressors D. Substance use history

ANS: B Although all of the listed aspects of assessment are important, the priority is to evaluate cardiac status since a person having an MI, CHF, or mitral valve prolapse can present with symptoms of anxiety. KEY: Cognitive Level: Application | Integrated Processes: Nursing Process: Planning | Client Need: Safe and Effective Care Environment: Management of Care

A client diagnosed with bipolar I disorder: manic episode refuses to take lithium carbonate because he complains that it makes him feel sick. Which of the following medications might be alternatively prescribed for mood stabilization in bipolar disorders? A. Sertraline (Zoloft) B. Valproic acid (Depakote) C. Trazodone (Desyrel) D. Paroxetine (Paxil)

ANS: B Although lithium is a prototype drug in the treatment of bipolar disorders, anticonvulsants such as valproic acid also have demonstrated efficacy for mood stabilization. KEY: Cognitive Level: Application | Integrated Processes: Nursing Process: Assessment | Client Need: Safe and Effective Care Environment

A client diagnosed with obsessive-compulsive disorder is admitted to a psychiatric unit. The client has an elaborate routine for toileting activities. Which would be an appropriate initial client outcome during the first week of hospitalization? A. The client will refrain from ritualistic behaviors during daylight hours. B. The client will wake early enough to complete rituals prior to breakfast. C. The client will participate in three unit activities by day 3. D. The client will substitute a productive activity for rituals by day 1.

ANS: B An appropriate initial client outcome is for the client to wake early enough to complete rituals prior to breakfast. The nurse should also provide a structured schedule of activities and later in treatment begin to gradually limit the time allowed for rituals. KEY: Cognitive Level: Application | Integrated Processes: Nursing Process: Planning | Client Need: Psychosocial Integrity

Bipolar epidemiology

*Gender* Incidence is roughly equal: *Average age at onset* Early 20s. more common in single than in married persons. It occurs more often in the higher socioeconomic classes.

Mood-Stabilizing Agents

*Indications:* prevention and treatment of manic episodes associated with bipolar disorder *Examples:* lithium carbonate, clonazepam, carbamazepine, valproic acid, lamotrigine, topiramate, oxcarbazepine, verapamil, antipsychotics

Treatment Modalities for Bipolar Disorder

*Individual Psychotherapy* not best because client is trying to please the therapist and relationship remains shallow and rigid *Group Therapy* helpful after acute phase has passed, have peer support *Family Therapy* goal is to restore adaptive family functioning, most effective with psychotherapeutic and pharmacotherapeutic treatment *Cognitive Therapy* Client is taught to control thoughts and distortions Also: ECT, Meds

Lithium mech of action

*May modulate the effects of certain neurotransmitters* 1) norepinephrine 2) Serotonin 3) Dopamine 4) Glutamate 5) GABA Thereby stabilizing symptoms associated with bipolar disorder

children/adolescent Treatment strategies

*Medications* -Lithium -Divalproex -Carbamazepine -Atypical antipsychotics *Family* -Interventions -Psychoeducation about bipolar disorder -Communication training -Problem-solving -Skills training

Client and family education topics

*Nature of the Illness* 1) Causes of bipolar disorder 2) Cyclic nature of the illness 3) Symptoms of depression 4) Symptoms of mania *Management of the Illness* 1) Medication management 2) Assertive techniques 3) Anger management *Support Services* 1) Crisis hotline 2) Support groups 3) Individual psychotherapy 4) Legal/financial assistance

Priority nursing diagnoses

*Risk for Injury related to:* Extreme hyperactivity, increased agitation, and lack of control over purposeless and potentially injurious movements *Risk for Violence Self-directed or other-directed related to:* Manic excitement Delusional thinking Hallucinations Impulsivity

Lithium toxicity

*Therapeutic range* 1.0 to 1.5 mEq/L (acute mania) 0.6 to 1.2 mEq/L (maintenance) *Initial symptoms of toxicity include* Blurred vision, ataxia, tinnitus, persistent nausea and vomiting, and severe diarrhea Ensure that client consumes adequate sodium and fluid in diet.

A client is taking chlordiazepoxide (Librium) for generalized anxiety disorder symptoms. In which situation should a nurse recognize that this client is at greatest risk for drug overdose? A. When the client has a knowledge deficit related to the effects of the drug B. When the client combines the drug with alcohol C. When the client takes the drug

ANS: B Both Librium and alcohol are central nervous system depressants. In combination, these drugs have an additive effect and can suppress the respiratory system, leading to respiratory arrest and death. KEY: Cognitive Level: Application | Integrated Processes: Nursing Process: Evaluation | Client Need: Physiological Integrity: Pharmacological and Parenteral Therapies

*Stage III: Delirious mania*

1) A grave form of the disorder characterized by an intensification of the symptoms associated with acute mania. 2) The condition is rare since the advent of antipsychotic medication. *Symptomology* 1) Labile mood, panic anxiety 2) Clouding of consciousness, disorientation 3) Frenzied psychomotor activity 4) Exhaustion and possibly death without intervention

Physiological Influences

1) Brain lesions 2) Enlarged ventricles 3) Medication side effects

Bipolar II Disorder

1) Characterized by bouts of *major depression* with episodic occurrence of *hypomania* 2) Has never met criteria for full manic episode

*Stage I: Hypomania*

1) Cheerful mood but underlying irritability that surfaces when wishes are unfulfilled. 2) Great worth, rapid flow of ideas. 3) Distracted easily, Extroverted and sociable. 4) Not causing marked impairment or hospitalization

Cyclothymic Disorder: duration?

1) Chronic mood disturbance 2) At least 2-year duration (1 yr children/adolescents) Numerous episodes of hypomanic/depressive symptoms 3) never w/o symptoms for more than 2 months 4) Hypomania and depressed mood of insufficient severity to meet the criteria for either bipolar I or II disorder

Episodes of mania may be treated with ECT when:

1) Client does not tolerate medication. 2) Client fails to respond to medication. 3) Client's life is threatened by dangerous behavior or exhaustion.

Client education for Antipsychotics

1) Do not discontinue drug abruptly. 2) Use sunblock lotion when outdoors. 3) Rise slowly from a sitting or lying position. 4) Avoid alcohol and over-the-counter medications. 5) Continue to take the medication, even if feeling well and as though it is not needed. 6) Symptoms may return if medication is discontinued 7) Report the following symptoms to physician: -Sore throat, fever, malaise -Unusual bleeding, easy bruising, skin rash -Persistent nausea and vomiting -Severe headache, rapid heart rate -Difficulty urinating or excessive urination -Muscle twitching, tremors -Darkly colored urine, pale stools -Yellow skin or eyes -Excessive thirst or hunger -Muscular incoordination or weakness

Client education for Verapamil

1) Do not discontinue the drug abruptly. 2) Rise slowly from sitting or lying position to prevent sudden drop in blood pressure. 3) Report: Irregular heart beat, chest pain Shortness of breath, pronounced dizziness, Swelling of hands and feet, Profound mood swings Severe and persistent headache

*Stage II: Acute mania*

1) Marked impairment in functioning; usually requires hospitalization 2) Elation and euphoria, a continuous "high" 3) Flight of ideas, accelerated, pressured speech 4) Hallucinations and delusions 5) Excessive psychomotor activity 6) Social and sexual inhibition 7) Little need for sleep

Bipolar disorder

1) Mood disorder involving swings of mood from profound depression to extreme euphoria (mania) with intervening periods of normalcy. 2) Can have a seasonal aspect to s/s onset 3) May or may not have Delusions or hallucinations

For Lithium, notify Physician in the following cases:

1) Persistent nausea and vomiting 2) Severe diarrhea 3) Ataxia 4) Blurred vision 5) Tinnitus 6) Excessive output of urine 7) Increasing tremors 8) Mental confusion

Client education for Anticonvulsants

1) Refrain from discontinuing the drug abruptly. 2) Report: skin rash, unusual bleeding, spontaneous bruising, sore throat, fever, malaise, dark urine, and yellow skin or eyes. 3) Avoid using alcohol and over-the-counter medications.

Client education for Lithium

1) Take the medication regularly. 2) Do not skimp on dietary sodium. 3) Drink 6 to 8 glasses of water each day. 4) Notify physician if vomiting or diarrhea occur. 5) Have serum lithium level checked every 1 to 2 months or as advised by physician.

To assist the psychiatrist in determining appropriate medication needs, the nurse has been asked to assess whether a patient is in a hypomanic or an acute manic state. Which of the following symptoms are consistent with hypomania? Select all that apply. 1) Cheerful mood, but underlying irritability surfaces rapidly when needs are not fulfilled 2) Fragmented cognition and perception; often psychotic 3) Delusions of grandeur 4) Easily distracted, which sometimes interferes with completing goal-directed activity 5) Extroverted and sociable

1, 4, 5.

32. A client diagnosed with Cluster C traits sits alone and ignores other's attempts to converse. When ask to join a group the client states, "No, thanks." In this situation, which should the nurse assign as an initial nursing diagnosis? A. Fear R/T hospitalization B. Social isolation R/T poor self-esteem C. Risk for suicide R/T to hopelessness D. Powerlessness R/T dependence issues

ANS: B Clients diagnosed with Cluster C traits are described as anxious and fearful. The DSM-5 divides Cluster C personality disorders into three categories: avoidant, dependent, and obsessive-compulsive. Anxiety and fear contribute to social isolation.

A newly admitted client is diagnosed with bipolar disorder: manic episode. Which symptom related to altered thought is the nurse most likely to assess? A. Pacing B. Flight of ideas C. Lability of mood D. Irritability

ANS: B Clients diagnosed with bipolar disorder: manic episode experience cognition and perception fragmentation often with psychosis during acute mania. Rapid thinking proceeds to racing and disjointed thinking (flight of ideas) and may be manifested by a continuous flow of accelerated, pressured speak with abrupt changes from topic to topic. KEY: Cognitive Level: Application | Integrated Processes: Nursing Process: Assessment | Client Need: Psychosocial Integrity

A 27-year-old client was diagnosed 5 years ago with schizophrenia. What course of treatment should the nurse expect to be implemented? A. Eventual admission for long-term care in a psychiatric facility B. Community-based care with numerous brief hospitalizations C. Case management in the community with few relapses D. Occasional contact with outpatient counselors and psychiatrists

ANS: B Community-based care is the standard of treatment that followed the deinstitutionalization movement. Schizophrenia is a chronic disease that includes both exacerbations and remissions in the course of the illness, leading to numerous brief hospitalizations. KEY: Cognitive Level: Application | Integrated Processes: Nursing Process: Implementation | Client Need: Psychosocial Integrity

What are 5 of the cognitive distortions typical of someone w/ an eating d/o?

1. Dichotomous or all-or-nothing thinking: "ive gained 2 lbs so i'll be up to 100 lbs soon" 2. Magnification: i binged last night so i cant go out w/ anyone 3. Selective abstraction: "I can only be happier 10 lbs lighter" 4. Overgeneralization: "I didnt eat anything yesterday and did ok so it's fine to not eat for a wk or 2 5. Catastrophizing: "I purged last night for the first time in 4 months, ill never recover"

5. A nurse working in a psychiatric institution during the hospital expansion era would have most likely worked in which location? a. Near small communities with access to families and activities b. In rural areas removed from family and social activities c. Near urban areas with access to families and low-paying jobs d. In urban areas without access to families and jobs

ANS: B During the hospital expansion era, most psychiatric institutions were located in rural areas removed from family and social activities.

What are the 2 types of anorexia nervosa?

1. Restricting: dieting & exercising w/ no binge eating or misuse of laxatives, diuretics, or enemas 2. Binge eating & purging: binge eating with misuse of laxatives, diuretics, or enemas

The DSM-5 requires at least 4 out of 7 criteria be met for a diagnosis of paranoids personality disorder. What are the 7 criteria?

1. Suspects, without sufficient basis, that others are exploiting, harming, deceiving them.. 2. Is preoccupied with unjustifiable doubts about the loyalty or trustworthiness of friends/associates 3. Is reluctant to confide in others b/c of unwarranted fear that info will be used against themself 4. Reads hidden demeaning or threatening meanings into benign remarks or events 5. Persistently bears grudges 6. Perceives attacks on his or her character or reputation that are not apparent to others and is quick to counterattack or react angrily 7. Has recurrent suspicions, without justification, regarding fidelity of spouse/partner

What are the four primary modes of treatment in DBT?

1. group skills training 2. individual psychotherapy 3. telephone contact 4. therapist consultation/team meeting

Diagnostic criteria for narcissistic personality disorder includes at least 5 of 9 traits. What are those criteria?

1. has grandiose sense of self-importance 2. is preoccupied with fantasies of unlimited success, power, brilliance, beauty 3. Believe that he/she is special and unique and should only associate with others like him/her 4. requires excessive admiration 5. Has a sense of entitlement 6. Is interpersonally exploitative 7. Lacks empathy 8, is often envious of others or believes others are envious of him/her 9. shows arrogant, haughty behaviors or attitudes

22. Which intervention is most appropriate for a nurse to implement when working with older adults and their caregivers? a. Work with legislators to advocate for policies which support families. b. Refer clients and families to community support groups to reduce stress. c. Involve clients and families in case management programs to coordinate care. d. Conduct depression screenings with clients and families on a regular basis.

ANS: B Family caregivers and older adults are at risk for health disruptions. Involvement in a community support group can help reduce stress of caregiving and provide networking opportunities for the older adult.

A client's spouse asks, "What evidence supports the possibility of genetic transmission of bipolar disorder?" Which is the best nursing reply? A. "Clients diagnosed with bipolar disorders have alterations in neurochemicals that affect behaviors." B. "Higher rates of relatives diagnosed with bipolar disorder are found in families of clients diagnosed with this disorder." C. "Higher rates of relatives of clients diagnosed with bipolar disorder respond in an exaggerated way to daily stress." D. "More individuals diagnosed with bipolar disorder come from higher socioeconomic and educational backgrounds."

ANS: B Family studies have shown that if one parent is diagnosed with bipolar disorder, the risk that a child will have the disorder is around 28%. If both parents are diagnosed with the disorder, the risk is two to three times as great. KEY: Cognitive Level: Application | Integrated Processes: Teaching/Learning | Client Need: Health Promotion and Maintenance

A nurse learns at report that a newly admitted client experiencing mania is demonstrating grandiose delusions. The nurse should recognize that which client statement would provide supportive evidence of this symptom? A. "I can't stop my sexual urges. They have led me to numerous affairs." B. "I'm the world's most perceptive attorney." C. "My wife is distraught about my overspending." D. "The FBI is out to get me."

ANS: B Grandiosity is defined as a belief that personal abilities are better than anyone else's. This client is experiencing delusions of grandeur, which are commonly experienced in mania. KEY: Cognitive Level: Application | Integrated Processes: Evaluation | Client Need: Psychosocial Integrity

The nurse is prioritizing nursing diagnoses in the plan of care for a client experiencing a manic episode. Number the diagnoses in order of the appropriate priority. ___Disturbed sleep pattern evidenced by sleeping on 4 to 5 hours per night. ___Risk for injury related to manic hyperactivity ___Impaired social interaction evidenced by manipulation of others. ___Imbalanced nutrition: Less than body requirements evidenced by loss of weight and poor skin turgor.

3. Disturbed sleep pattern evidenced by sleeping on 4 to 5 hours per night. 1. Risk for injury related to manic hyperactivity 4. Impaired social interaction evidenced by manipulation of others. 2. Imbalanced nutrition: Less than body requirements evidenced by loss of weight and poor skin turgor.

A community health nurse is teaching a class to expectant parents. All participants lack infant care knowledge. A student nurse asks, "If you had to assign a nursing diagnosis to this group, what would it be?" What is the best nursing reply? A. "I would assign the nursing diagnosis of cognitive deficit." B. "I would assign the nursing diagnosis of knowledge deficit." C. "I would assign the nursing diagnosis of altered family processes." D. "I would assign the nursing diagnosis of risk for caregiver role strain."

ANS: B Knowledge deficit is defined as the absence or deficiency of cognitive information related to a specific topic. Cognitive deficit would indicate an alteration in the ability to process information, and this evidence is not provided in the question. KEY: Cognitive Level: Analysis | Integrated Processes: Nursing Process: Analysis | Client Need: Health Promotion and Maintenance

2. A 12-year-old girl suddenly refuses to change for gym, participate in physical activities, has difficulty walking and sitting, and will not eat her food at lunchtime. What should the school nurse consider when assessing this child's symptoms? A- Sexual abuse B- Emotional neglect C- Physical neglect D- Emotional abuse

A

What is anorexia nervosa?

A mixture of symptoms including significantly low body weight, intense fear of gaining weight or becoming fat, and a disturbance in experiencing body weight or shape

Where is a pt w/ anorexia discharged to once an acceptable weight is est?

A partial hospitalization program or an intensive outpatient program

24. The nurse should recognize which factors that distinguish personality disorders from psychosis? A. Functioning is more limited in personality disorders than in psychosis. B. Major disturbances of thought are absent in personality disorders. C. Personality disordered clients require hospitalization more frequently. D. Personality disorders do not affect family relationships as much as psychosis.

ANS: B Major disturbances of thought are absent in personality disorders and are a classic symptom of psychosis.

What differentiates mood disorder from the typical stormy behavior of youth?

A visible manifestation of behavior change that lasts for several weeks

What are the diagnostic criteria for Major Depressive Disorder?

A) 5 or more of the following symptoms for at least 2 weeks that represent a change from previous functioning: 1. Depressed mood most of the day, most days (in children it can be irritable) 2. Markedly diminished interest/pleasure in most/all activities 3. Significant weight loss when not dieting or weight gain (change of 5% or more of body weight in a month) 4. Insomnia/hypersomnia nearly q day 5. Psychomotor agitation or retardation q day 6. Fatigue or loss of energy nearly q day 7. Feelings of worthlessness or excessive guilt 8. Diminished ability to think or concentrate 9. Recurrent thoughts of death, recurrent suicidal ideation B) Symptoms cause impairment in social/occupational functioning C) Not attributable to other physiological effects D) Major depressive episode is not better explained by schizoaffective disorder or other psychotic disorders E) There has never been a manic or hypomanic episode

Annie has trichotillomania. She is receiving treatment at the mental health clinic with habit-reversal therapy. Which of the following elements would be included in this therapy? (Select all that apply) A. Awareness training B. Competing response training C. Social Support D. Hypnotherapy E. Aversive therapy

A. Awareness training, B. Competing response training, C. Social support

The nurse can anticipate a prescription for what medication for the client who was just diagnosed with obsessive compulsive disorder? A. Clomipramine B. Clonidine C. Clonazepam D. Propranolol

A. Clomipramine, a tricyclic antidepressant, as well as SSRIs such as fluoxetine (Prozac), paroxetine (Paxil), sertraline (Zoloft) and fluvoxamine (Luvox) have been approved for treatment of OCD. Clonidine and propranolol, are anthypertensives that have been used used successful to treat anxiety disorders. Clonazepam and other benzodiazepines are used to treat social anxiety disorder.

Which statement about antidepressant medications, in general, can serve as a basis for client and family teaching? A. Onset of action is from 1 to 6 weeks. B. They tend to be more effective for men. C. Recent memory impairment is commonly observed. D. They often cause the client to have diurnal variation.

A. Onset of action is from 1 to 6 weeks. People are accustomed to fast results from medication: thirty minutes for aspirin, 24 hours for antibiotics. Information is necessary to prevent discouragement and maintain compliance.

A client with OCD spends many hours each day washing her hands. The most likely reason she washes her hands so much is that it: A. Relieves her anxiety B. Reduced her probability of infection C. Gives her a feeling of control over her life D. Increases her self-concept

A. Relieves her anxiety

The nurse has developed a plan for a client with a severe sleep pattern disturbance to spend 20 minutes in the gym exercising each afternoon. Which intervention should be scheduled for upon returning to the unit? A. Rest B. Group therapy C. A protein-based snack D. Unstructured private time

A. Rest A depressed client usually has little energy. After even a short exercise period, the client may feel exhausted and need rest.

A client who is experiencing a panic attack just arrived at the ER. Which is the priority nursing intervention for this client? A. Stay with the client and reassure safety B. Administer a dose of diazepam C. Leave the client alone in a quiet room so that she can calm down. D. Encourage the client to talk about what triggered the attack.

A. Stay with the client and reassure safety

The client with bulimia differs from the client with anorexia nervosa by a. maintaining a normal weight. b. holding a distorted body image. c. doing more rigorous exercising. d. purging to keep weight down.

A. maintaining a normal weight. Many bulimics are at or near normal weight, whereas clients with anorexia nervosa are underweight.

The mental health nurse practitioner would include what initial intervention in the care of the client with hoarding disorder: A. Psychoeducation about their disorder B. Ordering neuroimaging to determine activity in the cingulate cortex. C. Psychopharmacology including an SSRI D. Cognitive-behavioral therapy

A. psychoeducation about their disorder. This is the most likely INITIAL intervention. Treatment for hoarding disorder is most commonly a combination of cognitive-behavioral therapy and SSRIs. Decreased activity in the cingulate cortex IS associated with hoarding disorder (pg 540) but neuroimaging of the client's brain is unlikely to be ordered to diagnose/treat this disorder.

A depressed, socially withdrawn client tells the nurse, "There is no sense in trying. I am never able to do anything right!" The nurse can best begin to attack this cognitive distortion by: A. suggesting, "Let's look at what you just said, that you can 'never do anything right.'" B. querying, "Tell me what things you think you are not able to do correctly." C. asking, "Is this part of the reason you think no one likes you?" D. saying, "That is the most unrealistic thing I have ever heard."

A. suggesting, "Let's look at what you just said, that you can 'never do anything right.'" Cognitive distortions can be refuted by examining them, but to examine them the nurse must gain the client's willingness to participate.

When the nurse remarks to a depressed client, "I see you are trying not to cry. Tell me what is happening." The nurse should be prepared to A. wait quietly for the client to reply. B. prompt the client if the reply is slow. C. repeat the question if the client does not answer promptly. D. review the client's medical record to support the client's response.

A. wait quietly for the client to reply. Depressed clients think slowly and take long periods to formulate answers and respond. The nurse must be prepared to wait for a reply.

24. A nurse who is working with persons with serious mental illness should recognize that: a. Persons with severe mental illness require institutionalization until they are functional. b. Inadequate community resources have caused problems with homelessness among this population. c. Serious mental illness originates from childhood events and therapy is ineffective. d. Motivation influences a persons ability to earn a living and purchase necessary medications.

ANS: B Many people with serious mental illness live in poverty because they lack the ability to maintain a suitable standard of living. Brief hospital stays and inadequate community resources have resulted in an increased number of persons with serious mental illness living on the streets or in jail.

8. A pessimistic client expresses low self-worth, has much difficulty making decisions, avoids positions of responsibility, and has a behavioral pattern of "suffering" in silence. Which underlying cause of this client's personality disorder should a nurse recognize? A. "Nurturance was provided from many sources, and independent behaviors were encouraged." B. "Nurturance was provided exclusively from one source, and independent behaviors were discouraged." C. "Nurturance was provided exclusively from one source, and independent behaviors were encouraged." D. "Nurturance was provided from many sources, and independent behaviors were discouraged."

ANS: B Nurturance provided from one source and discouragement of independent behaviors can attribute to the etiology of dependent personality disorder. Dependent behaviors may be rewarded by a parent who is overprotective and discourages autonomy.

16. An example of primary prevention concerning mental health is: a. Group psychotherapy b. Stress reduction education c. Case management d. Monitoring illness symptoms

ANS: B Primary prevention refers to the reduction of health risks, thus stress reduction education is the correct response. Group psychotherapy and case management are secondary preventions, and monitoring illness symptoms is a tertiary prevention.

25. Which intervention would a nurse most likely use when implementing relapse management? a. Use a holistic view of the system. b. Identify triggers. c. Understand the individuals personality. d. Provide crisis intervention.

ANS: B Recognizing triggers that may lead to illness helps the consumer manage the illness and promotes recovery. Examples of triggers are poor social skills, hopelessness, and poor symptom management.

What tool should a nurse use to differentiate occasional spontaneous behaviors of children from behaviors associated with bipolar disorder? A. "Risky Activity" tool B. "FIND" tool C. "Consensus Committee" tool D. "Monotherapy" tool

ANS: B The Consensus Group recommends that clinicians use the FIND tool to differentiate occasional spontaneous behaviors of children from behaviors associated with bipolar disorder. FIND is an acronym that stands for frequency, intensity, number, and duration and is used to assess behaviors in children. KEY: Cognitive Level: Application | Integrated Processes: Nursing Process: Assessment | Client Need: Psychosocial Integrity

1. During an assessment interview, a client diagnosed with antisocial personality disorder spits, curses, and refuses to answer questions. Which is the most appropriate nursing statement to address this behavior? A. "You are very disrespectful. You need to learn to control yourself." B. "I understand that you are angry, but this behavior will not be tolerated." C. "What behaviors could you modify to improve this situation?" D. "What anti-personality-disorder medications have helped you in the past?"

ANS: B The appropriate nursing statement is to reflect the client's feeling while setting firm limits on behavior. Clients diagnosed with antisocial personality disorder have a low tolerance for frustration, see themselves as victims, and use projection as a primary ego defense mechanism. Antidepressants and anxiolytics are used for symptom relief; however, there are no specific medications targeted for the treatment of a personality disorder.

A client diagnosed with schizophrenia is hospitalized because of an exacerbation of psychosis related to antipsychotic medication nonadherence. Which level of care does the client's hospitalization reflect? A. Primary prevention level of care B. Secondary prevention level of care C. Tertiary prevention level of care D. Case management level of care

ANS: B The client's hospitalization reflects the secondary prevention level of care. Secondary prevention aims at minimizing symptoms and is accomplished through early identification of problems and prompt initiation of effective treatment. KEY: Cognitive Level: Application | Integrated Processes: Nursing Process: Implementation | Client Need: Safe and Effective Care Environment

A client is diagnosed with cyclothymic disorder. What client behaviors should the nurse expect to assess? A. The client expresses "feeling blue most of the time." B. The client has endured periods of elation and dysphoria lasting for more than 2 years. C. The client fixates on hopelessness and thoughts of suicide continually. D. The client has labile moods with periods of acute mania.

ANS: B The essential feature of cyclothymic disorder is a chronic mood disturbance of at least 2 years' duration, involving numerous episodes of hypomania and depressed mood of insufficient severity or duration to meet the criteria for bipolar I or II disorder. KEY: Cognitive Level: Application | Integrated Processes: Nursing Process: Assessment | Client Need: Psychosocial Integrity

After teaching a client about lithium carbonate (Lithane), a nurse would consider the teaching successful on the basis of which client statement? A. "I should expect to feel better in a couple of days." B. "I'll call my doctor immediately if I experience any diarrhea or ringing in my ears." C. "If I forget a dose, I can double the dose the next time I take this drug." D. "I need to restrict my intake of any food containing salt."

ANS: B The initial signs of lithium toxicity include ataxia, blurred vision, severe diarrhea, persistent nausea and vomiting, and tinnitus. KEY: Cognitive Level: Application | Integrated Processes: Nursing Process: Evaluation | Client Need: Physiological Integrity: Pharmacological and Parenteral Therapies

A nursing instructor is teaching about case management. What student statement indicates that learning has occurred? A. "Case management is a method used to achieve independent client care." B. "Case management provides coordination of services required to meet client needs." C. "Case management exists to facilitate client admission to needed inpatient services." D. "Case management is a method to facilitate physician reimbursement."

ANS: B The instructor evaluates that learning has occurred when a student defines case management as providing coordination of services required to meet client needs. Case management strives to organize client care so that specific outcomes are achieved within allotted time frames. KEY: Cognitive Level: Application | Integrated Processes: Nursing Process: Evaluation | Client Need: Psychosocial Integrity

11. A nursing instructor is teaching students about clients diagnosed with histrionic personality disorder and the quality of their relationships. Which student statement indicates that learning has occurred? A. "Their dramatic style tends to make their interpersonal relationships quite interesting and fulfilling." B. "Their interpersonal relationships tend to be shallow and fleeting, serving their dependency needs." C. "They tend to develop few relationships because they are strongly independent but generally maintain deep affection." D. "They pay particular attention to details, which can frustrate the development of relationships."

ANS: B The instructor should evaluate that learning has occurred when the student describes clients diagnosed with histrionic personality disorder as having shallow, fleeting interpersonal relationships that serve their dependency needs. Histrionic personality disorder is characterized by colorful, dramatic, and extroverted behavior. These individuals also have difficulty maintaining long-lasting relationships.

A nursing student questions an instructor regarding the order for fluvoxamine (Luvox), 300 mg daily, for a client diagnosed with obsessive-compulsive disorder (OCD). Which instructor reply is most accurate? A. "High doses of tricyclic medications will be required for effective treatment of OCD." B. "Selective serotonin reuptake inhibitor (SSRI) doses, in excess of what is effective for treating depression, may be required for OCD." C. "The dose of Luvox is low due to the side effect of daytime drowsiness and nighttime insomnia." D. "The dosage of Luvox is outside the therapeutic range and needs to be questioned."

ANS: B The most accurate instructor response is that SSRI doses in excess of what is effective for treating depression may be required in the treatment of OCD. SSRIs have been approved by the U.S. Food and Drug Administration for the treatment of OCD. Common side effects include headache, sleep disturbances, and restlessness. KEY: Cognitive Level: Application | Integrated Processes: Nursing Process: Implementation | Client Need: Physiological Integrity: Pharmacological and Parenteral Therapies

A client is brought to an emergency department after being violently raped. Which nursing action is appropriate? A. Discourage the client from discussing the event, as this may lead to further emotional trauma. B. Remain nonjudgmental and actively listen to the client's description of the event. C. Meet the client's self-care needs by assisting with showering and perineal care. D. Provide cues, based on police information, to encourage further description of the event.

ANS: B The most appropriate nursing action is to remain nonjudgmental and actively listen to the client's description of the event. It is important to also communicate to the victim that he or she is safe and that it is not his or her fault. Nonjudgmental listening provides an avenue for client catharsis needed in order to begin the process of healing. KEY: Cognitive Level: Application | Integrated Processes: Nursing Process: Implementation | Client Need: Psychosocial Integrity

16. Using a behavioral approach, which nursing intervention is most appropriate when caring for a client diagnosed with borderline personality disorder? A. Seclude the client when inappropriate behaviors are exhibited. B. Contract with the client to reinforce positive behaviors with unit privileges. C. Teach the purpose of antianxiety medications to improve medication compliance. D. Encourage the client to journal feelings to improve awareness of abandonment issues.

ANS: B The most appropriate nursing intervention from a behavioral perspective is to contract with the client to reinforce positive behaviors with unit privileges. Behavioral strategies offer reinforcement for positive change.

A family member is seeking advice about an elderly parent who seems to worry unnecessarily about everything. The family member states, "Should I seek psychiatric help for my mother?" Which is an appropriate nursing reply? A. "My mother also worries unnecessarily. I think it is part of the aging process." B. "Anxiety is considered abnormal when it is out of proportion to the stimulus causing it and when it impairs functioning." C. "From what you have told me, you should get her to a psychiatrist as soon as possible." D. "Anxiety is a complex phenomenon and is effectively treated only with psychotropic medications."

ANS: B The most appropriate reply by the nurse is to explain to the family member that anxiety is considered abnormal when it is out of proportion and impairs functioning. Anxiety is a normal reaction to a realistic danger or threat to biological integrity or self-concept. KEY: Cognitive Level: Application | Integrated Processes: Teaching/Learning | Client Need: Psychosocial Integrity

18. Samuel, a 19-year-old high school student, has been admitted to the psychiatric unit with a diagnosis of adjustment disorder with disturbance of conduct. He assaulted a teacher when he was told he was receiving detentions for a pattern of tardiness. The nurse, while completing rounds, finds the patient in his room crying, and one of his wrists is bleeding from a self-inflicted cut made by a piece of metal from an unknown source. Prioritize each of the following nursing interventions from 1 to 5, with 1 being the highest priority. ___ A. Check the patients vital signs. ___ B. Assess the wound site. ___ C. Contact the parents. ___ D. Discuss with Samuel what precipitated this event. ___ E. Cleanse and treat the wound site to prevent infection.

A: 3 B: 1 C: 5 D: 4 E: 2 The first priority is assessment (Item B), followed by providing care to meet physical and safety needs (Items E and A). The next priority is responding to the patients emotional needs (Item D), and finally, contacting the patients parents (Item C) in accordance with standards for confidentiality of medical information.

13. When planning care for a client diagnosed with borderline personality disorder, which self-harm behavior should a nurse expect the client to exhibit? A. The use of highly lethal methods to commit suicide B. The use of suicidal gestures to evoke a rescue response from others C. The use of isolation and starvation as suicidal methods D. The use of self-mutilation to decrease endorphins in the body

ANS: B The nurse should expect that a client diagnosed with borderline personality disorder might use suicidal gestures to evoke a rescue response from others. Repetitive, self-mutilative behaviors are common in clients diagnosed with borderline personality disorders. These behaviors are generated by feelings of abandonment following separation from significant others.

21. Which client is a nurse most likely to admit to an inpatient facility for self-destructive behaviors? A. A client diagnosed with antisocial personality disorder B. A client diagnosed with borderline personality disorder C. A client diagnosed with schizoid personality disorder D. A client diagnosed with paranoid personality disorder

ANS: B The nurse should expect that a client diagnosed with borderline personality disorder would be most likely to be admitted to an inpatient facility for self-destructive behaviors. Clients diagnosed with this disorder often exhibit repetitive, self-mutilative behaviors. Most gestures are designed to evoke a rescue response.

A client refuses to go on a cruise to the Bahamas with his spouse because of fearing that the cruise ship will sink and all will drown. Using a cognitive theory perspective, the nurse should use which of these statements to explain to the spouse the etiology of this fear? A. "Your spouse may be unable to resolve internal conflicts, which result in projected anxiety." B. "Your spouse may be experiencing a distorted and unrealistic appraisal of the situation." C. "Your spouse may have a genetic predisposition to overreacting to potential danger." D. "Your spouse may have high levels of brain chemicals that may distort thinking."

ANS: B The nurse should explain that from a cognitive perspective the client is experiencing a distorted and unrealistic appraisal of the situation. From a cognitive perspective, fear is described as the result of faulty cognitions. KEY: Cognitive Level: Analysis | Integrated Processes: Nursing Process: Implementation | Client Need: Psychosocial Integrity

A client diagnosed with bipolar I disorder is distraught over insomnia experienced over the last 3 nights and a 12-pound weight loss over the past 2 weeks. Which should be this client's priority nursing diagnosis? A. Knowledge deficit R/T bipolar disorder AEB concern about symptoms B. Altered nutrition: less than body requirements R/T hyperactivity AEB weight loss C. Risk for suicide R/T powerlessness AEB insomnia and anorexia D. Altered sleep patterns R/T mania AEB insomnia for the past 3 nights

ANS: B The nurse should identify that the priority nursing diagnosis for this client is altered nutrition: less than body requirements R/T hyperactivity AEB weight loss. Due to the client's rapid weight loss, the nurse should prioritize interventions to ensure proper nutrition and health. KEY: Cognitive Level: Analysis | Integrated Processes: Nursing Process: Analysis | Client Need: Physiological Integrity

A homeless client comes to an emergency department reporting cough, night sweats, weight loss, and blood-tinged sputum. What disease that has recently become more prevalent among the homeless community should a nurse suspect? A. Meningitis B. Tuberculosis C. Encephalopathy D. Mononucleosis

ANS: B The nurse should suspect that the homeless client has contracted tuberculosis. Tuberculosis is a growing problem among homeless individuals because of being in crowded shelters, which are ideal conditions for the spread of respiratory tuberculosis. Alcoholism, drug addiction, HIV infection, and poor nutrition also contribute to the increase in cases of tuberculosis. KEY: Cognitive Level: Application | Integrated Processes: Nursing Process: Assessment | Client Need: Physiological Integrity

An anorexic client states to a nurse, "My father has recently moved back to town." Since that time the client has experienced insomnia, nightmares, and panic attacks that occur nightly. She has never married or dated and lives alone. What should the nurse suspect? A. Possible major depressive disorder B. Possible history of childhood incest C. Possible histrionic personality disorder D. Possible history of childhood bulimia

ANS: B The nurse should suspect that this client might have a history of childhood incest. Adult survivors of incest are at risk for developing posttraumatic stress disorder, sexual dysfunction, somatization disorders, compulsive sexual behavior disorders, depression, anxiety, eating disorders, and substance abuse disorders. KEY: Cognitive Level: Application | Integrated Processes: Nursing Process: Assessment | Client Need: Psychosocial Integrity

A nursing instructor is teaching about specific phobias. Which student statement should indicate that learning has occurred? A. "These clients do not recognize that their fear is excessive, and they rarely seek treatment." B. "These clients have overwhelming symptoms of panic when exposed to the phobic stimulus." C. "These clients experience symptoms that mirror a cerebrovascular accident (CVA)." D. "These clients experience the symptoms of tachycardia, dysphagia, and diaphoresis."

ANS: B The nursing instructor should evaluate that learning has occurred when the student knows that clients experiencing phobias have a panic level of fear that is overwhelming and unreasonable. Phobia is fear cued by a specific object or situation in which exposure to the stimulus produces an immediate anxiety response. KEY: Cognitive Level: Application | Integrated Processes: Nursing Process: Evaluation | Client Need: Psychosocial Integrity

5. A client diagnosed with borderline personality disorder brings up a conflict with the staff in a community meeting and develops a following of clients who unreasonably demand modification of unit rules. How can the nursing staff best handle this situation? A. Allow the clients to apply the democratic process when developing unit rules. B. Maintain consistency of care by open communication to avoid staff manipulation. C. Allow the client spokesperson to verbalize concerns during a unit staff meeting. D. Maintain unit order by the application of autocratic leadership.

ANS: B The nursing staff can best handle this situation by maintaining consistency of care by open communication to avoid staff manipulation. Clients diagnosed with borderline personality disorder can exhibit negative patterns of interaction, such as clinging and distancing, splitting, manipulation, and self-destructive behaviors.

A nursing instructor is teaching about the prevalence of bipolar disorder. Which student statement indicates that learning has occurred? A. "This disorder is more prevalent in the lower socioeconomic groups." B. "This disorder is more prevalent in the higher socioeconomic groups." C. "This disorder is equally prevalent in all socioeconomic groups." D. "This disorder's prevalence cannot be evaluated on the basis of socioeconomic groups."

ANS: B The nursing student is accurate when stating that bipolar disorder is more prevalent in higher socioeconomic groups. Theories consider both hereditary and environmental factors in the etiology of bipolar disorder. KEY: Cognitive Level: Application | Integrated Processes: Nursing Process: Evaluation | Client Need: Health Promotion and Maintenance

A client has the following symptoms: preoccupation with imagined defect, verbalizations that are out of proportion to actual physical abnormalities, and numerous visits to plastic surgeons to seek relief. Which nursing diagnosis would best describe the problems evidenced by these symptoms? A. Ineffective coping B. Disturbed body image C. Complicated grieving D. Panic anxiety

ANS: B The symptoms presented describe the DSM-5 diagnosis of body dysmorphic disorder, and the related nursing diagnosis is disturbed body image. KEY: Cognitive Level: Analysis | Integrated Processes: Nursing Process: Analysis | Client Need: Psychosocial Integrity

A client has been taking lithium for several years with good symptom control. The client presents in the emergency department with blurred vision, tinnitus, and severe diarrhea. The nurse should correlate these symptoms with which lithium level? A. 1.3 mEq/L B. 1.7 mEq/L C. 2.3 mEq/L D. 3.7 mEq/L

ANS: B The therapeutic level of lithium carbonate is 1.0 to 1.5 mEq/L for acute mania and 0.6 to 1.2 mEq/L for maintenance therapy. There is a narrow margin between the therapeutic and toxic levels. The symptoms presented in the question can be correlated with a lithium level of 1.7 mEq/L. Levels of 2.3 mEq/L and 3.7 mEq/L would produce more extreme symptoms of intensified toxicity, eventually leading to death. KEY: Cognitive Level: Application | Integrated Processes: Nursing Process: Evaluation | Client Need: Physiological Integrity: Pharmacological and Parenteral Therapies

13. Which factor has influenced the advancement in the treatment of mental illness? a. The movement of clients out of mental institutions to the community b. A better understanding of the neurobiology of mental illness c. More interest in helping persons with mental illness d. A change in the culture about what constitutes mental illness

ANS: B Two major movements have influenced the treatment of mental illness: consumer advocacy and a better understanding of neurobiology

28. A nurse is discussing treatment options with a client whose life has been negatively impacted by claustrophobia. The nurse would expect which of the following behavioral therapies to be most commonly used in the treatment of phobias? Select all that apply. A. Benzodiazepine therapy B. Systematic desensitization C. Imploding (flooding) D. Assertiveness training E. Aversion therapy

ANS: B, C The nurse should explain to the client that systematic desensitization and imploding are the most commonly used behavioral therapies in the treatment of phobias. Systematic desensitization involves the gradual exposure of the client to anxiety-provoking stimuli. Imploding is the intervention used in which the client is exposed to extremely frightening stimuli for prolonged periods of time.

A nurse is discussing treatment options with a client whose life has been negatively impacted by claustrophobia. The nurse would expect which of the following behavioral therapies to be most commonly used in the treatment of phobias? Select all that apply. A. Benzodiazepine therapy B. Systematic desensitization C. Imploding (flooding) D. Assertiveness training E. Aversion therapy

ANS: B, C The nurse should explain to the client that systematic desensitization and imploding are the most commonly used behavioral therapies in the treatment of phobias. Systematic desensitization involves the gradual exposure of the client to anxiety-provoking stimuli. Imploding is the intervention used in which the client is exposed to extremely frightening stimuli for prolonged periods of time. KEY: Cognitive Level: Application | Integrated Processes: Nursing Process: Implementation | Client Need: Psychosocial Integrity

14. A client living on the beachfront seeks help with an extreme fear of crossing bridges, which interferes with daily life. A psychiatric nurse practitioner decides to try systematic desensitization. Which explanation of this therapy should the nurse convey to the client? A. Using your imagination, we will attempt to achieve a state of relaxation that you can replicate when faced with crossing a bridge. B. Because anxiety and relaxation are mutually exclusive states, we can attempt to substitute a relaxation response for the anxiety response. C. Through a series of increasingly anxiety-provoking steps, we will gradually increase your tolerance to anxiety. D. In one intense session, you will be exposed to a maximum level of anxiety that you will learn to tolerate.

ANS: C The nurse should explain to the client that systematic desensitization exposes the client to a series of increasingly anxiety-provoking steps that will gradually increase anxiety tolerance. Systematic desensitization was introduced by Joseph Wolpe in 1958 and is based on behavioral conditioning principles.

5. Which treatment should a nurse identify as most appropriate for clients diagnosed with generalized anxiety disorder (GAD)? A. Long-term treatment with diazepam (Valium) B. Acute symptom control with citalopram (Celexa) C. Long-term treatment with buspirone (BuSpar) D. Acute symptom control with ziprasidone (Geodon)

ANS: C The nurse should identify that an appropriate treatment for clients diagnosed with GAD is long-term treatment with buspirone. Buspirone is an anxiolytic medication that is effective in 60% to 80% of clients with GAD. It takes 10 to 14 days for alleviation of symptoms but does not have the dependency concerns of other anxiolytics.

An inpatient client is newly diagnosed with dissociative identity disorder (DID) stemming from severe childhood sexual abuse. Which nursing intervention takes priority? A. Encourage exploration of sexual abuse B. Encourage guided imagery C. Establish trust and rapport D. Administer antianxiety medications

ANS: C The nurse should prioritize establishing trust and rapport when beginning to work with a client diagnosed with dissociative identity disorder. DID was formerly called multiple personality disorder. Each personality views itself as a separate entity and must be treated as such to establish rapport. Trust is the basis of every therapeutic relationship.

1. An angry client on an inpatient unit approaches a nurse, stating, Someone took my lunch! People need to respect others, and you need to do something about this now! The nurses response should be guided by which basic assumption of milieu therapy? A. Conflict should be avoided at all costs on inpatient psychiatric units. B. Conflict should be resolved by the nursing staff. C. Every interaction is an opportunity for therapeutic intervention. D. Conflict resolution should be addressed only during group therapy.

ANS: C The nurses response should be guided by the basic assumption that every interaction is an opportunity for therapeutic intervention. The nurse can utilize milieu therapy to effect behavioral change and improve psychological health and functioning.

13. A college student is unable to take a final examination because of severe test anxiety. Instead of studying, the student relieves stress by attending a movie. Which priority nursing diagnosis should a campus nurse assign for this client? A. Noncompliance R/T test taking B. Ineffective role performance R/T helplessness C. Altered coping R/T anxiety D. Powerlessness R/T fear

ANS: C The priority nursing diagnosis for this client is altered coping R/T anxiety. The nurse should assist in implementing interventions that should improve the clients healthy coping skills and reduce anxiety.

25. A client diagnosed with generalized anxiety states, I know the best thing for me to do now is to just forget my worries. How should the nurse evaluate this statement? A. The client is developing insight. B. The clients coping skills are improving. C. The client has a distorted perception of problem resolution. D. The client is meeting outcomes and moving toward discharge.

ANS: C This client has a distorted perception of how to deal with the problem of anxiety. Clients should be encouraged to openly deal with anxiety and recognize the triggers that precipitate anxiety responses.

A client is diagnosed with bipolar disorder and admitted to an inpatient psychiatric unit. Which is the priority outcome for this client? A. The client will accomplish activities of daily living independently by discharge. B. The client will verbalize feelings during group sessions by discharge. C. The client will remain safe throughout hospitalization. D. The client will use problem-solving to cope adequately after discharge.

ANS: C A client diagnosed with bipolar disorder is at risk for injury in either pole of this disorder. In the manic phase the client is hyperactive and can injure self inadvertently, and in the depressive phase the client can be at risk for suicide. KEY: Cognitive Level: Application | Integrated Processes: Nursing Process: Planning | Client Need: Safe and Effective Care Environment

A nursing instructor is teaching students about the Community Health Centers Act of 1963. What was a deterring factor to the proper implementation of this act? A. Many perspective clients did not meet criteria for mental illness diagnostic-related groups. B. Zoning laws discouraged the development of community mental health centers. C. States could not match federal funds to establish community mental health centers. D. There was not a sufficient employment pool to staff community mental health centers.

ANS: C A deterring factor to the proper implementation of the Community Mental Health Centers Act of 1963 was that states could not match federal funds to establish community mental health centers. This act called for the construction of comprehensive community mental health centers to offset the effects of deinstitutionalization caused by the closing of state mental health hospitals. KEY: Cognitive Level: Application | Integrated Processes: Nursing Process: Evaluation | Client Need: Safe and Effective Care Environment

7. Which adult client should a nurse identify as exhibiting the characteristics of a dependent personality disorder? A. A physically healthy client who is dependent on meeting social needs by contact with 15 cats B. A physically healthy client who has a history of depending on intense relationships to meet basic needs C. A physically healthy client who lives with parents and relies on public transportation D. A physically healthy client who is serious, inflexible, perfectionistic, and depends on rules to provide security

ANS: C A physically healthy adult client who lives with parents and relies on public transportation exhibits signs of dependent personality disorder. Dependent personality disorder is characterized by a pervasive and excessive need to be taken care of that leads to submissive and clinging behavior.

8. Approximately what percentage of the adult population (ages 18 and older) suffers from a mental disorder in a given year in the United States? a. 6 b. 14 c. 26 d. 30

ANS: C Approximately 26% of the adult population has a mental disorder in the United States.

1. A mother brings her son to the Emergency Department and tells the nurse that her son must have PTSD, because 2 days ago he witnessed a car accident in which there were fatalities. She is convinced that her son has PTSD because he has been crying when he talks about the incident. She believes that boys are at greater risk for PTSD because they dont typically cry. She read on the internet that PTSD can have dangerous consequences, so she wants her son to get some medication to cure the PTSD before it gets too bad. Which of these statements by the nurse would accurately correct this mothers misunderstanding about PTSD? Select all that apply. A. There are no long-term or dangerous consequences from PTSD. B. Women appear to be at greater risk of this disorder than men. C. Medications have been found to be effective in treating symptoms of depression or anxiety but do not represent a cure for the disorder. D. Fewer than 10% of trauma victims develop PTSD.

ANS: B, C, D Items B, C, and D are evidence-based pieces of information. Item A is incorrect since, in fact, dangerous consequences of unmanaged PTSD may include depression and/or suicide.

11. A nurse attends an interdisciplinary team meeting on an inpatient unit. Which of the following individuals are typically included as members of the interdisciplinary treatment team in psychiatry? Select all that apply. A. Respiratory therapist B. Occupational therapist C. Recreational therapist D. Social worker E. Mental health technician

ANS: B, C, D, E The typical interdisciplinary treatment team in a psychiatric inpatient setting consists of a psychiatrist, psychiatric nurse, psychiatric social worker, music therapist, dietician, psychologist, occupational therapist, recreational therapist, art therapist, mental health technician, and chaplain. Other disciplines may be included on the basis of resources available in a particular hospital setting and individual patient needs.

15. A nurse categorizes mental illness as a biopsychosocial disorder. What is meant by this classification? a. Mental illness is an abnormal brain vasculature that can be detected with angiography. b. Antipsychotic drugs can be used for all types of mental illness. c. Experience and psychosocial factors affect the etiology and treatment of mental illness. d. Mental illness can be cured with the right drugs and therapy.

ANS: C Biochemical changes of the brain are being studied as causes of mental illness.

This is most common comorbid condition in children with bipolar disorder

ADHD

How do meds for ADHD affect bipolar symptoms in children?

ADHD agents may exacerbate mania and should be administered only after bipolar symptoms have been controlled.

13. A military vet who recently returned from active duty in a Middle Eastern country and suffers from PTSD states he will not allow the lab tech, who is Iranian, to draw his blood. The patient states Hell probably use a contaminated needle on me. Which of these is the most appropriate response by the nurse? A. Let me see if I can arrange for a different technician to draw your blood. B. Let me help you overcome your cultural bias by letting him draw your blood. C. There is no other technician, so youre just going to have to let him draw your blood. D. I dont think the technician is really Middle Eastern.

ANS: A Item A demonstrates acceptance of the patient and attempts to create a less threatening situation for the patient. Item B makes an unsubstantiated assumption about the patients biases. Item C will not contribute to the patients sense of control, and sense of comfort and control is important in managing symptoms of PTSD. Item D minimizes the patients concerns rather than responding empathically to them.

7. Arthur, who is diagnosed with obsessive-compulsive disorder, reports to the nurse that he cant stop thinking about all the potentially life threatening germs in the environment. What is the most accurate way for the nurse to document this symptom? A. Patient is expressing an obsession with germs. B. Patient is manifesting compulsive thinking. C. Patient is expressing delusional thinking about germs. D. Patient is manifesting arachnophobia of germs.

ANS: A Obsessions are unwanted, intrusive, repetitive thoughts. Compulsions are unwanted, repetitive behavior patterns in response to obsessive thoughts that are efforts to reduce anxiety.

16. A nurse is providing discharge teaching to a client taking a benzodiazepine. Which client statement would indicate a need for further follow-up instructions? A. I will need scheduled bloodwork in order to monitor for toxic levels of this drug. B. I wont stop taking this medication abruptly, because there could be serious complications. C. I will not drink alcohol while taking this medication. D. I wont take extra doses of this drug because I can become addicted.

ANS: A The client indicates a need for additional information about taking benzodiazepines when stating the need for blood work to monitor for toxic levels. No blood work is needed when taking a short-acting benzodiazepine. The client should understand that taking extra doses of a benzodiazepine may result in addiction and that the drug should not be taken in conjunction with alcohol.

The family of a client diagnosed with conversion disorder asks the nurse, Will his paralysis ever go away? Which of these responses by the nurse is evidence-based? A. Most symptoms of conversion disorder resolve within a few weeks. B. Typically people who have conversion disorder symptoms that include paralysis will be paralyzed for the rest of their lives. C. The only people who recover are those that develop conversion disorder symptoms without a precipitating stressful event. D. Technically, he could walk now since he is intentionally feigning paralysis.

ANS: A The evidence supports that most conversion disorder symptoms resolve within a few weeks, and about 20% will have a relapse within 1 year.

9. A client diagnosed with panic disorder states, When an attack happens, I feel like I am going to die. Which is the most appropriate nursing reply? A. I know its frightening, but try to remind yourself that this will only last a short time. B. Death from a panic attack happens so infrequently that there is no need to worry. C. Most people who experience panic attacks have feelings of impending doom. D. Tell me why you think you are going to die every time you have a panic attack.

ANS: A The most appropriate nursing reply to the clients concerns is to empathize with the client and provide encouragement that panic attacks last only a short period. Panic attacks usually last minutes but can, rarely, last hours. Symptoms of depression are also common with this disorder.

11. Neurological tests have ruled out pathology in a clients sudden lower-extremity paralysis. Which nursing care should be included for this client? A. Deal with physical symptoms in a detached manner. B. Challenge the validity of physical symptoms. C. Meet dependency needs until the physical limitations subside. D. Encourage a discussion of feelings about the lower-extremity problem.

ANS: A The nurse should assist the client in dealing with physical symptoms in a detached manner to avoid reinforcing the symptoms by providing secondary gains. This is an example of a conversion disorder in which symptoms affect voluntary motor or sensory functioning. Examples include paralysis, aphonia, seizures, coordination disturbance, difficulty swallowing, urinary retention, akinesia, blindness, deafness, double vision, anosmia, and hallucinations.

2. A client has a history of excessive fear of water. What is the term that a nurse should use to describe this specific phobia, and under what subtype is this phobia identified? A. Aquaphobia, a natural environment type of phobia B. Aquaphobia, a situational type of phobia C. Acrophobia, a natural environment type of phobia D. Acrophobia, a situational type of phobia

ANS: A The nurse should determine that an excessive fear of water is identified as aquaphobia, which is a natural environment type of phobia. Natural environmenttype phobias are fears about objects or situations that occur in the natural environment, such as a fear of heights or storms.

3. A newly admitted client asks, Why do we need a unit schedule? Im not going to these groups. Im here to get some rest. Which is the most appropriate nursing reply? A. Group therapy provides the opportunity to learn and practice new coping skills. B. Group therapy is mandatory. All clients must attend. C. Group therapy is optional. You can go if you find the topic helpful and interesting. D. Group therapy is an economical way of providing therapy to many clients concurrently.

ANS: A The nurse should explain to the client that the purpose of group therapy is to learn and practice new coping skills. A basic assumption of milieu therapy is that every interaction, including group therapy, is an opportunity for therapeutic intervention.

21. A client is prescribed alprazolam (Xanax) for acute anxiety. What client history should cause a nurse to question this order? A. History of alcohol dependence B. History of personality disorder C. History of schizophrenia D. History of hypertension

ANS: A The nurse should question a prescription of alprazolam (Xanax) for acute anxiety if the client has a history of alcohol dependence. Alprazolam is a benzodiazepine used in the treatment of anxiety and has an increased risk for physiological dependence and tolerance. A client with a history of substance abuse may be more likely to abuse other addictive substances and/or combine this drug with alcohol.

A nursing instructor is teaching about the etiology of dissociative disorders from a psychoanalytical perspective. What student statement about clients diagnosed with this disorder indicates that learning has occurred? A. Dissociative behaviors occur when individuals repress distressing mental information from their conscious awareness. B. When their physical symptoms relieve them from stressful situations, their amnesia is reinforced. C. People with dissociative disorders typically have strong egos. D. There is clear and convincing evidence of a familial predisposition to this disorder.

ANS: A The nurse should understand that from a psychoanalytical perspective, dissociation occurs because of repression of painful information or experiences.

A nurse is working with a client diagnosed with somatic symptom disorder. What predominant symptoms should a nurse expect to assess? A. Disproportionate and persistent thoughts about the seriousness of ones symptoms B. Amnestic episodes in which the client is pain free C. Excessive time spent discussing psychosocial stressors D. Lack of physical symptoms

ANS: A The primary focus in somatic symptom disorder is on physical symptoms that suggest medical disease but which have no basis in organic pathology. Although the symptoms are associated with psychosocial distress, the individual focuses on the seriousness of the physical symptoms rather than the underlying psychosocial issues.

5. To promote self-reliance, how should a psychiatric nurse best conduct medication administration? A. Encourage clients to request their medications at the appropriate times. B. Refuse to administer medications unless clients request them at the appropriate times. C. Allow the clients to determine appropriate medication times. D. Take medications to the clients bedside at the appropriate times.

ANS: A The psychiatric nurse promoting self-reliance would encourage clients to request their medications at the appropriate times. Nurses are responsible for the management of medication administration on inpatient psychiatric units; however, nurses must work with clients to foster independence and provide experiences that would foster increased self-esteem.

10. A nursing instructor is teaching about the medications used to treat panic disorder. Which student statement indicates that learning has occurred? A. Clonazepam (Klonopin) is particularly effective in the treatment of panic disorder. B. Clozapine (Clozaril) is used off-label in long-term treatment of panic disorder. C. Doxepin (Sinequan) can be used in low doses to relieve symptoms of panic attacks. D. Buspirone (BuSpar) is used for its immediate effect to lower anxiety during panic attacks.

ANS: A The student indicates learning has occurred when he or she states that clonazepam is a particularly effective treatment for panic disorder. Clonazepam is a type of benzodiazepine that can be abused and lead to physical dependence and tolerance. It can be used on an as-needed basis to reduce anxiety and its related symptoms.

30. Which nursing statement reflects a common characteristic of a client diagnosed with paranoid personality disorder? A. "This client consistently criticizes care and has difficulty getting along with others." B. "This client is shy and fades into the background." C. "This client expects special treatment, and setting limits will be necessary." D. "This client is expressive during group and is very pleased with self."

ANS: A A client diagnosed with paranoid personality disorder has a pervasive distrust and suspiciousness of others. Anticipating humiliation and betrayal, the paranoid individual characteristically learns to attack first.

26. When a client on an acute care psychiatric unit demonstrates behaviors and verbalizations indicating a lack of guilt feelings, which nursing intervention would help the client to meet desired outcomes? A. Provide external limits on client behavior. B. Foster discussions of rationales for behavioral change. C. Implement interventions consistently by only one staff member. D. Encourage the client to involve self in care.

ANS: A Because the client, due to a lack of guilt, cannot or will not impose personal limits on maladaptive behaviors, these limits must be delineated and enforced by staff.

A client is admitted in a manic episode of bipolar I disorder. Which nursing intervention should be most therapeutic for this client? A. Using a calm, unemotional approach during client interactions B. Focusing primarily on enforcing limits C. Limiting interactions to decrease external stimuli D. Encouraging the client to establish social relationships with peers

ANS: A Clients experiencing mania are subject to frequent mood variations, easily changing from irritability and anger to sadness and crying. Therefore, it is necessary to maintain a calm, unemotional approach during client interactions. KEY: Cognitive Level: Application | Integrated Processes: Nursing Process: Implementation | Client Need: Psychosocial Integrity

27. Which characteristics should a nurse recognize as being exhibited by individuals diagnosed with any personality disorder? A. These clients accept and are comfortable with their altered behaviors. B. These clients understand that their altered behaviors result from anxiety. C. These clients seek treatment to avoid interpersonal discomfort. D. These clients avoid relationships due to past negative experiences.

ANS: A Clients who are diagnosed with personality disorders accept and are comfortable with their altered behaviors. Personalities that develop in a disordered pattern remain somewhat unstable and unpredictable throughout the lifetime.

1. Who led reform efforts to correct types of inhumane practices in the care of criminals, those with mental disorders, and victims of the Civil War? a. Dorothea Dix b. Philippe Pinel c. Benjamin Rush d. Clifford Beers

ANS: A Dorothea Dix led reform efforts to correct types of inhumane practices in the care of criminals, those with mental disorders, and victims of the Civil War.

A nurse begins the intake assessment of a client diagnosed with bipolar I disorder. The client shouts, "You can't do this to me. Do you know who I am?" Which is the priority nursing action in this situation? A. To provide self and client with a safe environment B. To redirect the client to the needed assessment information C. To provide high-calorie finger foods to meet nutritional needs D. To reorient the client to person, place, time, and situation

ANS: A During a manic episode the client's mood is elevated, expansive, and irritable. Providing a safe environment should be prioritized to protect the client and staff from potential injury. KEY: Cognitive Level: Application | Integrated Processes: Nursing Process: Implementation | Client Need: Safe and Effective Care Environment

12. A client who was deinstitutionalized during the 1960s may have experienced difficulties living in the community because of: a. Insufficient community mental health services b. Difficulties connecting with community resources c. Lack of affordable housing d. Increased cost of care

ANS: A Families were not prepared for treatment responsibilities. There were no education and support programs for families. Staff in nursing homes lacked skills to treat people with mental disorders, and clients had little or no supervision in independent settings.

3. Many clients with mental disorders were institutionalized because: a. There was a continued fear of people with mental disorders. b. The success rate was higher with hospital treatment. c. It was easier to stabilize clients with psychotropic medications. d. It provided a less stressful environment for clients to recuperate.

ANS: A Many clients with mental disorders were institutionalized because of a continued fear of people with mental disorders.

Arthur, who is diagnosed with obsessive-compulsive disorder, reports to the nurse that he can't stop thinking about all the potentially life threatening germs in the environment. What is the most accurate way for the nurse to document this symptom? A. Patient is expressing an obsession with germs. B. Patient is manifesting compulsive thinking. C. Patient is expressing delusional thinking about germs. D. Patient is manifesting arachnophobia of germs.

ANS: A Obsessions are unwanted, intrusive, repetitive thoughts. Compulsions are unwanted, repetitive behavior patterns in response to obsessive thoughts that are efforts to reduce anxiety. KEY: Cognitive Level: Analysis | Integrated Processes: Communication and Documentation | Client Need: Psychosocial Integrity

Providing nursing education on drug abuse to a high school class is an example of which level of preventive care? A. Primary prevention B. Secondary prevention C. Tertiary prevention D. Primary intervention

ANS: A Providing nursing education on drug abuse to a high school class is an example of primary prevention. Primary prevention services are aimed at reducing the incidence of mental health disorders within the population. KEY: Cognitive Level: Application | Integrated Processes: Nursing Process: Implementation | Client Need: Health Promotion and Maintenance

23. When a nurse is involved with a group who advocates for improved and effective psychiatric services and consumer empowerment, the nurse is working with the: a. National Alliance for Mentally Ill (NAMI) Consumer Council b. National Mental Health Consumers Association (NMHCA) c. National Association of Psychiatric Survivors (NAPS) d. Consumer/Survivor Mental Health Research and Policy Work Group

ANS: A The NAMI Consumer Counsel advocates for improved and effective psychiatric services and consumer empowerment.

6. Which nursing approach should be used to maintain a therapeutic relationship with a client diagnosed with borderline personality disorder? A. Being firm, consistent, and empathetic, while addressing specific client behaviors B. Promoting client self-expression by implementing laissez-faire leadership C. Using authoritative leadership to help clients learn to conform to societal norms D. Overlooking inappropriate behaviors to avoid promoting secondary gains

ANS: A The best nursing approach when working with a client diagnosed with borderline personality disorder is to be firm, consistent, and empathetic while addressing specific client behaviors. Individuals diagnosed with borderline personality disorder always seem to be in a state of crisis and can often have negative patterns of interaction, such as manipulation and splitting.

A nurse is providing discharge teaching to a client taking a benzodiazepine. Which client statement would indicate a need for further follow-up instructions? A. "I will need scheduled bloodwork in order to monitor for toxic levels of this drug." B. "I won't stop taking this medication abruptly, because there could be serious complications." C. "I will not drink alcohol while taking this medication." D. "I won't take extra doses of this drug because I can become addicted."

ANS: A The client indicates a need for additional information about taking benzodiazepines when stating the need for blood work to monitor for toxic levels. No blood work is needed when taking a short-acting benzodiazepine. The client should understand that taking extra doses of a benzodiazepine may result in addiction and that the drug should not be taken in conjunction with alcohol. KEY: Cognitive Level: Application | Integrated Processes: Nursing Process: Evaluation | Client Need: Physiological Integrity: Pharmacological and Parenteral Therapies

A college student was sexually assaulted when out on a date. After several weeks of crisis intervention therapy, which client statement should indicate to a nurse that the student is handling this situation in a healthy manner? A. "I know that it was not my fault." B. "My boyfriend has trouble controlling his sexual urges." C. "If I don't put myself in a dating situation, I won't be at risk." D. "Next time I will think twice about wearing a sexy dress."

ANS: A The client who realizes that sexual assault was not her fault is handling the situation in a healthy manner. The nurse should provide nonjudgmental listening and communicate statements that instill trust and validate self-worth. KEY: Cognitive Level: Application | Integrated Processes: Nursing Process: Evaluation | Client Need: Psychosocial Integrity

A client diagnosed with panic disorder states, "When an attack happens, I feel like I am going to die." Which is the most appropriate nursing reply? A. "I know it's frightening, but try to remind yourself that this will only last a short time." B. "Death from a panic attack happens so infrequently that there is no need to worry." C. "Most people who experience panic attacks have feelings of impending doom." D. "Tell me why you think you are going to die every time you have a panic attack."

ANS: A The most appropriate nursing reply to the client's concerns is to empathize with the client and provide encouragement that panic attacks last only a short period. Panic attacks usually last minutes but can, rarely, last hours. Symptoms of depression are also common with this disorder. KEY: Cognitive Level: Application | Integrated Processes: Nursing Process: Implementation | Client Need: Psychosocial Integrity

A client on the inpatient unit tells a student nurse, "My life has no purpose. I can't think about living another day, but please don't tell anyone about the way I feel. I know you are obligated to protect my confidentiality." Which is the most appropriate reply by the student nurse? A. "The treatment team is composed of many specialists who are working to improve your ability to function. Sharing this information with the team is critical to your care." B. "Let's discuss steps that will resolve negative lifestyle choices that may increase your suicidal risk." C. "You seem to be preoccupied with self. You should concentrate on hope for the future." D. "This information is secure with me because of client confidentiality."

ANS: A The most appropriate response by the student nurse is to explain that sharing the information with the treatment team is critical to the client's care. The nurse's priority is to ensure client safety and to inform others of the client's suicidal ideation. KEY: Cognitive Level: Application | Integrated Processes: Nursing Process: Implementation | Client Need: Safe and Effective Care Environment

Which statement made by an emergency department nurse indicates accurate knowledge of domestic violence? A. "Power and control are central to the dynamic of domestic violence." B. "Poor communication and social isolation are central to the dynamic of domestic violence." C. "Erratic relationships and vulnerability are central to the dynamic of domestic violence." D. "Emotional injury and learned helplessness are central to the dynamic of domestic violence."

ANS: A The nurse accurately states that power and control are central to the dynamic of domestic violence. Battering is defined as a pattern of coercive control founded on physical and/or sexual violence or threat of violence. The typical abuser is very possessive and perceives the victim as a possession. KEY: Cognitive Level: Application | Integrated Processes: Assessment | Client Need: Psychosocial Integrity

A kindergarten student is frequently violent toward other children. A school nurse notices bruises and burns on the child's face and arms. What other symptom should indicate to the nurse that the child might have been physically abused? A. The child shrinks at the approach of adults. B. The child begs or steals food or money. C. The child is frequently absent from school. D. The child is delayed in physical and emotional development.

ANS: A The nurse should determine that a child who shrinks at the approach of adults in addition to having bruises and burns might be a victim of abuse. Whether or not the adult intended to harm the child, maltreatment should be considered. KEY: Cognitive Level: Application | Integrated Processes: Nursing Process: Assessment | Client Need: Safe and Effective Care Environment

A client has a history of excessive fear of water. What is the term that a nurse should use to describe this specific phobia, and under what subtype is this phobia identified? A. Aquaphobia, a natural environment type of phobia B. Aquaphobia, a situational type of phobia C. Acrophobia, a natural environment type of phobia D. Acrophobia, a situational type of phobia

ANS: A The nurse should determine that an excessive fear of water is identified as aquaphobia, which is a natural environment type of phobia. Natural environment-type phobias are fears about objects or situations that occur in the natural environment, such as a fear of heights or storms. KEY: Cognitive Level: Application | Integrated Processes: Nursing Process: Assessment | Client Need: Psychosocial Integrity

9. Family members of a client ask a nurse to explain the difference between schizoid and avoidant personality disorders. Which is the appropriate nursing reply? A. "Clients diagnosed with avoidant personality disorder desire intimacy but fear it, and clients diagnosed with schizoid personality disorder prefer to be alone." B. "Clients diagnosed with schizoid personality disorder exhibit odd, bizarre, and eccentric behavior, whereas clients diagnosed with avoidant personality disorder do not." C. "Clients diagnosed with avoidant personality disorder are eccentric, and clients diagnosed with schizoid personality disorder are dull and vacant." D. "Clients diagnosed with schizoid personality disorder have a history of psychotic thought processes, whereas clients diagnosed with avoidant personality disorder remain based in reality."

ANS: A The nurse should educate the family that clients diagnosed with avoidant personality disorder desire intimacy but fear it, whereas clients diagnosed with schizoid personality disorder prefer to be alone. Avoidant personality disorder is characterized by an extreme sensitivity to rejection, which leads to social isolation. Schizoid personality disorder is characterized by a profound deficit in the ability to form personal relationships.

A client is diagnosed with bipolar I disorder: manic episode. Which nursing intervention would be implemented to achieve the outcome of "Client will gain 2 pounds by the end of the week?" A. Provide client with high-calorie finger foods throughout the day. B. Accompany client to cafeteria to encourage adequate dietary consumption. C. Initiate total parenteral nutrition to meet dietary needs. D. Teach the importance of a varied diet to meet nutritional needs.

ANS: A The nurse should provide the client with high-calorie finger foods throughout the day to help the client achieve the outcome of gaining 2 pounds by the end of the week. Because of hyperactivity, the client will have difficulty sitting still to consume large meals. KEY: Cognitive Level: Application | Integrated Processes: Nursing Process: Implementation | Client Need: Physiological Integrity

A client is prescribed alprazolam (Xanax) for acute anxiety. What client history should cause a nurse to question this order? A. History of alcohol dependence B. History of personality disorder C. History of schizophrenia D. History of hypertension

ANS: A The nurse should question a prescription of alprazolam (Xanax) for acute anxiety if the client has a history of alcohol dependence. Alprazolam is a benzodiazepine used in the treatment of anxiety and has an increased risk for physiological dependence and tolerance. A client with a history of substance abuse may be more likely to abuse other addictive substances and/or combine this drug with alcohol. KEY: Cognitive Level: Application | Integrated Processes: Nursing Process: Evaluation | Client Need: Psychosocial Integrity

Which assessment data should a school nurse recognize as signs of physical neglect? A. The child is often absent from school and seems apathetic and tired. B. The child is very insecure and has poor self-esteem. C. The child has multiple bruises on various body parts. D. The child has sophisticated knowledge of sexual behaviors.

ANS: A The nurse should recognize that a child who is often absent from school and seems apathetic and tired might be a victim of neglect. Other indicators of neglect are stealing food or money, lacking medical or dental care, being consistently dirty, lacking sufficient clothing, or stating that there is no one home to provide care. KEY: Cognitive Level: Application | Integrated Processes: Nursing Process: Assessment | Client Need: Safe and Effective Care Environment

A client who has been diagnosed with bipolar I disorder states, "God has taught me how to decode the Bible." A nurse should anticipate that which combination of medications would be ordered to address this client's symptoms? A. Lithium carbonate (Lithobid) and risperidone (Risperdal) B. Lithium carbonate (Lithobid) and carbamazepine (Tegretol) C. Valproic acid (Depakote) and sertraline (Zoloft) D. Valproic acid (Depakote) and lamotrigine (Lamictal)

ANS: A The patient who is experiencing psychosis (in this case, delusions of grandeur) may be benefited by the addition of an antipsychotic medication (risperidone) to the mood stabilizer (lithium). In addition, since lithium does not immediately reach therapeutic levels, the sedative properties of an antipsychotic may be useful in reducing agitation, hyperactivity, and/or insomnia. KEY: Cognitive Level: Application | Integrated Processes: Nursing Process: Implementation | Client Need: Physiological Integrity: Pharmacological and Parenteral Therapies

15. Which nursing diagnosis should be prioritized when providing care to a client diagnosed with paranoid personality disorder? A. Risk for violence: directed toward others R/T suspicious thoughts B. Risk for suicide R/T altered thought C. Altered sensory perception R/T increased levels of anxiety D. Social isolation R/T inability to relate to others

ANS: A The priority nursing diagnosis for a client diagnosed with paranoid personality disorder should be risk for violence: directed toward others R/T suspicious thoughts. Clients diagnosed with paranoid personality disorder have a pervasive distrust and suspiciousness of others that may result in hostile actions to protect self. They are often tense and irritable, which increases the likelihood of violent behavior.

A client diagnosed with bipolar disorder: depressive episode intentionally overdoses on sertraline (Zoloft). Family reports that the client has experienced anorexia, insomnia, and recent job loss. What should be the priority nursing diagnosis for this client? A. Risk for suicide R/T hopelessness B. Anxiety: severe R/T hyperactivity C. Imbalanced nutrition: less than body requirements R/T refusal to eat D. Dysfunctional grieving R/T loss of employment

ANS: A The priority nursing diagnosis for this client should be risk for suicide R/T hopelessness. The nurse should prioritize diagnoses on the basis of physical and safety needs. This client continues to be at risk for suicide related to an intentional Zoloft overdose. KEY: Cognitive Level: Analysis | Integrated Process: Nursing Process: Analysis | Client Need: Safe and Effective Care Environment: Management of Care

A nursing instructor is teaching about the medications used to treat panic disorder. Which student statement indicates that learning has occurred? A. "Clonazepam (Klonopin) is particularly effective in the treatment of panic disorder." B. "Clozapine (Clozaril) is used off-label in long-term treatment of panic disorder." C. "Doxepin (Sinequan) can be used in low doses to relieve symptoms of panic attacks." D. "Buspirone (BuSpar) is used for its immediate effect to lower anxiety during panic attacks."

ANS: A The student indicates learning has occurred when he or she states that clonazepam is a particularly effective treatment for panic disorder. Clonazepam is a type of benzodiazepine that can be abused and lead to physical dependence and tolerance. It can be used on an as-needed basis to reduce anxiety and its related symptoms. KEY: Cognitive Level: Application | Integrated Processes: Nursing Process: Evaluation | Client Need: Physiological Integrity: Pharmacological and Parenteral Therapies

6. Why must a nurse have specialized knowledge and skills in mental health illness and problems in order to provide effective nursing care for this population? a. Services for people with mental disorders are inadequate. b. The effects of mental health are universal. c. Mental health disorders affect small sectors of the population. d. Current health objectives cannot be met without the influence of nursing.

ANS: A There are two universal truths that require nurses to have specialized knowledge and skills in this area: services for people with mental disorders are inadequate in all countries, and mental illness has a major impact on families, communities, and nations.

A client diagnosed with bipolar disorder states, "I hate oatmeal. Let's get everybody together to do exercises. I'm thirsty and I'm burning up. Get out of my way; I have to see that guy." What should be the priority nursing action? A. Assess the client's vital signs. B. Offer to have the dietitian discuss food preferences. C. Encourage the client to lead the exercise program in the community meeting. D. Acknowledge the client briefly and then walk away.

ANS: A When assessing a client diagnosed with bipolar disorder, the nurse should not lose sight of the fact that co-occurring physical problems could be masked by hyperactive, manic, or both behaviors. The client's statement of "I'm thirsty and I'm burning up" could be a symptom of either infection or dehydration and must be assessed. KEY: Cognitive Level: Application | Integrated Processes: Nursing Process: Implementation | Client Need: Safe and Effective Care Environment: Management of CareANS: A When assessing a client diagnosed with bipolar disorder, the nurse should not lose sight of the fact that co-occurring physical problems could be masked by hyperactive, manic, or both behaviors. The client's statement of "I'm thirsty and I'm burning up" could be a symptom of either infection or dehydration and must be assessed. KEY: Cognitive Level: Application | Integrated Processes: Nursing Process: Implementation | Client Need: Safe and Effective Care Environment: Management of Care

6. A patient admitted to the hospital with PTSD is ordered the following medications. Which of these medications has a direct use in treating symptoms that are common in PTSD? Select all that apply. A. Alprazolam B. Propanolol C. Colace D. Dulcolax

ANS: A, B Alprazolam is an antianxiety agent and anxiety symptoms are common in PTSD. Propanolol is an antihypertensive medication and evidence has demonstrated its effectiveness in treating symptoms of PTSD, including nightmares, intrusive recollections, and insomnia. The last two medications are used to treat constipation, and this symptom is not directly related to PTSD.

4. A patients wife reports to the nurse that she was told her husbands PTSD may be related to cognitive problems. She is asking the nurse to explain what that means. Which of the following are accurate statements about the cognitive theory as it applies to PTSD? Select all that apply. A. People are vulnerable to trauma-related disorders when their fundamental beliefs are invalidated. B. Cognitive theory addresses the importance of how people think (or cognitively appraise) events. C. Dementia is a common symptom of PTSD. D. Amnesia is the biggest cognitive problem in PTSD and is the primary cause of trauma-related disorders.

ANS: A, B Both A and B address aspects of cognitive theory and its relevance in PTSD. Dementia includes cognitive symptoms but is not a symptom of PTSD. Amnesia does not cause PTSD but is a symptom of PTSD.

8. Joe, a patient being treated for PTSD, tells the nurse that his therapist is recommending cognitive therapy. He asks the nurse how thats supposed to help his nightmares. Which of these responses by the nurse provides accurate information about the benefits of this type of therapy? Select all that apply. A. The nightmares may be related to troubling thoughts and feelings; cognitive therapy will help you explore and modify those thoughts and feelings. B. It is designed to help you cope with anxiety, anger, and other feelings that may be related to your symptoms. C. It is designed to repeatedly expose you to the trauma you experienced so you can regain a sense of safety. D. Once you learn to repress these troubling feelings, the nightmares should cease.

ANS: A, B Both A and B are desired outcomes in cognitive therapy. Item C more aptly describes prolonged exposure therapy. D is incorrect because exploration and awareness (rather than repression) are fundamental to cognitive therapy.

Which of the following nursing diagnoses would be expected for an adult survivor of incest? Select all that apply. A. Low self-esteem B. Powerlessness C. Disturbed personal identity D. Knowledge deficit E. Noncompliance

ANS: A, B An adult survivor of incest would most likely have low self-esteem and a sense of powerlessness. Adult survivors of incest are at risk for developing post-traumatic stress disorder, sexual dysfunction, somatization disorders, compulsive sexual behavior disorders, depression, anxiety, eating disorders, and substance abuse disorders. Disturbed personal identity refers to an inability to distinguish between self and nonself and is seen in disorders such as autistic disorders, borderline personality disorders, dissociative disorders, and gender identity disorders. KEY: Cognitive Level: Application | Integrated Processes: Nursing Process: Analysis | Client Need: Psychosocial Integrity

Which of the following explanations should a nurse include when teaching parents why is it difficult to diagnose a child or adolescent exhibiting symptoms of bipolar disorder? Select all that apply. A. Bipolar symptoms are similar to attention deficit-hyperactivity disorder symptoms. B. Children are naturally active, energetic, and spontaneous. C. Neurotransmitter levels vary considerably in accordance with age. D. The diagnosis of bipolar disorder cannot be assigned prior to the age of 18. E. Genetic predisposition is not a reliable diagnostic determinant.

ANS: A, B It is difficult to diagnose a child or adolescent with bipolar disorder because bipolar symptoms are similar to attention deficit-hyperactivity disorder symptoms and because children are naturally active, energetic, and spontaneous. Symptoms may also be comorbid with other childhood disorders, such as conduct disorder. KEY: Cognitive Level: Application | Integrated Processes: Teaching/Learning | Client Need: Psychosocial Integrity

3. A military veteran is being assessed for outpatient therapy after he reports having problems at home and at work. Which of the symptoms that he describes are commonly associated with PTSD? Select all that apply. A. Ive been drinking and smoking pot daily. B. Ive been having trouble sleeping and I think Ive been having nightmares but I cant remember them. C. I slapped my wife when she was trying to hug me. D. Ive been having intense pain in the leg where I sustained a combat wound.

ANS: A, B, C Common symptoms associated with PTSD include substance abuse, sleep disturbances, nightmares, and aggression. Whereas the combat exposure and wounding could be described as traumas, the patients complaint of pain requires further physical assessment rather than assuming this symptom is related to PTSD.

Which of the following clients should a nurse recommend for a structured day program? Select all that apply. A. An acutely suicidal teenager who has had three previous suicide attempts B. A chronically mentally ill woman who has a history of medication noncompliance C. An elderly individual with end-stage Alzheimer's disease D. A depressed individual who is able to participate in a safety plan E. A client who is hearing voices that tell him or her to harm others

ANS: B, D The nurse should recommend a structured day program for a chronically mental ill woman who has a history of medication noncompliance and for a depressed individual who is able to participate in a safety plan. Day programs (also called partial hospitalizations) are designed to prevent institutionalization or to ease the transition from inpatient hospitalization to community living. KEY: Cognitive Level: Application | Integrated Processes: Nursing Process: Evaluation | Client Need: Safe and Effective Care Environment: Management of Care

12. Which of the following are accurate descriptors of a therapeutic community? Select all that apply. A. The unit schedule includes unlimited free time for personal reflection. B. Unit responsibilities are assigned according to client capabilities. C. A flexible schedule is determined by client needs. D. The individual is the sole focus of therapy. E. A democratic form of government exists.

ANS: B, E In a therapeutic community the unit responsibilities are assigned according to client capability, and a democratic form of government exists. Therapeutic communities are structured and provide therapeutic interventions that focus on communication and relationship-development skills.

9. Joshua recently moved into a dormitory to begin his freshman year in college. He was reprimanded by the dormitory supervisor for not properly disposing of food items and responded by throwing all of his belongings from a second story window while shouting obscenities. The campus police escorted him to campus health services, where he was diagnosed with an Adjustment Disorder with Disturbance of Conduct. Which of the following items in Joshuas history predispose him to this disorder? Select all that apply. A. Joshua reports that he doesnt have any friends in the dormitory. B. Joshuas family currently lives out of the country and are often difficult to reach. C. Joshua was notified the same day that he would have to withdraw from one of his classes because he didnt have the prerequisite credits needed to register for the class. D. Joshua has a higher than average GPA and is a member of The National Honor Society.

ANS: A, B, C Items A and B may suggest lack of available support systems, which is identified as a predisposing factor for Adjustment Disorders. Item C presents evidence of another stressor occurring in proximity to the reprimand from the dormitory supervisor, which may also predispose to the development of an Adjustment Disorder.

33. Which statements represent positive outcomes for clients diagnosed with narcissistic personality disorder? Select all that apply. A. The client will relate one empathetic statement toward another client in group, by day 2. B. The client will identify one personal limitation by day 1. C. The client will acknowledge one strength that another client possesses by day 2. D. The client will list four personal strengths by day 3. E. The client will list two lifetime achievements by discharge.

ANS: A, B, C The nurse should determine that appropriate outcomes for a client diagnosed with narcissistic personality disorder include relating empathetic statements to other clients, identifying one personal limitation, and acknowledging one strength in another client. An exaggerated sense of self-worth, a lack of empathy, and exploitation of others are characteristics of narcissistic personality disorder.

8. How does a democratic form of self-government in the milieu contribute to client therapy? A. By setting punishments for clients who violate the community rules B. By dealing with inappropriate behaviors as they occur C. By setting community expectations wherein all clients are treated on an equal basis D. By interacting with professional staff members to learn about therapeutic interventions

ANS: C A democratic form of self-government in the milieu contributes to client therapy by setting the expectation that all clients should be treated on an equal basis. Clients participate in the decision-making and problem-solving aspects that affect treatment setting. The norms, rules, and behavioral limits are established by the staff and clients. All individuals have input.

Which should the nurse recognize as an example of localized amnesia? A. A client cannot relate any lifetime memories, including personal identity. B. A client can relate family memories but has no recollection of a particular brother. C. A client cannot remember events surrounding a fatal car accident. D. A client whose home was destroyed by a tornado only remembers waking up in the hospital.

ANS: C An individual with localized amnesia is unable to recall all incidents associated with a stressful event.

Which of the following instructions regarding lithium therapy should be included in a nurse's discharge teaching? Select all that apply. A. Avoid excessive use of beverages containing caffeine. B. Maintain a consistent sodium intake. C. Consume at least 2,500 to 3,000 mL of fluid per day. D. Restrict sodium content. E. Restrict fluids to 1,500 mL per day.

ANS: A, B, C The nurse should instruct the client taking lithium to avoid excessive use of caffeine, maintain a consistent sodium intake, and consume at least 2,500 to 3,000 mL of fluid per day. The risk of developing lithium toxicity is high due to the narrow margin between therapeutic doses and toxic levels. KEY: Cognitive Level: Application | Integrated Processes: Nursing Process: Implementation | Client Need: Physiological Integrity: Pharmacological and Parenteral Therapies

30. A client who has been diagnosed with a phobic disorder asks the nurse if there are any medications that would be beneficial in treating phobic disorders. Which of the following would be accurate responses by the nurse? Select all that apply. A. Some antianxiety agents have been successful in treating social phobias. B. Some antidepressant agents have been successful in diminishing symptoms of agoraphobia and social anxiety disorder (social phobia). C. Specific phobias are generally not treated with medication unless accompanied by panic attacks. D. Beta-blockers have been used successfully to treat phobic responses to public performance.

ANS: A, B, C, D All of the listed pharmacological treatments are evidence-based treatments for phobic disorders.

3. A nurse is referring a client with a mental illness to a community group for additional information and support. Where would the nurse most likely refer a client? Select all that apply. a. Alcoholics Anonymous b. Obsessive-Compulsive Foundation c. Schizophrenics Anonymous d. Anxiety Disorders Association of America

ANS: A, B, C, D Alcoholics Anonymous, Obsessive-Compulsive Foundation, Schizophrenics Anonymous, and Anxiety Disorders Association of America are all sources of information and help for people with mental illness.

A client who has been diagnosed with a phobic disorder asks the nurse if there are any medications that would be beneficial in treating phobic disorders. Which of the following would be accurate responses by the nurse? Select all that apply. A. Some antianxiety agents have been successful in treating social phobias. B. Some antidepressant agents have been successful in diminishing symptoms of agoraphobia and social anxiety disorder (social phobia). C. Specific phobias are generally not treated with medication unless accompanied by panic attacks. D. Beta-blockers have been used successfully to treat phobic responses to public performance.

ANS: A, B, C, D All of the listed pharmacological treatments are evidence-based treatments for phobic disorders. KEY: Cognitive Level: Application | Integrated Processes: Teaching/Learning | Client Need: Physiological Integrity: Pharmacological and Parenteral Therapies

MULTIPLE RESPONSE 1. According to the Surgeon Generals report, how can people receive assistance for mental health problems? Select all that apply. a. Specialty mental health systems b. General medical or primary care sectors c. Human service sectors d. Voluntary support networks

ANS: A, B, C, D The Surgeon Generals report defined specialty mental health systems, general medical or primary care sectors, human service sectors, and voluntary support networks as ways through which people could receive assistance.

When planning care for women in abusive relationships, which of the following information is important for the nurse to consider? Select all that apply. A. It often takes several attempts before a woman leaves an abusive situation. B. Substance abuse is a common factor in abusive relationships. C. Until children reach school age, they are usually not affected by parental discord. D. Women in abusive relationships usually feel isolated and unsupported. E. Economic factors rarely play a role in the decision to stay in abusive relationships.

ANS: A, B, D When planning care for women who have been victims of domestic abuse, the nurse should be aware that it often takes several attempts before a woman leaves an abusive situation, that substance abuse is a common factor in abusive relationships, and that women in abusive relationships usually feel isolated and unsupported. Children can be affected by domestic violence from infancy, and economic factors often play a role in the victim's decision to stay. KEY: Cognitive Level: Applications | Integrated Processes: Nursing Process: Assessment | Client Need: Psychosocial Integrity

2. A nurse is using Healthy People 2020 as a guide to develop community programming for mental health. Which objectives for mental health would be found in this document? Select all that apply. a. Reduce suicide rates. b. Increase employment of persons with severe mental illness. c. Increase the proportion of children treated for mental health problems. d. Decrease the number of juvenile facilities that screen new admissions for mental health problems.

ANS: A, C Reducing suicide attempts and increasing the proportion of children with mental health problems who receive treatment are two objectives for mental health targeted by Healthy People 2020.

3. How would a nurse differentiate a client diagnosed with a social phobia from a client diagnosed with a schizoid personality disorder (SPD)? A. Clients diagnosed with social phobia can manage anxiety without medications, whereas clients diagnosed with SPD can manage anxiety only with medications. B. Clients diagnosed with SPD are distressed by the symptoms experienced in social settings, whereas clients diagnosed with social phobia are not. C. Clients diagnosed with social phobia avoid interactions only in social settings, whereas clients diagnosed with SPD avoid interactions in all areas of life. D. Clients diagnosed with SPD avoid interactions only in social settings, whereas clients diagnosed with social phobias tend to avoid interactions in all areas of life.

ANS: C Clients diagnosed with social phobia avoid interactions only in social settings, whereas clients diagnosed with SPD avoid interactions in all areas of life. Social phobia is an excessive fear of situations in which a person might do something embarrassing or be evaluated negatively by others.

11. Studies have suggested that re-experiencing a traumatic event can become an addiction of sorts. The evidence suggests that the reason for this is: A. People with PTSD often have addictive personalities. B. Perpetuating the traumatic experience yields secondary gains. C. The re-experiencing of trauma enhances production of endogenous opioid peptides. D. People with PTSD often have concurrent substance abuse issues.

ANS: C Hollander and Simeon (2008) report on studies suggesting that the release of endogenous opioid peptides can produce an addiction to the trauma. There is no evidence suggesting that addictive personality traits are responsible for chronicity in PTSD symptoms. Items B and D are possible outcomes in any individual with PTSD, but neither has been correlated to an addiction to re-experiencing trauma.

A client diagnosed with somatic symptom disorder is most likely to exhibit which personality disorder characteristics? A. Uses splitting and manipulation in relationships B. Is socially irresponsible, exploitative, and guiltless and disregards rights of others C. Expresses heightened emotionality, seductiveness, and strong dependency needs D. Uncomfortable in social situations; perceived as timid, withdrawn, cold, and strange

ANS: C It has been suggested that in somatic symptom disorder, there may be some overlapping of personality characteristics and features associated with histrionic personality disorder. These features include heightened emotionality, impressionistic thought and speech, seductiveness, strong dependency needs, and a preoccupation with symptoms and oneself. Somatic symptom disorder is characterized by the expression of multiple somatic complaints associated with psychosocial distress and without medical basis.

2. A patient is admitted to the community mental health center for outpatient therapy with a diagnosis of Adjustment Disorder. Which of the following subjective statements by the patient support this diagnosis? Select all that apply. A. I was divorced 3 months ago and I cant seem to cope. B. I was a victim of date rape 15 years ago when I was in college. C. My partner came home last week and told me he just didnt love me anymore. D. I failed one of my classes last month and I cant get motivated to register for my next semester.

ANS: A, C, D A diagnosis of Adjustment Disorder is appropriate when the stressors are related to relational conflict, where there are significant emotional or behavioral symptoms, and when the response occurs within 3 months after the onset of the stressor (and persists no longer than 6 months). Item B would be more aptly described as a traumatic event.

7. A patient who is being seen in the community mental health center for PTSD is being considered for EMDR (Eye Movement Desensitization and Reprocessing) therapy. The nurse is being asked to conduct an assessment to validate the patients appropriateness for this treatment. Which of the following pieces of data, collected by the nurse, are most important to document when determining appropriateness for treatment with EMDR? Select all that apply. A. The patient has a history of a seizure disorder. B. The patient has a history of ECT. C. The patient reports suicidal ideation with a plan. D. The patient has been using alcohol in increasing quantities over the last 3 months.

ANS: A, C, D Items A, C, and D are all factors that would contraindicate the use of EMDR. A history of ECT is not directly relevant in determining appropriateness for EMDR.

29. A nurse has been caring for a client diagnosed with generalized anxiety disorder (GAD). Which of the following nursing interventions would address this clients symptoms? Select all that apply. A. Encourage the client to recognize the signs of escalating anxiety. B. Encourage the client to avoid any situation that causes stress. C. Encourage the client to employ newly learned relaxation techniques. D. Encourage the client to cognitively reframe thoughts about situations that generate anxiety. E. Encourage the client to avoid caffeinated products.

ANS: A, C, D, E Nursing interventions that address GAD symptoms should include encouraging the client to recognize signs of escalating anxiety, to employ relaxation techniques, to cognitively reframe thoughts about anxiety-provoking situations, and to avoid caffeinated products. Avoiding situations that cause stress is not an appropriate intervention, because avoidance does not help the client overcome anxiety. Stress is a component of life and is not easily evaded.

Carly has been diagnosed with somatic symptom disorder. As the nurse is talking with Carly and her family, which of the following statements suggest primary or secondary gains that the physical symptoms are providing for the client? A. The family agrees that Carly began having physical symptoms after she lost her job. B. Carly states that even though medical tests have not found anything wrong, she is convinced her headaches are indicative of a brain tumor. C. Carlys mother reports that someone from the family stays with Carly each night because the physical symptoms are incapacitating. D. Carly states she noticed feeling hotter than usual the last time she had a headache.

ANS: C It is important for the nurse to identify gains that the symptoms might be providing for the client, since these can reinforce illness behavior. Having family attend to the patient when she is symptomatic could reinforce increased dependency and attention needs.

12. Sandy, a rape survivor, is being treated for PTSD. Which of these statements are good indications that Sally is beginning to recover from PTSD? A. I still have nightmares every night, but I dont always remember them anymore. B. Im not drinking as much alcohol as I had been over the last several months. C. This traumatic event immobilized me for awhile, but I have found imagery helpful in reducing my anxiety. D. All of the above.

ANS: C Item C demonstrates evidence of awareness of the impact the trauma had on Sandys life and demonstrates evidence of effective coping skills. Item A indicates continued presence of symptoms and possibly amnesia. Although item B may be evidence of a positive coping strategy, evaluation of recovery from PTSD must also include assessment for less symptoms such as nightmares and flashbacks.

22. Warrens college roommate actively resists going out with friends whenever they invite him. He says he cant stand to be around other people and confides to Warren They wouldnt like me anyway. Which disorder is Warrens roommate likely suffering from? A. Agoraphobia B. Mysophobia C. Social anxiety disorder (social phobia) D. Panic disorder

ANS: C Social anxiety disorder is an excessive fear of social situations R/T fear that one might do something embarrassing or be evaluated negatively by others.

A nurse has been caring for a client diagnosed with generalized anxiety disorder (GAD). Which of the following nursing interventions would address this client's symptoms? Select all that apply. A. Encourage the client to recognize the signs of escalating anxiety. B. Encourage the client to avoid any situation that causes stress. C. Encourage the client to employ newly learned relaxation techniques. D. Encourage the client to cognitively reframe thoughts about situations that generate anxiety. E. Encourage the client to avoid caffeinated products.

ANS: A, C, D, E Nursing interventions that address GAD symptoms should include encouraging the client to recognize signs of escalating anxiety, to employ relaxation techniques, to cognitively reframe thoughts about anxiety-provoking situations, and to avoid caffeinated products. Avoiding situations that cause stress is not an appropriate intervention, because avoidance does not help the client overcome anxiety. Stress is a component of life and is not easily evaded. KEY: Cognitive Level: Application | Integrated Processes: Nursing Process: Implementation | Client Need: Psychosocial Integrity

34. A nurse is caring for a group of clients within the DSM-5 Cluster B category of personality disorders. Which factors should the nurse consider when planning client care? Select all that apply. A. These clients have personality traits that are deeply ingrained and difficult to modify. B. These clients need medications to treat the underlying physiological pathology. C. These clients use manipulation, making the implementation of treatment problematic. D. These clients have poor impulse control that hinders compliance with a plan of care. E. These clients commonly have secondary diagnoses of substance abuse and depression.

ANS: A, C, D, E The nurse should consider that individuals diagnosed with cluster B-type personality disorders have deeply ingrained personality traits, use manipulation, have poor impulse control, and often have secondary diagnoses of substance abuse and/or depression. This cluster includes antisocial, borderline, histrionic, and narcissistic personality disorders.

A client is exhibiting symptoms of generalized amnesia. Which of the following questions should the nurse ask to confirm this diagnosis? Select all that apply. A. Can you tell me your name and where you live? B. Have you ever traveled suddenly or unexpectedly away from home? C. Have you recently experienced any traumatic event? D. Have you ever felt detached from your environment? E. Have you had any history of memory problems?

ANS: A, C, E An individual who has generalized amnesia has amnesia for his or her identity and total life history. The first question assesses orientation to identity. Items C and D assess for awareness of current issues and historical issues, respectively. Affirmative descriptions of either of these issues would rule out generalized amnesia.

A client is diagnosed with illness anxiety disorder. Which of the following symptoms is the client most likely to exhibit? Select all that apply. A. Obsessive-compulsive traits B. Pseudocyesis C. Disabling fear of having a serious illness D. Multiple pronounced physical symptoms E. Depression

ANS: A, C, E Illness anxiety disorder involves a preoccupation with and fear of having or acquiring a serious disease. Somatic symptoms are either absent or mild in intensity.

24. How should a nurse best describe the major maladaptive client response to panic disorder? A. Clients overuse medical care because of physical symptoms. B. Clients use illegal drugs to ease symptoms. C. Clients perceive having no control over life situations. D. Clients develop compulsions to deal with anxiety.

ANS: C The major maladaptive client response to panic disorder is the perception of having no control over life situations, which leads to nonparticipation in decision making and doubts regarding role performance.

12. A client is experiencing a severe panic attack. Which nursing intervention would meet this clients immediate need? A. Teach deep breathing relaxation exercises B. Place the client in a Trendelenburg position C. Stay with the client and offer reassurance of safety D. Administer the ordered prn buspirone (BuSpar)

ANS: C The nurse can meet this clients immediate need by staying with the client and offering reassurance of safety and security. The client may fear for his or her life, and the presence of a trusted individual provides assurance of personal safety.

5. A nurse who works on an inpatient psychiatric unit is working on developing a treatment plan for a patient admitted with PTSD. The patient, a military veteran, reports that sometimes he thinks he sees bombs exploding and the enemy rushing toward him. He has had aggressive outbursts and was hospitalized after assaulting a coworker during one of these episodes. Which of these interventions by the nurse are evidence-based responses? Select all that apply. A. Collaborate with the patient about how he would like staff to respond when he has episodes of re-experiencing traumatic events. B. Tell the patient it is not appropriate to hit other patients or staff and if that occurs he will have to be discharged from the hospital. C. Contact the doctor and recommend that the patient be ordered an antipsychotic medication. D. Refer the patient to a support group with other military veterans.

ANS: A, D Collaborating with the patient demonstrates an environment of mutual respect and is helpful in establishing a trusting relationship. Both of these are identified as essential in effective treatment of PTSD. Evidence also supports that a group with other people who have experienced similar traumas is helpful in reducing the sense of isolation that some people with PTSD experience. Items B and C are incorrect since they both reflect an inaccurate understanding of the dynamics of PTSD.

A client diagnosed with dissociative identity disorder (DID) switches personalities when confronted with destructive behavior. The nurse recognizes that this dissociation serves which function? A. It is a means to attain secondary gain. B. It is a means to explore feelings of excessive and inappropriate guilt. C. It serves to isolate painful events so that the primary self is protected. D. It serves to establish personality boundaries and limit inappropriate impulses.

ANS: C The nurse should anticipate that a client who switches personalities when confronted with destructive behavior is dissociating in order to isolate painful events so that the primary self is protected. The transition between personalities is usually sudden, dramatic, and precipitated by stress.

Which of the following somatic symptom and dissociative disorders are identified with known effective pharmacological treatments for that disorder? Select all that apply. A. Antidepressants have been used effectively in treating pain associated with somatic symptom disorder. B. Lithium has been effective in treating illness anxiety disorder. C. Muscle relaxants have been effective in resolving conversion disorder symptoms. D. The antidepressant clomipramine (Anafranil) has shown promise in treating depersonalization-derealization disorder.

ANS: A, D The nurse should recognize that psychological therapies are central treatment modalities for these disorders, although evidence exists that for specific issues (as those identified in items A and C), psychopharmacological agents have demonstrated effectiveness.

27. A college student has been diagnosed with generalized anxiety disorder (GAD). Which of the following symptoms should a campus nurse expect this client to exhibit? Select all that apply. A. Fatigue B. Anorexia C. Hyperventilation D. Insomnia E. Irritability

ANS: A, D, E The nurse should expect that a client diagnosed with GAD would experience fatigue, insomnia, and irritability. GAD is characterized by chronic, unrealistic, and excessive anxiety and worry.

9. A client has undergone psychological testing. With which member of the interdisciplinary team should a nurse collaborate to review these results? A. The psychiatrist B. The psychiatric social worker C. The clinical psychologist D. The clinical nurse specialist

ANS: C The nurse should consult with the clinical psychologist to review psychological testing results for the client. Clinical psychologists can administer, interpret, and evaluate psychological tests to assist in the diagnostic process

A college student has been diagnosed with generalized anxiety disorder (GAD). Which of the following symptoms should a campus nurse expect this client to exhibit? Select all that apply. A. Fatigue B. Anorexia C. Hyperventilation D. Insomnia E. Irritability

ANS: A, D, E The nurse should expect that a client diagnosed with GAD would experience fatigue, insomnia, and irritability. GAD is characterized by chronic, unrealistic, and excessive anxiety and worry. KEY: Cognitive Level: Application | Integrated Processes: Nursing Process: Assessment | Client Need: Psychosocial Integrity

20. Joe, who recently lost both parents in a tragic automobile accident, has been diagnosed with an adjustment disorder after he struck a friend who told him he needed to get his feelings out. The stage of grieving that Joe is struggling with is ___________________.

ANS: Anger The stages of grieving include denial, anger, bargaining, depression, and resolution. Joe is expressing anger but in a way that is impairing his relationships with others.

20. A client presents in the emergency department with complaints of overwhelming anxiety. Which of the following is a priority for the nurse to assess? A. Risk for suicide B. Cardiac status C. Current stressors D. Substance use history

ANS: B Although all of the listed aspects of assessment are important, the priority is to evaluate cardiac status since a person having an MI, CHF, or mitral valve prolapse can present with symptoms of anxiety.

15. A client diagnosed with obsessive-compulsive disorder is admitted to a psychiatric unit. The client has an elaborate routine for toileting activities. Which would be an appropriate initial client outcome during the first week of hospitalization? A. The client will refrain from ritualistic behaviors during daylight hours. B. The client will wake early enough to complete rituals prior to breakfast. C. The client will participate in three unit activities by day 3. D. The client will substitute a productive activity for rituals by day 1.

ANS: B An appropriate initial client outcome is for the client to wake early enough to complete rituals prior to breakfast. The nurse should also provide a structured schedule of activities and later in treatment begin to gradually limit the time allowed for rituals.

26. A client is taking chlordiazepoxide (Librium) for generalized anxiety disorder symptoms. In which situation should a nurse recognize that this client is at greatest risk for drug overdose? A. When the client has a knowledge deficit related to the effects of the drug B. When the client combines the drug with alcohol C. When the client takes the drug on an empty stomach D. When the client fails to follow dietary restrictions

ANS: B Both Librium and alcohol are central nervous system depressants. In combination, these drugs have an additive effect and can suppress the respiratory system, leading to respiratory arrest and death.

Which of the following statements accurately describes dissociative fugue? A. Dissociative fugue is not precipitated by stressful events. B. Dissociative fugue is characterized by sudden, unexpected travel or bewildered wandering with inability to recall some or all of ones past. C. Dissociative amnesia and dissociative fugue are completely different types of disorders. D. Dissociative fugue is characterized by a sense of observing oneself from outside the body.

ANS: B Dissociative fugue is characterized by unexpected travel or bewildered wandering, and amnesia for all or part of ones past. Dissociative fugue is a subtype of dissociative amnesia.

10. Jane presents in the Emergency Department with a friend, who reports that Jane has been sitting in her apartment staring off into space and doesnt seem interested in doing anything. During the assessment Jane reveals, with little emotion, that she was raped 4 months ago. Which of these is the most appropriate interpretation of Janes lack of emotion? A. Jane is probably hearing voices telling her to be emotionless. B. Jane is experiencing numbing of emotional response, which is a common symptom of PTSD. C. Jane is trying to be secretive, and lying is a common symptom in PTSD. D. Jane is currently re-experiencing the traumatic event and is having a dissociative episode.

ANS: B General numbing of emotional response is a common symptom of PTSD. Items A and D are not the most appropriate interpretations because the data are inadequate to make that inference. Item C is incorrect; lying is not a common symptom in PTSD.

17. Brandy is an 18-year-old being treated in the Community Mental Health Clinic for an adjustment disorder after receiving news of her parents impending divorce. While talking about her feelings she becomes angry and starts shouting and crying. She screams, I wish they would both die! Which of these is the most appropriate response by the nurse at this point? A. Contact the parents and the police to report that Brandy is expressing homicidal ideation. B. Encourage Brandy to talk more about her anger. C. Instruct Brandy that its okay to cry but that it is not acceptable to talk that way about her parents. D. Assess Brandy for suicidal ideation.

ANS: B It is important in treating patients with adjustment disorders to allow them to express anger. Item C discourages the patient from expressing anger. Items A and D would be premature, since there is inadequate evidence to warrant those responses.

4. A client diagnosed with schizophrenia functions well and is bright, spontaneous, and interactive during hospitalization but then decompensates after discharge. What does the milieu provide that may be missing in the home environment? A. Peer pressure B. Structured programming C. Visitor restrictions D. Mandated activities

ANS: B The milieu, or therapeutic community, provides the client with structured programming that may be missing in the home environment. The therapeutic community provides a structured schedule of activities in which interpersonal interaction and communication with others are emphasized. In the milieu, time is also devoted to personal problems and focus groups.

19. A nursing student questions an instructor regarding the order for fluvoxamine (Luvox), 300 mg daily, for a client diagnosed with obsessive-compulsive disorder (OCD). Which instructor reply is most accurate? A. High doses of tricyclic medications will be required for effective treatment of OCD. B. Selective serotonin reuptake inhibitor (SSRI) doses, in excess of what is effective for treating depression, may be required for OCD. C. The dose of Luvox is low due to the side effect of daytime drowsiness and nighttime insomnia. D. The dosage of Luvox is outside the therapeutic range and needs to be questioned.

ANS: B The most accurate instructor response is that SSRI doses in excess of what is effective for treating depression may be required in the treatment of OCD. SSRIs have been approved by the U.S. Food and Drug Administration for the treatment of OCD. Common side effects include headache, sleep disturbances, and restlessness.

11. A family member is seeking advice about an elderly parent who seems to worry unnecessarily about everything. The family member states, Should I seek psychiatric help for my mother? Which is an appropriate nursing reply? A. My mother also worries unnecessarily. I think it is part of the aging process. B. Anxiety is considered abnormal when it is out of proportion to the stimulus causing it and when it impairs functioning. C. From what you have told me, you should get her to a psychiatrist as soon as possible. D. Anxiety is a complex phenomenon and is effectively treated only with psychotropic medications.

ANS: B The most appropriate reply by the nurse is to explain to the family member that anxiety is considered abnormal when it is out of proportion and impairs functioning. Anxiety is a normal reaction to a realistic danger or threat to biological integrity or self-concept.

Which would be considered an appropriate outcome when planning care for an inpatient client diagnosed with somatic symptom disorder? A. The client will admit to fabricating physical symptoms to gain benefits by day 3. B. The client will list three potential adaptive coping strategies to deal with stress by day 2. C. The client will comply with medical treatments for physical symptoms by day 3. D. The client will openly discuss physical symptoms with staff by day 4.

ANS: B The nurse should determine that an appropriate outcome for a client diagnosed with somatic symptom disorder would be for the client to list three potential adaptive coping strategies to deal with stress by day 2. Because the symptoms of somatic symptom disorder are associated with psychosocial distress, increasing coping skills may help the client reduce symptoms.

6. A client refuses to go on a cruise to the Bahamas with his spouse because of fearing that the cruise ship will sink and all will drown. Using a cognitive theory perspective, the nurse should use which of these statements to explain to the spouse the etiology of this fear? A. Your spouse may be unable to resolve internal conflicts, which result in projected anxiety. B. Your spouse may be experiencing a distorted and unrealistic appraisal of the situation. C. Your spouse may have a genetic predisposition to overreacting to potential danger. D. Your spouse may have high levels of brain chemicals that may distort thinking.

ANS: B The nurse should explain that from a cognitive perspective the client is experiencing a distorted and unrealistic appraisal of the situation. From a cognitive perspective, fear is described as the result of faulty cognitions.

7. What is the best rationale for including the clients family in therapy within the inpatient milieu? A. To structure a program of social and work-related activities B. To facilitate discharge from the hospital C. To provide a concrete demonstration of caring D. To encourage the family to model positive behaviors

ANS: B The nurse should include the clients family in therapy within the inpatient milieu to facilitate discharge from the hospital. Family members are invited to participate in some therapy groups and to share meals with the client in the communal dining room. Family involvement may also serve to prevent the client from becoming too dependent on the therapeutic environment.

1. A nursing instructor is teaching about specific phobias. Which student statement should indicate that learning has occurred? A. These clients do not recognize that their fear is excessive, and they rarely seek treatment. B. These clients have overwhelming symptoms of panic when exposed to the phobic stimulus. C. These clients experience symptoms that mirror a cerebrovascular accident (CVA). D. These clients experience the symptoms of tachycardia, dysphagia, and diaphoresis.

ANS: B The nursing instructor should evaluate that learning has occurred when the student knows that clients experiencing phobias have a panic level of fear that is overwhelming and unreasonable. Phobia is fear cued by a specific object or situation in which exposure to the stimulus produces an immediate anxiety response.

16. Major Smith, who is being treated for PTSD symptoms following a course of military duty, reports, I think I was in denial about even having PTSD. I thought I was just having trouble sleeping. Which of these is an accurate evaluation of the patients comments? A. The patient is still in denial and unable to recognize that he is having flashbacks rather than insomnia. B. The patient is beginning to recognize stages of grieving and reevaluating his symptoms. C. The patient is beginning to recognize that he may be at risk for suicide. D. The patient is trying to avoid discussing symptoms of PTSD.

ANS: B The patient is expressing recognition that he was in denial, which is a stage of grieving. It is not uncommon for people to recognize that they are having troubling symptoms but not immediately recognize this as PTSD.

Psychosocial Theories

Credibility of psychosocial theories has declined in recent years. Bipolar disorder is viewed as a disease of the brain.

Assessment of the biological domain for bulimia

Current eating patterns, # of times per day of binging/purging, dietary restraint practices, sleep patterns, exercise habits

Which of the following are true of antisocial personality disorder (APD)? (select all that apply): A) It is the least studied of the personality disorders. B) It is characterized by rigidity and inflexible standards of self and others. C) Persons with APD display magical thinking. D) Persons with APD are concerned with personal pleasure and power. E) It is characterized by deceitfulness, disregard for others, and manipulation. F) Persons with APD usually present for treatment because of awareness of how their behavior is affecting others. G) Frontal lobe dysfunction is a brain change identified in APD.

D) Persons with APD are concerned with personal pleasure and power. E) It is characterized by deceitfulness, disregard for others, and manipulation. G) Frontal lobe dysfunction is a brain change identified in APD. APD is the most studied and researched personality disorder. Rigidity and inflexible standards describe obsessive-compulsive personality disorder. Magical thinking describes schizotypal personality disorder. People with APD usually present with depression or because of the consequences of their behaviors, not because they care about the effects of their actions on others.

Neurotransmitters have been implicated in the pathophysiology of anxiety disorders. Select the disturbances that are associated with anxiety disorders: A. Increased seratonin, decreased norepinephrine, and decreased GABA. B. Increased seratonin, decreased norepinephrine, and increased GABA. C. Decreased seratonin, decreased norepinephrine, and decreased GABA. D. Decreased seratonin, increased norepinephrine, and decreased GABA.

D. Decreased seratonin, increased norepinephrine, and decreased GABA. pg. 530

Which of the following is not a common traits/symptom of hoarding disorder? A. Perfectionism B. Indecisiveness C. Distractibility D. narcissistic personality disorder

D. Narcissistic personality disorder is associated body dysmorphic disorder. Associated symptoms of hoarding disorder include: perfectionism, indecisiveness, anxiety, depression, distractibility, and difficulty planning and organizing.

With implosion therapy, a client with phobic anxiety would be: A. Taught relaxation exercises. B. Subjected to graded intensities of the fear C. Instructed to stop the therapeutic session as soon as anxiety is experienced. D. Presented with massive exposure to a variety of stimuli associated with the phobic object/situation.

D. Presented with massive exposure to a variety of stimuli associated with the phobic object/situation.

Sasha's roommate Kate was admitted with major depression and suicidal ideation with a plan to overdose. Kate is preparing for discharge and asks you, "Why did Dr. Travis give me a prescription for only 7 days of amitriptyline?" Your response is based on the knowledge that: A. amitriptyline (Elavil) is very expensive, so the patient may have to buy fewer at a time. B. Dr. Travis is going to see how Kate responds to the first week of medication to evaluate its effectiveness. C. Dr. Travis wants to see whether any minor side effects occur within the first week of administration. D. amitriptyline (Elavil) is lethal in overdose.

D. amitriptyline (Elavil) is lethal in overdose. Amitriptyline is a tricyclic antidepressant (TCA); these drugs are known to be lethal in smaller doses than other antidepressants. Because the patient had a plan of overdose, the best course of action is to give a small prescription requiring her to visit her provider's office more often for monitoring of suicidal ideation and plan. Tricyclics are not known to be expensive. Antidepressant therapy usually takes several weeks to produce full results, so the patient would not be evaluated after only one week. Side effects are always a consideration but not the most important consideration with TCAs.

Dysthymia cannot be diagnosed unless it has existed for A. at least 3 months. B. at least 6 months. C. at least 1 year. D. at least 2 years.

D. at least 2 years. Dysthymia is a chronic condition that by definition has to have existed for longer than 2 years.

A client who has been assessed by the nurse as moderately depressed is given a prescription for daily doses of a selective serotonin reuptake inhibitor. The client mentions that she will take the medication along with the St. John's wort she uses daily. The nurse should: A. agree that taking the drugs at the same time will help her remember them daily. B. caution the client to drink several glasses of water daily. C. suggest that the client also use a sun lamp daily. D. explain the high possibility of an adverse reaction.

D. explain the high possibility of an adverse reaction. Serotonin malignant syndrome is a possibility if St. John's wort is taken with other antidepressants.

A depressed client tells the nurse he is in the "acute phase" of his treatment for depression. The nurse recognizes that the client has been in treatment: A. for more than 4 months. B. that is directed toward relapse prevention. C. that focuses on prevention of future depression. D. to reduce depressive symptoms.

D. to reduce depressive symptoms. The acute phase of depression therapy (6-12 weeks) is directed toward the reduction of symptoms and restoration of psychosocial and work function and may require some hospitalization.

Comorbidities with anorexia

Depression and suicide common. Anxiety d/o's such as OCD, phobias, and panic d/o are strongly associated.

Bipolar Disorder Associated with Another Medical Condition

Determined to be a result of direct physiological effects of another medical condition.

Substance-Induced Bipolar Disorder

Disturbances of mood do to ingestion/withdrawal from drug abuse

Epidemiology of anorexia

Females affected 10x more than males. More common among hispanics and whites. Age of onset typically b/w 14 and 16 but can occur earlier.

Neuroendocrine and NT changes theory for anorexia

Increase in endogenous opioids (thru exercise) contributes to denial of hunger. Malnutrition leads to a decrease in thyroid function. Serotonergic function is decreased in low-weight pts

what is the cycle of battering?

Phase 1= tension building phase phase 2= acute battering incident phase 3= calm, loving respite (honeymoon) phase

When the nurse remarks to a depressed client, "I see you are trying not to cry. Tell me what is happening." The nurse should be prepared to A. wait quietly for the client to reply. B. prompt the client if the reply is slow. C. repeat the question if the client does not answer promptly. D. review the client's medical record to support the client's response.

When the nurse remarks to a depressed client, "I see you are trying not to cry. Tell me what is happening." The nurse should be prepared to A. wait quietly for the client to reply. Depressed clients think slowly and take long periods to formulate answers and respond. The nurse must be prepared to wait for a reply.

Which is true of pharmacological therapies for treatment of personality disorders? A) Although there are no FDA-approved drugs specific to the treatment of personality disorders, patients benefit from specific off-label uses of antipsychotics, mood stabilizers, and antidepressants, depending on which personality disorder is evident. B) Research has shown that currently available psychotropic drugs have not been shown to be effective in treating personality disorders. C) Patients with narcissistic personality disorder and obsessive-compulsive personality disorder have shown the most benefit from the use of antianxiety medications along with use of selective serotonin reuptake inhibitors. D) Patients with personality disorders have been shown to be resistant to accepting medication, and as a result most providers do not prescribe psychotropic drugs to these patients.

a)Although there are no FDA-approved drugs specific to the treatment of personality disorders, patients benefit from specific off-label uses of antipsychotics, mood stabilizers, and antidepressants, depending on which personality disorder is evident. At this time in the United States, there are no specifically FDA-approved medications for treating personality disorders. Prescribers are using the medications "off- label" until evidence-based pharmacotherapies are proven to be safe and effective. There is evidence that mood stabilizers, antidepressants, and atypical antipsychotics are helpful in specific personality disorders. Pharmacologic evidence is lacking for the treatment of persons with narcissistic and obsessive-compulsive personality disorders. Although patients with personality disorders usually do not like taking medicine unless it calms them down and are fearful about taking something over which they have no control, providers do attempt to mediate symptoms with psychotropic agents for improved quality of life.

A client diagnosed with obsessive-compulsive personality disorder takes the nurse aside and mentions, "I've observed you interacting with that new patient. You are not approaching him properly. You should be more forceful with him." The best response for the nurse would be a. "I will be continuing to follow the care plan for the patient." b. "I see you are trying to control that patient's therapy as well as your own." c. "Your eye for perfection extends even to my nursing interventions." d. "That patient's care is really of no concern to you or to other clients."

a. "I will be continuing to follow the care plan for the patient." Obsessive-compulsive personality disorder has the key factor of perfectionism with a focus on orderliness and control. These individuals get so preoccupied with details and rules that they may not be able to accomplish the tasks. Guard against engaging in power struggles with a client with obsessive-compulsive disorder.

A nurse who is idealized by a client is at risk for a. becoming overinvolved and being protective and indulgent. b. becoming indecisive about planned interventions. c. developing a prejudicial, blaming orientation. d. stringent enforcement of boundaries and limits. F

a. becoming overinvolved and being protective and indulgent. Finding an approach for helping clients with personality disorders who have overwhelming needs can be challenging for caregivers. For example, a borderline female client may briefly idealize her male nurse on the inpatient unit, telling staff and clients alike that she is "the luckiest client because she has the best nurse in the hospital." The rest of the team initially realizes that this behavior is an exaggeration, and they have a neutral response. But after days of constant dramatic praise, some members of the team may start to feel inadequate and jealous of the nurse. They begin to make critical remarks about minor events to prove that the nurse is not perfect. Open communication in staff meetings and ongoing clinical supervision are important aspects of self-care for the nurse working with these clients to maintain objectivity.

A client has been hospitalized with anorexia nervosa. The client's weight is 65% of normal. For this client, a realistic short-term goal for the first week of hospitalization would be: By the end of week 1, the client will a. gain a maximum of 3 lb. b. develop a pattern of normal eating behavior. c. discuss fears and feelings about gaining weight. d. verbalize awareness of the sensation of hunger.

a. gain a maximum of 3 lb. The critical outcome during hospitalization for anorexia nervosa is weight gain. A maximum of 3 pounds weekly is considered sufficient initially. Too-rapid weight gain can cause pulmonary edema.

When providing care for a client diagnosed with borderline personality disorder, the nurse will need to consider strategies for dealing with the client's a. mood shifts, impulsivity, and splitting. b. grief, anger, and social isolation. c. altered sensory perceptions and suspicion. d. perfectionism and preoccupation with detail.

a. mood shifts, impulsivity, and splitting. Borderline personality disorder has the central characteristic of instability in affect, identity, and relationships. Borderline individuals desperately seek relationships to avoid feeling abandoned, but they often drive others away with excessive demands, impulsive behavior, or uncontrolled anger. Their frequent use of the defense of splitting strains personal relationships and creates turmoil in health care settings.

Psychoanalytically, the theory of obesity relates to the individual's unconscious equation of food with: a. nurturance and caring b. power and control c. autonomy & emotional growth d. strength and endurance

a. nurturance and caring

Which of the following behavioral patterns is characteristic of individuals with narcissistic personality disorder? a. overly self-centered and exploitative of others b. suspicious and mistrustful of others c. rule conscious and disapproving of change d. Anxious and socially isolated

a. overly self-centered and exploitative of others

Kim, a ct diagnosed with borderline personality disorder, manipulates the staff in an effort to fulfill her own desires. All of the following may be examples of manipulative behaviors in the borderline client except: a. refusal to stay in room alone, stating, "It's so lonely" b. asking Nurse Jones for cigarettes after 30 minutes, knowing the assigned nurse has explained she must wait one hour c. stating to Nurse Jones "I really like having you for my nurse. You're the best one around here." d. cutting arms with a razor blade after discussing dismissal plans with physician

a. refusal to stay in room alone, stating, "It's so lonely"

The nurse working with clients diagnosed with eating disorders can help families develop effective coping mechanisms by a. teaching the family about the disorder and the client's behaviors. b. stressing the need to suppress overt conflict within the family. c. urging the family to demonstrate greater caring for the client. d. encouraging the family to use their usual social behaviors at meals.

a. teaching the family about the disorder and the client's behaviors. Families need information about specific eating disorders and the behaviors often seen in clients with these disorders. This information can serve as a basis for additional learning about how to support the family member.

Which statement is descriptive of clients with a personality disorder? a. They are resistant to behavioral change. b. They have an ability to tolerate frustration and pain. c. They usually seek help to change maladaptive behaviors. d. They have little difficulty forming satisfying and intimate relationships.

a.They are resistant to behavioral change. Personality disorders are deeply ingrained and pervasive. Clients with personality disorders find it very difficult, if not nearly impossible, to change. Change proceeds very slowly.

The primary goal of milieu therapy for clients diagnosed with personality disorders is a. to manage the effect the behavior has on the entire group. b. to provide one-on-one therapy for each member of the milieu. c. to help the client remain uninvolved with other patients. d. to promote a laissez-faire attitude among the staff members.

a.to manage the effect the behavior has on the entire group. The primary goal of milieu therapy is affect management in a group context.

Survivors of Abuse: Background Assessment Data: what are char's of the abuser?

abuser often victim of abuse themselves char's r/t abusive parent: isolated /c little support from family expects child to fulfill their emotional needs prone to depression low frustration tolerance frequent outbursts, anger, rage

what are the different types off sexual assault?

acquaintance rape, date rape, marital rape, statutory rape

Survivors of Abuse: Background Assessment Data: Cycle of Battering: what is char'd / phase 2?

acute battering incident lasts up to 24 hours; most violent and shortest period batterer justifies his bx to himself (batterer may not know what happened, just lost rage) victim: feels only safe thing to do is leave; can undergo disassociation during beating; help is sought only when fearing for her life or for children's lives

Sociocultural Theories: Societal Influences: what does sociocultural factors state regarding aggressive bx?

aggressive bx => product of culture and social structure ie: relative deprivation (lack of needed things compared to another group) poverty prolonged unemployment family breakdown exposure to violence in community

Survivors of Abuse: Background Assessment Data: whatis acquantince rape?

aka date rape

Bipolar disorder most likely results from

an interaction between genetic, biological, and psychosocial determinants.

Belinda is a 24-year-old patient with borderline personality disorder (BPD). She is admitted to the inpatient psychiatric unit following a suicide attempt. You are caring for Belinda. Which of the following statements by Belinda illustrates a primary coping style of persons with BPD? a. "My provider says I might get out of here tomorrow. Do you think I'm ready to go?" b. "Last night the nurse let me go outside and smoke. I can't believe you aren't letting me. I used to think you were the best nurse here." c. "I will never again speak to any of my messed up family members. I know that this will help me be more functional." d. "I promise I am not feeling suicidal. I won't hurt myself."

b. "Last night the nurse let me go outside and smoke. I can't believe you aren't letting me. I used to think you were the best nurse here." A primary coping style used by patients with BPD is called splitting. Splitting is the inability to incorporate positive and negative aspects of oneself or others into a whole image. The individual may tend to idealize another person (friend, lover, health care professional) at the start of a new relationship and hope that this person will meet all of his or her needs. At the first disappointment or frustration, however, the individual quickly shifts to devaluation, despising the other person. The other options do not describe splitting, which is a primary coping style of patients with BPD.

The nurse is interviewing a client on the psychiatric unit. The client tilts his head to the side, stops talking in midsentence, and listens intently. The nurse recognizes these behaviors as a symptom of the client's illness. The most appropriate nursing intervention for this symptom is to: a. Ask the client to describe his physical symptoms. b. Ask the client to describe what he is hearing. c. Administer a dose of benztropine. d. Call the physician for additional orders.

b. Ask the client to describe what he is hearing.

Bupropion (Wellbutrin), although seemingly effective, is contraindicated in patients who purge because of a. historically poor patient compliance. b. an increased risk of seizures. c. the long-term effects on liver function. d. the potential to cause gastric ulcers.

b. an increased risk of seizures. Bupropion (Wellbutrin), although seemingly effective, is contraindicated in patients who purge because of an increased risk of seizures.

Jane is hospitalized on the psychiatric unit. she has a history and current diagnosis of bulimia nervosa. Which of the following symptoms would be congruent with Jane's diagnosis? a. binging, purging, obesity, hyperkalemia b. binging, purging, normal weight, hypokalemia c. binging, laxative abuse, amenorrhea, severe weight loss d. binging, purging, severe weight loss, hyperkalemia

b. binging, purging, normal weight, hypokalemia

A client with bulimia nervosa uses enemas and laxatives to purge to maintain her weight. The imbalance for which the nurse should assess is a(n) a. increase in the red blood cell count. b. disruption of the fluid and electrolyte balance. c. elevated serum potassium level. d. elevated serum sodium level.

b. disruption of the fluid and electrolyte balance. Disruption of the fluid and electrolyte balance is usually the result of excessive use of enemas and laxatives.

Milieu therapy is a good choice for clients with antisocial personality disorder because it: a. provides a system of punishment and rewards for behavior modification b. emulates a social community in which the client may learn to live harmoniously with others c. provides mostly one-to-one interaction between the client and therapist d. provides a very structured setting in which the clients have very little input into the planning of their care

b. emulates a social community in which the client may learn to live harmoniously with others

Nancy, age 14, has just been admitted to the psychiatric unit for anorexia nervosa. She is emaciated and refusing to eat. What is the primary nursing diagnosis for Nancy? a. complicated grieving b. imbalanced nutrition, less than body requirements c. interrupted family processes d. anxiety (severe)

b. imbalanced nutritition: less than body requirements

Research has indicated that the antisocial personality may be characterized by a. social isolation. b. lack of remorse. c. learning difficulties. d. difficulty with reality testing.

b. lack of remorse. Individuals with an antisocial personality exhibit a lack of remorse when confronted with the results of their thoughtless, irresponsible behavior toward others.

From a physiological point of view, the most common cause of obesity is probably: a. lack of nutritional education b. more calories consumed than expended c. impaired endocrine functioning d. low basal metabolic rate

b. more calories consumed than expended

"Splitting" by the client with borderline personality disorder denotes: a. evidence of precocious development b. primitive defense mechanism in which the ct sees objects as all good or all bad c. a brief psychotic episode in which the client loses contact with reality d. two distinct personalities within the borderline client

b. primitive defense mechanism in which the ct sees objects as all good or all bad

A newly admitted client has an axis II diagnosis of schizoid personality disorder. The nursing intervention of highest priority will be to a. set firm limits on behavior. b. respect need for social isolation. c. encourage expression of feelings. d. involve in milieu and group activities.

b. respect need for social isolation. Schizoid personality disorder has the primary feature of emotional detachment. Individuals do not seek out or enjoy close relationships. They are reclusive, avoidant, and uncooperative. They do not do well with resocialization.

When a client suddenly becomes aggressive and violent on the unit, which of the following approaches would be best for the nurse to use first? a. Provide large motor activities to relieve the client's pent-up tension. b. Administer a dose of prn chlorpromazine to keep the client calm. c. Call for sufficient help to control the situation safely. d. Convey to the client that his behavior is unacceptable and will not be permitted.

c. Call for sufficient help to control the situation safely.

A child with bipolar disorder also has attention-deficit/hyperactivity disorder (ADHD). How would these comorbid conditions most likely be treated? a. No medication would be given for either condition. b. Medication would be given for both conditions simultaneously. c. The bipolar condition would be stabilized before ADHD medication would be given. d. The ADHD would be treated before consideration of the bipolar disorder.

c. The bipolar condition would be stabilized before ADHD medication would be given.

According to current theory, eating disorders: a. are psychotic disorders in which patients experience body dysmorphic disorder. b. are frequently misdiagnosed. c. are possibly influenced by sociocultural factors. d. are rarely comorbid with other mental health disorders.

c. are possibly influenced by sociocultural factors. The Western cultural ideal that equates feminine beauty with tall, thin models has received much attention in the media as a cause of eating disorders. Studies have shown that culture influences the development of self-concept and satisfaction with body size. Eating disorders are not psychotic disorders. There is no evidence that eating disorders are frequently misdiagnosed. Comorbidity for patients with eating disorders is more likely than not. Personality disorders, affective disorders, and anxiety frequently occur with eating disorders.

The priority nursing intervention for a client diagnosed with borderline personality disorder is to a. protect other clients from manipulation. b. respect the client's need for social isolation. c. assess for suicidal and self-mutilating behaviors. d. provide clear, consistent limits and boundaries.

c. assess for suicidal and self-mutilating behaviors. One of the primary nursing guidelines/interventions for clients with a personality disorder is to assess for suicidal and self-mutilating behaviors, especially during times of stress.

The client diagnosed with a personality disorder who is most likely to be admitted to a psychiatric unit is one who has a. paranoid personality disorder and is suspicious of his neighbors. Incorrect b. narcissistic personality disorder and is highly self-important. c. borderline personality disorder and is impulsive. d. dependent personality disorder and clings to her husband.

c. borderline personality disorder and is impulsive. Clients with borderline disorder can decompensate into psychotic states under stress. Hospitalization is needed at these times.

Clients demonstrating characacteristics of personality disorders have various self-defeating behaviors and interpersonal problems despite having near-normal ego functioning and intact reality testing. A nursing diagnosis that addresses this sort of interpersonal dysfunction is a. spiritual distress. b. defensive coping. c. impaired social interaction. d. disturbed sensory perception.

c. impaired social interaction. For a client who has difficulty in relationships and is very manipulative, the nursing diagnosis of impaired social interaction would be used.

A nurse caring for a client who has been diagnosed with a personality disorder should expect that the client will exhibit a. frequent episodes of psychosis. b. constant involvement with the needs of significant others. c. inflexible and maladaptive responses to stress. d. abnormal ego functioning.

c. inflexible and maladaptive responses to stress. Personality patterns persist unmodified over long periods of time. Inflexible and maladaptive responses to stress are characteristic of individuals with a personality disorder.

Assessment of a client suspected of experiencing bulimia nervosa calls for the nurse to perform a. a range of motion assessment. b. inspection of body cavities. c. inspection of the oral cavity. d. body fat analysis.

c. inspection of the oral cavity. Repeated vomiting often causes dental erosions and caries.

Which medication has been used with some success in clients with anorexia nervosa? a. lorcaserin (Belviq) b. diazepam (valium) c. fluoxetine (prozac) d. carbamazepine (Tegretol)

c. prozac

Splitting is a process in which the client a. unconsciously represses undesirable aspects of self. b. places responsibility for his or her behavior outside the self. c. sees things as divided into "all good" or "all bad." d. evidences lack of personal boundaries.

c. sees things as divided into "all good" or "all bad." Splitting demonstrates the failure to integrate the positive and negative into a cohesive whole. An individual is not seen as a person with good and bad traits, but rather as all good or all bad.

Biological theorists suggest that the cause of eating disorders may be a. normal weight phobia. b. body image disturbance. c. serotonin imbalance. d. dopamine excess.

c. serotonin imbalance. The selective serotonin reuptake inhibitors have been shown to improve the rate of weight gain and reduce the occurrence of relapse.

Survivors of Abuse: Background Assessment Data: The Victim: what are two types of reactions survivors of rape can develop?

compounded rape rxn: additional sx's of the following present: depression suicide substance abuse psychotic bx's silent rape rxn: survivor tells no one about the assault anxiety is suppressed

Survivors of Abuse: Background Assessment Data: The Victim: what sx's are common weeks after rape attack?

contusions / abrasions h/a's fatigue sleep pattern disturbance stomach pains, n/v vaginal d/c and itching burning /c urination rage, humiliation, embarrasement self blame fear / physical violence

Survivors of Abuse: nursing dx and outcome identification: what interventions can a nurse do for a victim of rape?

convey: "you are safe here" " it's not your fault" explain assessment procedure before it is done ensure PT has adequate privacy encourage PT to give account of the assault take PT to private area to do interview assure PT of safety if coming to visit alone ensure "rescue" efforts are not done by nurse stress importance of safety /c PT see if explanation of child injury is reasonable use games or play therapy for child (if abuse is /c child)

Kim has a diagnosis of borderline personality disorder. She often exhibits alternating clinging and distancing behaviors. The most appropriate nursing intervention with this type of behavior would be to: a. Encourage Kim to establish trust in one staff person, with whom all therapeutic interaction should take place. b. Secure a verbal contract from Kim that she will discontinue these behaviors. c. Withdraw attention if these behaviors continue. d. Rotate staff members who work with Kim so that she will learn to relate to more than one person.

d. Rotate staff members who work with Kim so that she will learn to relate to more than one person.

The primary focus of family therapy for clients with schizophrenia and their families is: a. To discuss concrete problem-solving and adaptive behaviors for coping with stress. b. To introduce the family to others with the same problem. c. To keep the client and family in touch with the health-care system. d. To promote family interaction and increase understanding of the illness.

d. To promote family interaction and increase understanding of the illness.

A subjective symptom the nurse would expect to note during assessment of a client with anorexia nervosa is a. lanugo. b. hypotension. c. 25-lb weight loss. d. fear of gaining weight.

d. fear of gaining weight. Option D is the only subjective data listed, and it is universally true.

The nurse can determine that inpatient treatment for a client diagnosed with an eating disorder would be warranted when the client a. weighs 10% below ideal body weight. b. has a serum potassium level of 3 mEq/L or greater. c. has a heart rate less than 60 beats/min. d. has systolic blood pressure less than 70 mm Hg.

d. has systolic blood pressure less than 70 mm Hg. Systolic blood pressure of less than 70 mm Hg is one of the established criteria signaling the need for hospitalization of a client with anorexia nervosa. It suggests severe cardiovascular compromise.

Carol is a new nursing graduate being oriented on a medical/surgical unit by the head nurse, Mrs. Carey. When Carol describes a new technique she has learned for positioning immobile clients, Mrs. Carey states, "What are you trying to do...tell me how to do my job? We have always done it this way on this unit, and we will continue to do it this way until I say differently!" this is an example of which type of personality characteristic? a. antisocial b. paranoid c. passive-agressive d. obsessive-compulsive

d. obsessive-compulsive

According to Margaret Mahler, predisposition to borderline personality disorder occurs when developmental tasks go unfilled in which of the following phases? a. Autistic phase, during which the child's needs for security and comfort go unfulfilled b. symbiotic phase, during which the child fails to bond with the mother c. differentiation phase, during which the child fails to recognize a separateness between self and mother d. rapprochement phase, during which the mother withdraws emotional support in response to the child's increasing independence.

d. rapprochement phase, during which the mother withdraws emotional support in response to the child's increasing independence.

Kim has a dx of borderline personality disorder. She often exhibits alternating clinging and distancing behaviors. The most appropriate nursing intervention with this type of behavior would be to: a. encourage Kim to establish trust in one staff person, with who all therapeutic interaction should take place b. secure a verbal contract from Kim that she will DC these behaviors c. Withdraw attention if these behaviors continue d. rotate staff members who work with Kim so that she will learn to relate to more than one person

d. rotate staff members who work with Kim so that she will learn to relate to more than one person

Which behavior would be inconsistent with defining characteristics for the nursing diagnosis of ineffective coping? a. Difficulty in relationships b. High levels of anxiety c. Manipulation d. Interdependence

d.Interdependence The characteristics for the diagnosis of ineffective coping include crisis, high levels of anxiety, anger, and aggression; child, elder, or spouse abuse; and difficulty in relationships and manipulation. Interdependence would not be considered a symptom for ineffective coping

Mary Alice is a 37-year-old patient referred to the mental health clinic with a suspected personality disorder. She is withdrawn and suspicious and states she has always preferred to be alone. She describes herself as having "special powers" and states that she is thinking of opening a business where she gives "readings" to people about their future. She states, "I believe we can all read each other's thoughts at times." Based on this presentation, you suspect: a. obsessive-compulsive personality disorder. b. narcissistic personality disorder. c. avoidant personality disorder. d. schizotypal personality disorder (STPD).

d.schizotypal personality disorder (STPD). The main traits that describe STPD are psychoticism such as eccentricity, odd or unusual beliefs and thought processes, and social detachment by preferring to be socially isolated, as well as being overly suspicious or anxious. In obsessive-compulsive personality disorder the main pathological personality traits are rigidity and inflexible standards of self and others, along with persistence of goals long after they are necessary, even if they are self-defeating or negatively affect relationships. People with narcissistic personality disorder come across as arrogant, with an inflated view of their self-importance. They have a need for constant admiration, along with a lack of empathy for others, a factor that strains most relationships over time. Traits of avoidant personality disorder include low self-esteem, feelings of inferiority compared with peers, and a reluctance to engage in unfamiliar activities involving new people.

lithium primary therapeutic effect:

decrease hyperactivity.

Neurotransmitters: what neurotransmitters are involved /c aggressive bx?

decreases in striatal serotonin increases in norepi and dopamine

what are characteristics of a father who sexually abuses their child?

domineering, impulsive, physically abusive

what is the difference between emotional abuse and neglect?

emotional abuse= is when the parent or caregiver shows behavior pattern that leaves child with impaired social, emotional or intellectual functioning. emotional neglect= is when the parent or caregiver fails to give the child love, hope, and support necessary for the development of a sound and health personality

Survivors of Abuse: Background Assessment Data: How is rape defined?

expression of power and dominance by using sexual violence

When might hospitalization occur w/ bulimia?

extreme dehydration and electrolyte imbalance, severe depression or suicide attempt

Survivors of Abuse: Background Assessment Data: Incestuous relationship: what is char'd of this relationship?

impaired sexual relationship b/t parents mother strongly suspects relationship b/t father and daughter occurs when: daughter is 8-10 yo; begins /c genital touching and fondling daughter has: love/hate relationship /c father; wants father-daughter relationship BUT hates the sexual demands **if incest is reported /c one daughter => incest should be suspected /c all the other daughters

A client newly diagnosed in a manic episode of bipolar disorder tells the nurse, "Now that I'm only sleeping 4 hours a night, I can get so much more work accomplished." Which ego defense mechanism is this client using?

intellectualization. using logic, reasoning.

Survivors of Abuse: Background Assessment Data: Cycle of Battering: What is the cycle of battering?

phase 1: Tension building phase phase 2 Acute battering phase phase 3 Calm, loving (honeymoon phase)

what is the difference between physical abuse and neglect?

physical abuse= any nonaccidental physical injury caused by caregiver or parent physical neglect= refusal of or delay in seeking health care, abandonment, expulsion from the home

what are examples of genetic influences for abusive behaviors?

possible hereditary factor, genetic karyotype XYY has been implicated

Survivors of Abuse: Background Assessment Data: The Victim: what is rape trauma syndrome?

powerlessness and intimidation during rape leads to => overwhelming sense of violation and helplessness

Survivors of Abuse: nursing dx and outcome identification: what nursing dx r/t rape?

rape trauma syndrome powerlessness risk for delayed development

Survivors of Abuse: Background Assessment Data: Physical Abuse: how is physical abuse char'd?

result of: punching kicking biting shaking throwing stabbing choking hitting

profile of the rapist

the rapists mom was seductive but rejecting, but was quick to withdraw her "love" when the child goes against her wishes, they grow up in abusive homes; even when the abuse was conducted by the father the child anger was towards the mother for not doing anything

what are characteristics of a child that has been sexually abused?

they have difficulty walking or sitting, suddenly refuses to change for gym or to participate in physical activities, reports nightmares or bedwetting, experiences a sudden change in appetite, demonstrates bizarre, sophisticated, or unusual sexual knowledge or behavior

Treatment Modalities: Crisis Intervention: How does crisis intervention deal /c abuse / rape?

time limited 6 - 8 weeks if beyond this time / treatment: PT referred to long term psychotherapy or psychiatrist or mental health clinic during crisis period: attn is given to coping strategies sx's of PT may include: stark hysteria anger rage silence withdrawal guilt / responsibility for rape help PT draw upon positive coping strategies => help obtain control over their life

Psychological Theories: Psychodynamic Theory: what psychodynamic theory r/t aggression?

underdeveloped ego and weak superego immature ego => does not stop id bx; weak supergo => fails to prod. feelings / guilt

Survivors of Abuse: Background Assessment Data: Physical Abuse: what are sx's of physical abuse?

unexplained burns, bites, bruises fading bruises seems frightened of parents shrinks at approach of parents reports injury by parents abuses animals or pets suspect abuse when parent has: no explanation for child injries child is described in negative way harsh physical discipline /c child hx / abuse as child hx / abusing animals

Survivors of Abuse: Background Assessment Data: what is statutory rape?

unlawful intercourse b/t: person over age of consent AND person under age of consent

How would a nurse differentiate a client diagnosed with a social phobia from a client diagnosed with a schizoid personality disorder (SPD)? A. Clients diagnosed with social phobia can manage anxiety without medications, whereas clients diagnosed with SPD can manage anxiety only with medications. B. Clients diagnosed with SPD are distressed by the symptoms experienced in social settings, whereas clients diagnosed with social phobia are not. C. Clients diagnosed with social phobia avoid interactions only in social settings, whereas clients diagnosed with SPD avoid interactions in all areas of life. D. Clients diagnosed with SPD avoid interactions only in social settings, whereas clients diagnosed with social phobias tend to avoid interactions in all areas of life.

ANS: C Clients diagnosed with social phobia avoid interactions only in social settings, whereas clients diagnosed with SPD avoid interactions in all areas of life. Social phobia is an excessive fear of situations in which a person might do something embarrassing or be evaluated negatively by others. KEY: Cognitive Level: Analysis | Integrated Processes: Nursing Process: Assessment | Client Need: Psychosocial Integrity

28. A nurse would expect a client diagnosed with schizotypal personality disorder to exhibit which characteristic? A. The client has many friends and associates but prefers to interact in small groups. B. The client has many brief but intense relationships. C. The client experiences incorrect interpretations of external events. D. The client exhibits lack of tender feelings toward others

ANS: C Clients who are diagnosed with schizotypal personality disorder experience odd beliefs or magical thinking that influences behavior and is inconsistent with cultural norms. This results in incorrect interpretations of external events.

A client on an inpatient unit is diagnosed with bipolar disorder: manic episode. During a discussion in the dayroom about weekend activities, the client raises his voice, becomes irritable, and insists that plans change. What should be the nurse's initial intervention? A. Ask the group to take a vote on alternative weekend events. B. Remind the client to quiet down or leave the dayroom. C. Assist the client to move to a calmer location. D. Discuss with the client impulse control problems.

ANS: C During a manic episode, the client experiences increased agitation and extreme hyperactivity that can lead to a risk for injury. Overstimulation can exacerbate these symptoms. Therefore, the nurse's initial action should focus on removing the client from the stimulating environment to a calmer location. KEY: Cognitive Level: Application | Integrated Processes: Nursing Process: Implementation | Client Need: Safe and Effective Care Environment: Management of Care

21. When providing care to an African American population in the community, which would be the most appropriate intervention for the nurse to implement? a. Build on the cultural traditions of the community. b. Develop a support system in a nearby community. c. Integrate mental health services into primary care settings. d. Focus on migrant health issues.

ANS: C Nurses can promote the mental health of this population by integrating mental health services into primary care settings, providing services in community centers, collaborating with faith communities, providing education to decrease the stigma, working toward the provision of safer communities, and recruiting members of this population to work as community mental health providers. Focusing on migrant health issues would occur with the Latino population. Building on cultural traditions is important in the Native American population. Developing a support system outside of the community would not be an appropriate intervention for any population.

Warren's college roommate actively resists going out with friends whenever they invite him. He says he can't stand to be around other people and confides to Warren "They wouldn't like me anyway." Which disorder is Warren's roommate likely suffering from? A. Agoraphobia B. Mysophobia C. Social anxiety disorder (social phobia) D. Panic disorder

ANS: C Social anxiety disorder is an excessive fear of social situations R/T fear that one might do something embarrassing or be evaluated negatively by others. KEY: Cognitive Level: Application | Integrated Processes: Nursing Process: Assessment | Client Need: Psychosocial Integrity

A newly admitted client is experiencing a manic episode of bipolar I disorder and presents as very agitated. The nurse should assign which priority nursing diagnosis to this client? A. Ineffective individual coping R/T hospitalization AEB alcohol abuse B. Altered nutrition: less than body requirements R/T mania AEB 10-pound weight loss C. Risk for violence: directed toward others R/T agitation and hyperactivity D. Sleep pattern disturbance R/T flight of ideas AEB sleeps 1 to 2 hours per night

ANS: C Some signs and symptoms of mania include manic excitement, delusional thinking, and hallucinations, which may predispose the client to aggressive behavior. Nurses should be alert to the risk for self or other directed violence and intervene immediately at the first signs of agitation or aggression. KEY: Cognitive Level: Analysis | Integrated Processes: Nursing Process: Analysis | Client Need: Safe and Effective Care Environment: Management of Care

When questioned about bruises, a woman states, "It was an accident. My husband just had a bad day at work. He's being so gentle now and even brought me flowers. He's going to get a new job, so it won't happen again." This client is in which phase of the cycle of battering? A. Phase I: The tension-building phase B. Phase II: The acute battering incident phase C. Phase III: The honeymoon phase D. Phase IV: The resolution and reorganization phase

ANS: C The client is in the honeymoon phase of the cycle of battering. In this phase, the batterer becomes extremely loving, kind, and contrite. Promises are often made that the abuse will not happen again. KEY: Cognitive Level: Application | Integrated Processes: Nursing Process: Assessment | Client Need: Psychosocial Integrity

18. The nurse working in the role of educator with groups of clients may: a. Care for clients in acute care settings. b. Coordinate activities with staff members in community settings. c. Use learning principles to increase understanding about mental illness. d. Act as an advocate for mental health support groups.

ANS: C The educator role is foundational to health maintenance, health promotion, and community action.

How should a nurse best describe the major maladaptive client response to panic disorder? A. Clients overuse medical care because of physical symptoms. B. Clients use illegal drugs to ease symptoms. C. Clients perceive having no control over life situations. D. Clients develop compulsions to deal with anxiety.

ANS: C The major maladaptive client response to panic disorder is the perception of having no control over life situations, which leads to nonparticipation in decision making and doubts regarding role performance. KEY: Cognitive Level: Application | Integrated Processes: Nursing Process: Assessment | Client Need: Psychosocial Integrity

A newly admitted homeless client diagnosed with schizophrenia states, "I have been living in a cardboard box for 2 weeks. Why did the government let me down?" Which is an appropriate nursing reply? A. "Your discharge from the state hospital was done prematurely. Had you remained in the state hospital longer, you would not be homeless." B. "Your premature discharge from the state hospital was not intended for patients diagnosed with chronic schizophrenia." C. "Your discharge from the state hospital was based on firm principles; however, the resources were not available to make the transition a success." D. "Your discharge from the state hospital was based on presumed family support, and this was not forthcoming."

ANS: C The most accurate nursing reply is to explain to the client that the resources were not available for successful transitioning out of a state hospital to the community. There are several factors that are thought to contribute to homelessness among the mentally ill: deinstitutionalization, poverty, lack of affordable housing, lack of affordable health care, domestic violence, and addiction disorders. KEY: Cognitive Level: Application | Integrated Processes: Nursing Process: Implementation | Client Need: Psychosocial Integrity

3. A client diagnosed with paranoid personality disorder becomes violent on a unit. Which nursing intervention is most appropriate? A. Provide objective evidence that violence is unwarranted. B. Initially restrain the client to maintain safety. C. Use clear, calm statements and a confident physical stance. D. Empathize with the client's paranoid perceptions.

ANS: C The most appropriate nursing intervention is to use clear, calm statements and to assume a confident physical stance. A calm attitude avoids escalating the aggressive behavior and provides the client with a feeling of safety and security. It may also be beneficial to have sufficient staff on hand to present a show of strength.

A woman comes to an emergency department with a broken nose and multiple bruises after being beaten by her husband. She states, "The beatings have been getting worse, and I'm afraid that next time he might kill me." Which is the appropriate nursing reply? A. "Leopards don't change their spots, and neither will he." B. "There are things you can do to prevent him from losing control." C. "Let's talk about your options so that you don't have to go home." D. "Why don't we call the police so that they can confront your husband with his behavior?"

ANS: C The most appropriate reply by the nurse is to talk with the client about options so that the client does not have to return to the abusive environment. It is essential that clients make decisions independently without the nurse being the "rescuer." Imposing judgments and giving advice is nontherapeutic. KEY: Cognitive Level: Application | Integrated Processes: Nursing Process: Implementation | Client Need: Psychosocial Integrity

2. A client diagnosed with antisocial personality disorder comes to a nurses' station at 11:00 p.m., requesting to phone a lawyer to discuss filing for a divorce. The unit rules state that no phone calls are permitted after 10:00 p.m. Which nursing reply is most appropriate? A. "Go ahead and use the phone. I know this pending divorce is stressful." B. "You know better than to break the rules. I'm surprised at you." C. "It is after the 10:00 p.m. phone curfew. You will be able to call tomorrow." D. "The decision to divorce should not be considered until you have had a good night's sleep."

ANS: C The most appropriate response by the staff is to restate the unit rules in a calm, assertive manner. Because of the probability of manipulative behavior in this client population, it is imperative to maintain consistent application of rules.

A client is experiencing a severe panic attack. Which nursing intervention would meet this client's immediate need? A. Teach deep breathing relaxation exercises B. Place the client in a Trendelenburg position C. Stay with the client and offer reassurance of safety D. Administer the ordered prn buspirone (BuSpar)

ANS: C The nurse can meet this client's immediate need by staying with the client and offering reassurance of safety and security. The client may fear for his or her life, and the presence of a trusted individual provides assurance of personal safety. KEY: Cognitive Level: Application | Integrated Processes: Implementation | Client Need: Psychosocial Integrity

4. A highly emotional client presents at an outpatient clinic appointment wearing flamboyant attire, spiked heels, and theatrical makeup. Which personality disorder should a nurse associate with this assessment data? A. Compulsive personality disorder B. Schizotypal personality disorder C. Histrionic personality disorder D. Manic personality disorder

ANS: C The nurse should associate histrionic personality disorder with this assessment data. Individuals diagnosed with histrionic personality disorder tend to be self-dramatizing, attention seeking, overly gregarious, and seductive. They often use manipulation and exhibitionism as a means of gaining attention.

A nursing student asks an emergency department nurse, "Why does a rapist use a weapon during the act of rape?" Which nursing reply is most accurate? A. "A weapon is used to increase the victimizer's security." B. "A weapon is used to inflict physical harm." C. "A weapon is used to terrorize and subdue the victim." D. "A weapon is used to mirror learned family behavior patterns."

ANS: C The nurse should explain that a rapist uses weapons to terrorize and subdue the victim. Rape is the expression of power and dominance by means of sexual violence. Rape can occur over a broad spectrum of experience, from violent attack to insistence on sexual intercourse by an acquaintance or spouse. KEY: Cognitive Level: Application | Integrated Processes: Nursing Process: Implementation | Client Need: Psychosocial Integrity

A client living on the beachfront seeks help with an extreme fear of crossing bridges, which interferes with daily life. A psychiatric nurse practitioner decides to try systematic desensitization. Which explanation of this therapy should the nurse convey to the client? A. "Using your imagination, we will attempt to achieve a state of relaxation that you can replicate when faced with crossing a bridge." B. "Because anxiety and relaxation are mutually exclusive states, we can attempt to substitute a relaxation response for the anxiety response." C. "Through a series of increasingly anxiety-provoking steps, we will gradually increase your tolerance to anxiety." D. "In one intense session, you will be exposed to a maximum level of anxiety that you will learn to tolerate."

ANS: C The nurse should explain to the client that systematic desensitization exposes the client to a series of increasingly anxiety-provoking steps that will gradually increase anxiety tolerance. Systematic desensitization was introduced by Joseph Wolpe in 1958 and is based on behavioral conditioning principles. KEY: Cognitive Level: Application | Integrated Processes: Nursing Process: Implementation | Client Need: Psychosocial Integrity

A client began taking lithium for the treatment of bipolar disorder approximately 1 month ago. The client asks if it is normal to have gained 12 pounds in this time frame. Which is the appropriate nursing reply? A. "That's strange. Weight loss is the typical pattern." B. "What have you been eating? Weight gain is not usually associated with lithium." C. "Weight gain is a common but troubling side effect." D. "Weight gain occurs only during the first month of treatment with this drug."

ANS: C The nurse should explain to the client that weight gain is a common side effect of lithium carbonate. The nurse should educate the client on the importance of medication compliance and discuss concerns with the prescribing physician if the client does not wish to continue taking the medication. KEY: Cognitive Level: Application | Integrated Processes: Teaching/Learning | Client Need: Health Promotion and Maintenance

19. Which reaction to a compliment from another client should a nurse identify as a typical response from a client diagnosed with avoidant personality disorder? A. Interpreting the compliment as a secret code used to increase personal power B. Feeling the compliment was well deserved C. Being grateful for the compliment but fearing later rejection and humiliation D. Wondering what deep meaning and purpose are attached to the compliment

ANS: C The nurse should identify that a client diagnosed with avoidant personality disorder would be grateful for the comment but would fear later rejection and humiliation. Individuals with avoidant personality disorder are extremely sensitive to rejection and are often awkward and uncomfortable in social situations.

Which treatment should a nurse identify as most appropriate for clients diagnosed with generalized anxiety disorder (GAD)? A. Long-term treatment with diazepam (Valium) B. Acute symptom control with citalopram (Celexa) C. Long-term treatment with buspirone (BuSpar) D. Acute symptom control with ziprasidone (Geodon)

ANS: C The nurse should identify that an appropriate treatment for clients diagnosed with GAD is long-term treatment with buspirone. Buspirone is an anxiolytic medication that is effective in 60% to 80% of clients with GAD. It takes 10 to 14 days for alleviation of symptoms but does not have the dependency concerns of other anxiolytics. KEY: Cognitive Level: Application | Integrated Processes: Nursing Process: Implementation | Client Need: Physiological Integrity: Pharmacological and Parenteral Therapies

18. Looking at a slightly bleeding paper cut, the client screams, "Somebody help me, quick! I'm bleeding. Call 911!" A nurse should identify this behavior as characteristic of which personality disorder? A. Schizoid personality disorder B. Obsessive-compulsive personality disorder C. Histrionic personality disorder D. Paranoid personality disorder

ANS: C The nurse should identify this behavior as characteristic of histrionic personality disorder. Individuals diagnosed with this disorder tend to be self-dramatizing, attention seeking, over-gregarious, and seductive.

A nurse is planning care for a client diagnosed with bipolar disorder: manic episode. In which order should the nurse prioritize the listed client outcomes? Client Outcomes: 1. Maintains nutritional status. 2. Interacts appropriately with peers. 3. Remains free from injury. 4. Sleeps 6 to 8 hours a night. A. 2, 1, 3, 4 B. 4, 1, 2, 3 C. 3, 1, 4, 2 D. 1, 4, 2, 3

ANS: C The nurse should order client outcomes based on priority in the following order: Remains free of injury, maintains nutritional status, sleeps 6 to 8 hours a night, and interacts appropriately with peers. The nurse should prioritize the client's physical and safety needs. KEY: Cognitive Level: Analysis | Integrated Processes: Nursing Process: Planning | Client Need: Safe and Effective Care Environment: Management of Care

A client who has been raped is crying, pacing, and cursing her attacker in an emergency department. Which behavioral defense should a nurse recognize? A. Controlled response pattern B. Compounded rape reaction C. Expressed response pattern D. Silent rape reaction

ANS: C The nurse should recognize that this client is exhibiting an expressed response pattern. In the expressed response pattern, feelings of fear, anger, and anxiety are expressed through crying, sobbing, smiling, restlessness, and tension. In the controlled response pattern, the client's feelings are masked or hidden, and a calm, composed, or subdued affect is seen. KEY: Cognitive Level: Application | Integrated Processes: Nursing Process: Assessment | Client Need: Psychosocial Integrity

20. Which client symptoms should lead a nurse to suspect a diagnosis of obsessive-compulsive personality disorder? A. The client experiences unwanted, intrusive, and persistent thoughts. B. The client experiences unwanted, repetitive behavior patterns. C. The client experiences inflexibility and lack of spontaneity when dealing with others. D. The client experiences obsessive thoughts that are externally imposed.

ANS: C The nurse should suspect a diagnosis of obsessive-compulsive personality disorder when a client experiences inflexibility and lack of spontaneity. Individuals diagnosed with this disorder are very serious and formal and have difficulty expressing emotions. They are perfectionistic and preoccupied with rules.

A nurse is implementing care within the parameters of tertiary prevention. Which nursing action is an example of this type of care? A. Teaching an adolescent about pregnancy prevention B. Teaching an elderly client the reportable side effects of a newly prescribed neuroleptic medication C. Teaching a client with schizophrenia to cook meals, make a grocery list, and establish a budget D. Teaching a client about his or her new diagnosis of bipolar disorder

ANS: C The nurse who teaches a client to cook meals, make a grocery list, and establish a budget is implementing care within the parameters of tertiary prevention. Tertiary prevention is services aimed at reducing the residual effects that are associated with severe and persistent mental illness. It is accomplished by promoting rehabilitation that is directed toward achievement of maximum functioning. KEY: Cognitive Level: Application | Integrated Processes: Nursing Process: Implementation | Client Need: Psychosocial Integrity

22. When planning care for clients diagnosed with personality disorders, what should be the anticipated treatment outcome? A. To stabilize pathology with the correct combination of medications B. To change the characteristics of the dysfunctional personality C. To reduce inflexibility of personality traits that interfere with functioning and relationships D. To decrease the prevalence of neurotransmitters at receptor sites

ANS: C The outcome of treatment for clients diagnosed with personality disorders should be to reduce inflexibility of personality traits that interfere with functioning and relationships. Personality disorders are often difficult and, in some cases, seem impossible to treat.

A college student is unable to take a final examination because of severe test anxiety. Instead of studying, the student relieves stress by attending a movie. Which priority nursing diagnosis should a campus nurse assign for this client? A. Noncompliance R/T test taking B. Ineffective role performance R/T helplessness C. Altered coping R/T anxiety D. Powerlessness R/T fear

ANS: C The priority nursing diagnosis for this client is altered coping R/T anxiety. The nurse should assist in implementing interventions that should improve the client's healthy coping skills and reduce anxiety. KEY: Cognitive Level: Analysis | Integrated Processes: Nursing Process: Analysis | Client Need: Psychosocial Integrity

An adult client diagnosed with bipolar I disorder is prescribed lamotrigine (Lamictal), 400 mg three times a day, for mood stabilization. Which is a true statement about this medication order? A. This dosage is within the recommended dosage range. B. This dosage is lower than the recommended dosage range. C. This dosage is more than twice the recommended dosage range. D. This dosage is four times higher than the recommended dosage range.

ANS: C The recommended dose of lamotrigine for treatment of bipolar disorder in adult clients should not exceed 400 mg daily. KEY: Cognitive Level: Application | Integrated Processes: Implementation | Client Need: Physiological Integrity: Pharmacological and Parenteral Therapies

A client diagnosed with generalized anxiety states, "I know the best thing for me to do now is to just forget my worries." How should the nurse evaluate this statement? A. The client is developing insight. B. The client's coping skills are improving. C. The client has a distorted perception of problem resolution. D. The client is meeting outcomes and moving toward discharge.

ANS: C This client has a distorted perception of how to deal with the problem of anxiety. Clients should be encouraged to openly deal with anxiety and recognize the triggers that precipitate anxiety responses. KEY: Cognitive Level: Application | Integrated Processes: Nursing Process: Assessment | Client Need: Psychosocial Integrity

In the emergency department, a raped client appears calm and exhibits a blunt affect. The client answers a nurse's questions in a monotone using single words. How should the nurse interpret this client's responses? A. The client may be lying about the incident. B. The client may be experiencing a silent rape reaction. C. The client may be demonstrating a controlled response pattern. D. The client may be having a compounded rape reaction.

ANS: C This client is most likely demonstrating a controlled response pattern. In a controlled response pattern, the client's feelings are masked or hidden, and a calm, composed, or subdued affect is seen. In the expressed response pattern, feelings of fear, anger, and anxiety are expressed through crying, sobbing, smiling, restlessness, and tension. KEY: Cognitive Level: Application | Integrated Processes: Nursing Process: Assessment | Client Need: Psychosocial Integrity

2. The first major piece of legislation to influence mental health services in the United States was the: a. National Mental Health Act b. Mental Health Study Act c. Social Security Act d. Protection and Advocacy for Mentally Ill Individuals Act

ANS: C This was in response to economic and social problems of the era and shifted the responsibility of care for ill people from the state to the federal government.

When a home health nurse administers an outpatient's injection of haloperidol decanoate (Haldol decanoate), which level of care is the nurse providing? A. Primary prevention level of care B. Secondary prevention level of care C. Tertiary prevention level of care D. Case management level of care

ANS: C When administering this long-acting antipsychotic medication, the nurse is providing a tertiary prevention level of care. Tertiary prevention services are aimed at reducing the residual effects associated with severe and persistent mental illness. It is accomplished by preventing complications of the illness and promoting rehabilitation directed toward achievement of maximum functioning. KEY: Cognitive Level: Application | Integrated Processes: Nursing Process: Implementation | Client Need: Psychosocial Integrity

Which intervention should the nurse consider as primary prevention for an individual who is on the verge of being homeless because of a job layoff? A. Referral to primary care provider to improve general health status B. Encouraging client to recognize reasons for job layoff C. Job training to increase employment options D. Encouraging the use of prn medications to control symptoms

ANS: C When the nurse implements primary prevention interventions, the nurse is providing services aimed at reducing the incidences of mental disorders within the population. In this situation, there is emphasis on providing education and support to unemployed or homeless individuals. KEY: Cognitive Level: Application | Integrated Processes: Nursing Process: Implementation | Client Need: Psychosocial Integrity

19. According to NANDA (2012), a disorder that occurs after the death of a significant other or any loss perceived as significant to the individual, in which the experience of distress accompanying bereavement fails to follow normative expectations and manifests in functional impairment, is referred to as________________________.

ANS: Complicated grieving A grieving process that does not follow normative expectations may include fixation at a particular stage of grieving, psychosomatic symptoms, and/or impairment in occupational, social, intellectual, or emotional function.

14. Which of these statements by the patient are indications of complicated grieving? A. I feel like I should have been the one to die in that hurricane. B. Last year, several of my coworkers died in a hurricane and I still cant go back to work. C. Ive been having incapacitating migraines ever since the memorial services. D. All of the above

ANS: D Item A indicates survivor guilt, and items B and C are both indications that the trauma has contributed to functional impairment. All three are symptoms of complicated grieving.

10. In the role of milieu manager, which activity should the nurse prioritize? A. Setting the schedule for the daily unit activities B. Evaluating clients for medication effectiveness C. Conducting therapeutic group sessions D. Searching newly admitted clients for hazardous objects

ANS: D The milieu manager should search newly admitted clients for hazardous objects. Safety of the client and others is the priority. Nurses are responsible for ensuring that the clients safety and physiological needs are met within the milieu.

2. A client on an inpatient unit angrily states to a nurse, Peter is not cleaning up after himself in the community bathroom. You need to address this problem. Which is the appropriate nursing response? A. Ill talk to Peter and present your concerns. B. Why are you overreacting to this issue? C. You should bring this to the attention of your treatment team. D. I can see that you are angry. Lets discuss ways to approach Peter with your concerns.

ANS: D The most appropriate nursing response involves restating the clients feeling and developing a plan with the client to solve the problem. According to Skinner, every interaction in the therapeutic milieu is an opportunity for therapeutic intervention to improve communication and relationship-development skills.

A client is diagnosed with dissociative identity disorder (DID). What is the primary goal of therapy for this client? A. To recover memories and improve thinking patterns B. To prevent social isolation C. To decrease anxiety and need for secondary gain D. To collaborate among subpersonalities to improve functioning

ANS: D The nurse should anticipate that the primary therapeutic goal for a client diagnosed with DID is to collaborate among subpersonalities to improve functioning. Some clients choose to pursue a lengthy therapeutic regimen to achieve integration, a blending of all the personalities into one. The goal is to optimize the clients ability to function appropriately and achieve optimal personal potential.

18. A client is newly diagnosed with obsessive-compulsive disorder and spends 45 minutes folding clothes and rearranging them in drawers. Which nursing intervention would best address this clients problem? A. Distract the client with other activities whenever ritual behaviors begin. B. Report the behavior to the psychiatrist to obtain an order for medication dosage increase. C. Lock the room to discourage ritualistic behavior. D. Discuss the anxiety-provoking triggers that precipitate the ritualistic behaviors.

ANS: D The nurse should discuss with the client the anxiety-provoking triggers that precipitate the ritualistic behavior. If the client is going to be able to avoid the anxiety, he or she must first learn to recognize precipitating factors. Attempting to distract the client, seeking medication increase, and locking the clients room are not appropriate interventions because they do not help the client recognize anxiety triggers.

6. A nurse working on an inpatient psychiatric unit is assigned to conduct a 45-minute education group. What should the nurse identify as an appropriate group topic? A. Dream analysis B. Creative cooking C. Paint by number D. Stress management

ANS: D The nurse should identify that teaching clients about stress management is an appropriate education group topic. Nurses should be able to perform the role of client teacher in the psychiatric area. Nurses need to be able to assess a clients learning readiness. Other topics for education groups include medical diagnoses, side effects of medications, and the importance of medication compliance.

17. A client diagnosed with an obsessive-compulsive disorder spends hours bathing and grooming. During a one-on-one interaction, the client discusses the rituals in detail but avoids any feelings that the rituals generate. Which defense mechanism should the nurse identify? A. Sublimation B. Dissociation C. Rationalization D. Intellectualization

ANS: D The nurse should identify that the client is using the defense mechanism of intellectualization when discussing the rituals of obsessive-compulsive disorder in detail while avoiding discussion of feelings. Intellectualization is an attempt to avoid expressing emotions associated with a stressful situation by using the intellectual processes of logic, reasoning, and analysis.

4. How would a nurse differentiate a client diagnosed with panic disorder from a client diagnosed with generalized anxiety disorder (GAD)? A. GAD is acute in nature, and panic disorder is chronic. B. Chest pain is a common GAD symptom, whereas this symptom is absent in panic disorders. C. Hyperventilation is a common symptom in GAD and rare in panic disorder. D. Depersonalization is commonly seen in panic disorder and absent in GAD.

ANS: D The nurse should recognize that a client diagnosed with panic disorder experiences depersonalization, whereas a client diagnosed with GAD would not. Depersonalization refers to being detached from oneself when experiencing extreme anxiety.

8. A cab driver stuck in traffic is suddenly lightheaded, tremulous, and diaphoretic and experiences tachycardia and dyspnea. An extensive workup in an emergency department reveals no pathology. Which medical diagnosis is suspected, and what nursing diagnosis takes priority? A. Generalized anxiety disorder and a nursing diagnosis of fear B. Altered sensory perception and a nursing diagnosis of panic disorder C. Pain disorder and a nursing diagnosis of altered role performance D. Panic disorder and a nursing diagnosis of panic anxiety

ANS: D The nurse should suspect that the client has exhibited signs/symptoms of a panic disorder. The priority nursing diagnosis should be panic anxiety. Panic disorder is characterized by recurrent, sudden-onset panic attacks in which the person feels intense fear, apprehension, or terror.

When a nurse attempts to provide health-care services to the homeless, what should be a realistic concern? A. Most individuals who are homeless reject help. B. Most individuals who are homeless are suspicious of anyone who offers help. C. Most individuals who are homeless are proud and will often refuse charity. D. Most individuals who are homeless relocate frequently.

ANS: D A realistic concern in the provision of health-care services to the homeless is that individuals who are homeless relocate frequently. Frequent relocation confounds service delivery and interferes with providers' efforts to ensure appropriate care. KEY: Cognitive Level: Application | Integrated Processes: Nursing Process: Evaluation | Client Need: Safe and Effective Care Environment

12. Which nursing diagnosis should a nurse identify as appropriate when working with a client diagnosed with schizoid personality disorder? A. Altered thought processes R/T increased stress B. Risk for suicide R/T loneliness C. Risk for violence: directed toward others R/T paranoid thinking D. Social isolation R/T inability to relate to others

ANS: D An appropriate nursing diagnosis when working with a client diagnosed with schizoid personality disorder is social isolation R/T inability to relate to others. Clients diagnosed with schizoid personality disorder appear cold, aloof, and indifferent to others. They prefer to work in isolation and are unsociable.

25. Which client statement would demonstrate a common characteristic of Cluster "B" personality disorder? A. "I wish someone would make that decision for me." B. "I built this building by using materials from outer space." C. "I'm afraid to go to group because it is crowded with people." D. "I didn't have the money for the ring, so I just took it."

ANS: D Antisocial personality disorder is included in the Cluster "B" personality disorders. In this disorder there is a pervasive pattern of disregard for and violation of the rights of others.

A client diagnosed with schizophrenia was released from a state mental hospital after 20 years of institutionalization. A nurse should recognize which characteristic that is likely to be exhibited by this client? A. The client is likely to be compliant with treatment because of institutional dependency. B. The client is likely to find a variety of community support services to aid in the transition. C. The client is likely to adjust to the community environment if given sufficient support. D. The client is likely to be admitted at some time to an acute care unit for psychiatric treatment.

ANS: D Because of the chronic nature of this client's diagnosis and commonly occurring medication noncompliance, the nurse would expect recidivism during the course of the illness. KEY: Cognitive Level: Application | Integrated Processes: Nursing Process: Evaluation | Client Need: Psychosocial Integrity

The inpatient psychiatric unit is being redecorated. At a unit meeting, staff discusses bedroom décor for clients experiencing mania. The nurse manager evaluates which suggestion as most appropriate? A. Rooms should contain extra-large windows with views of the street. B. Rooms should contain brightly colored walls with printed drapes. C. Rooms should be painted deep colors and located close to the nurse's station. D. Rooms should be painted with neutral colors and contain pale-colored accessories.

ANS: D Clients experiencing mania are subject to frequent mood variations, easily changing from irritability and anger to sadness and crying. Therefore, it is necessary to maintain low levels of stimuli in the client's environment (low lighting, few people, simple décor, low noise levels). Anxiety levels rise in a stimulating environment. Neutral colors and pale accessories are most appropriate for a client experiencing mania. KEY: Cognitive Level: Analysis | Integrated Processes: Nursing Process: Implementation | Client Need: Psychosocial Integrity

When intervening with a married couple experiencing relationship discord, which reflects a nursing intervention at the secondary level of prevention? A. Assessing how the children are coping with the parents' relationship issues B. Supplying the couple with guidelines related to marital seminar leadership C. Teaching the couple about various methods of birth control D. Counseling the couple in relation to open and honest communication skills

ANS: D Counseling the couple in relation to open and honest communication skills is reflective of a nursing intervention at the secondary level of prevention. Secondary prevention aims at minimizing symptoms and is accomplished through early identification of problems and prompt initiation of effective treatment. KEY: Cognitive Level: Analysis | Integrated Processes: Nursing Process: Implementation | Client Need: Psychosocial Integrity

31. A client exhibits dependency on staff and peers and expresses fear of abandonment. Using Mahler's theory of object relations, which should the nurse expect to note in this client's childhood? A. Lack of fulfillment of basic needs by parental figures B. Absence of the client's maternal figure during symbiosis C. Difficulty establishing trust with the maternal figure D. Inconsistency by the maternal figure during individuation

ANS: D During phase 3 (5 to 36 months) of Margaret Mahler's individuation theory, there should be a strengthening of the ego and an acceptance of "self" with independent ego boundaries. Inconsistency by the maternal figure during individuation may in later years result in feelings of helplessness when the client is alone because of exaggerated fears of being unable to care for self.

4. An individual living during which era would have viewed physical and mental illness as interrelated, resulting from physical conditions? a. Ancient times b. Middle Ages c. Colonial d. Greco-Roman

ANS: D During the ancient times, Middle Ages, and colonial times, mental illness was viewed as resulting from supernatural forces.

10. A client who has been diagnosed with major depression works with other mental health consumers to advocate for establishment of additional self-help services for individuals and families with mental illness in a community. This client is likely involved with which organization? a. Community Support Program (CSP) b. National Institute of Mental Health (NIMH) c. Mental Health Study d. National Alliance for the Mentally Ill (NAMI)

ANS: D NAMI was the first consumer group to advocate for better services. This consumer advocacy group worked to establish education and self-help services for individuals and families with mental illness.

17. An example of tertiary prevention concerning mental health is: a. Screening for anxiety b. Depression education c. Counseling following a natural disaster d. Coordination transition from the hospital to the community

ANS: D Tertiary prevention efforts attempt to restore and enhance functioning, thus coordination of transition from the hospital to the community is tertiary prevention. Screening and counseling are secondary prevention activities, and depression education is a primary prevention.

14. Which client situation should a nurse identify as reflective of the impulsive behavior that is commonly associated with borderline personality disorder? A. As the day shift nurse leaves the unit, the client suddenly hugs the nurse's arm and whispers, "The night nurse is evil. You have to stay." B. As the day shift nurse leaves the unit, the client suddenly hugs the nurse's arm and states, "I will be up all night if you don't stay with me." C. As the day shift nurse leaves the unit, the client suddenly hugs the nurse's arm, yelling, "Please don't go! I can't sleep without you being here." D. As the day shift nurse leaves the unit, the client suddenly shows the nurse a bloody arm and states, "I cut myself because you are leaving me."

ANS: D The client's statement "I cut myself because you are leaving me" reflects impulsive behavior that is commonly associated with the diagnosis of borderline personality disorder. Repetitive, self-mutilative behaviors are common and are generated by feelings of abandonment following separation from significant others.

14. A nurse considers the effects of biology and environment, or nature and nurture, on the development of mental illness when providing care for clients with mental illness. Which nursing model/theory is being applied? a. Community mental health model b. Holistic model c. Systems theory d. Diathesis-stress model

ANS: D The diathesis-stress model integrates the effects of biology and environment on the development of mental illness. Certain genes or genetic combinations produce a predisposition to a disorder. When an environmental stressor challenges an individual with a predisposition to a disorder, the expression of a mental disorder may result.

A nursing instructor is teaching students about the differences between partial and inpatient hospitalization. In what way does partial hospitalization differ from traditional inpatient hospitalization? A. Partial hospitalization does not provide medication administration and monitoring. B. Partial hospitalization does not use an interdisciplinary team. C. Partial hospitalization does not offer a comprehensive treatment plan. D. Partial hospitalization does not provide supervision 24 hours a day.

ANS: D The instructor should explain that partial hospitalization does not provide supervision 24 hours a day. Partial hospitalization programs generally offer a comprehensive treatment plan formulated by an interdisciplinary team. This has proved to be an effective method of preventing hospitalization. KEY: Cognitive Level: Application | Integrated Processes: Nursing Process: Planning | Client Need: Psychosocial Integrity

Which nursing statement best describes the current nature of mental health care in the community? A. "All homeless people have a history of institutionalization and are frequently admitted to acute care settings." B. "In the United States, the rate of serious mental illness in the prison population is the same as the general population." C. "The deinstitutionalization movement in the United States was successful in transitioning clients into the community." D. "Today, the majority of clients admitted to psychiatric hospitals are in a crisis stage, and the treatment goal is stabilization."

ANS: D The majority of clients admitted to psychiatric hospitals are in a crisis stage, and the treatment goal is stabilization and reintroduction into the community. Crisis situations can occur because of treatment noncompliance and exacerbations of the chronic mental illness. KEY: Cognitive Level: Application | Integrated Processes: Nursing Process: Evaluation | Client Need: Psychosocial Integrity

10. During an interview, which client statement indicates to a nurse that a potential diagnosis of schizotypal personality disorder should be considered? A. "I really don't have a problem. My family is inflexible, and every relative is out to get me." B. "I am so excited about working with you. Have you noticed my new nail polish, 'Ruby Red Roses'?" C. "I spend all my time tending my bees. I know a whole lot of information about bees." D. "I am getting a message from the beyond that we have been involved with each other in a previous life."

ANS: D The nurse should assess that a client who states that he or she is getting a message from the beyond indicates a potential diagnosis of schizotypal personality disorder. Individuals with schizotypal personality disorder are aloof and isolated and behave in a bland and apathetic manner. The individual experiences magical thinking, ideas of reference, illusions, and depersonalization as part of daily life.

A client is newly diagnosed with obsessive-compulsive disorder and spends 45 minutes folding clothes and rearranging them in drawers. Which nursing intervention would best address this client's problem? A. Distract the client with other activities whenever ritual behaviors begin. B. Report the behavior to the psychiatrist to obtain an order for medication dosage increase. C. Lock the room to discourage ritualistic behavior. D. Discuss the anxiety-provoking triggers that precipitate the ritualistic behaviors.

ANS: D The nurse should discuss with the client the anxiety-provoking triggers that precipitate the ritualistic behavior. If the client is going to be able to avoid the anxiety, he or she must first learn to recognize precipitating factors. Attempting to distract the client, seeking medication increase, and locking the client's room are not appropriate interventions because they do not help the client recognize anxiety triggers. KEY: Cognitive Level: Application | Integrated Processes: Nursing Process: Implementation | Client Need: Psychosocial Integrity

A highly agitated client paces the unit and states, "I could buy and sell this place." The client's mood fluctuates from fits of laughter to outbursts of anger. Which is the most accurate documentation of this client's behavior? A. "Rates mood 8/10. Exhibiting looseness of association. Euphoric." B. "Mood euthymic. Exhibiting magical thinking. Restless." C. "Mood labile. Exhibiting delusions of reference. Hyperactive." D. "Agitated and pacing. Exhibiting grandiosity. Mood labile."

ANS: D The nurse should document that this client's behavior is "Agitated and pacing. Exhibiting grandiosity. Mood labile." The client is exhibiting signs of irritation accompanied by aggressive behavior. Grandiosity refers to an exaggerated sense of power, importance, knowledge, or identity. KEY: Cognitive Level: Application | Integrated Processes: Communication and Documentation | Client Need: Safe and Effective Care Environment

A client diagnosed with bipolar I disorder is exhibiting severe manic behaviors. A physician prescribes lithium carbonate (Eskalith) and olanzapine (Zyprexa). The client's spouse questions the Zyprexa order. Which is the appropriate nursing reply? A. "Zyprexa in combination with Eskalith cures manic symptoms." B. "Zyprexa prevents extrapyramidal side effects." C. "Zyprexa ensures a good night's sleep." D. "Zyprexa calms hyperactivity until the Eskalith takes effect."

ANS: D The nurse should explain to the client's spouse that Zyprexa can calm hyperactivity until the Eskalith takes effect. Eskalith may take 1 to 3 weeks to begin to decrease hyperactivity. Zyprexa is classified as an antipsychotic and can be used to immediately to reduce hyperactive symptoms in acute manic episodes. KEY: Cognitive Level: Application | Integrated Processes: Teaching/Learning | Client Need: Health Promotion and Maintenance

17. A nurse tells a client that the nursing staff will start alternating weekend shifts. Which response should a nurse identify as characteristic of clients diagnosed with obsessive-compulsive personality disorder? A. "You really don't have to go by that schedule. I'd just stay home sick." B. "There has got to be a hidden agenda behind this schedule change." C. "Who do you think you are? I expect to interact with the same nurse every Saturday." D. "You can't make these kinds of changes! Isn't there a rule that governs this decision?"

ANS: D The nurse should identify that a client diagnosed with obsessive-compulsive personality disorder would have a difficult time accepting change. This disorder is characterized by inflexibility and lack of spontaneity. Individuals diagnosed with this disorder are very serious, formal, over-disciplined, perfectionistic, and preoccupied with rules.

A client diagnosed with an obsessive-compulsive disorder spends hours bathing and grooming. During a one-on-one interaction, the client discusses the rituals in detail but avoids any feelings that the rituals generate. Which defense mechanism should the nurse identify? A. Sublimation B. Dissociation C. Rationalization D. Intellectualization

ANS: D The nurse should identify that the client is using the defense mechanism of intellectualization when discussing the rituals of obsessive-compulsive disorder in detail while avoiding discussion of feelings. Intellectualization is an attempt to avoid expressing emotions associated with a stressful situation by using the intellectual processes of logic, reasoning, and analysis. KEY: Cognitive Level: Application | Integrated Processes: Nursing Process: Assessment | Client Need: Psychosocial Integrity

A client diagnosed with bipolar disorder, who has taken lithium carbonate (Lithane) for 1 year, presents in an emergency department with severe diarrhea, blurred vision, and tinnitus. How should the nurse interpret these symptoms? A. Symptoms indicate consumption of foods high in tyramine. B. Symptoms indicate lithium carbonate discontinuation syndrome. C. Symptoms indicate the development of lithium carbonate tolerance. D. Symptoms indicate lithium carbonate toxicity.

ANS: D The nurse should interpret that the client's symptoms indicate lithium carbonate toxicity. The initial signs of toxicity include ataxia, blurred vision, severe diarrhea, nausea and vomiting, and tinnitus. Lithium levels should be monitored monthly during maintenance therapy to ensure proper dosage. KEY: Cognitive Level: Application | Integrated Processes: Nursing Process: Assessment | Client Need: Physiological Integrity: Pharmacological and Parenteral Therapies

Which teaching should the nurse in an employee assistance program provide to an employee who exhibits symptoms of domestic physical abuse? A. Have ready access to a gun and learn how to use it B. Research lawyers who can aid in divorce proceedings C. File charges of assault and battery D. Have ready access to the number of a safe house for battered women

ANS: D The nurse should provide information about safe houses for battered women when working with a client who has symptoms of domestic physical abuse. Many women feel powerless within the abusive relationship and may be staying in the abusive relationship out of fear for their lives. KEY: Cognitive Level: Application | Integrated Processes: Nursing Process: Implementation | Client Need: Safe and Effective Care Environment

How would a nurse differentiate a client diagnosed with panic disorder from a client diagnosed with generalized anxiety disorder (GAD)? A. GAD is acute in nature, and panic disorder is chronic. B. Chest pain is a common GAD symptom, whereas this symptom is absent in panic disorders. C. Hyperventilation is a common symptom in GAD and rare in panic disorder. D. Depersonalization is commonly seen in panic disorder and absent in GAD.

ANS: D The nurse should recognize that a client diagnosed with panic disorder experiences depersonalization, whereas a client diagnosed with GAD would not. Depersonalization refers to being detached from oneself when experiencing extreme anxiety. KEY: Cognitive Level: Analysis | Integrated Processes: Nursing Process: Assessment | Client Need: Psychosocial Integrity

A cab driver stuck in traffic is suddenly lightheaded, tremulous, and diaphoretic and experiences tachycardia and dyspnea. An extensive workup in an emergency department reveals no pathology. Which medical diagnosis is suspected, and what nursing diagnosis takes priority? A. Generalized anxiety disorder and a nursing diagnosis of fear B. Altered sensory perception and a nursing diagnosis of panic disorder C. Pain disorder and a nursing diagnosis of altered role performance D. Panic disorder and a nursing diagnosis of panic anxiety

ANS: D The nurse should suspect that the client has exhibited signs/symptoms of a panic disorder. The priority nursing diagnosis should be panic anxiety. Panic disorder is characterized by recurrent, sudden-onset panic attacks in which the person feels intense fear, apprehension, or terror. KEY: Cognitive Level: Analysis | Integrated Processes: Nursing Process: Analysis | Client Need: Psychosocial Integrity

A woman describes a history of physical and emotional abuse in intimate relationships. Which additional factor should a nurse suspect? A. The woman may be exhibiting a controlled response pattern. B. The woman may have a history of childhood neglect. C. The woman may be exhibiting codependent characteristics. D. The woman might be a victim of incest.

ANS: D The nurse should suspect that this client might be a victim of incest. Women in abusive relationships often grew up in abusive homes. KEY: Cognitive Level: Application | Integrated Processes: Nursing Process: Assessment | Client Need: Psychosocial Integrity

A nursing instructor is discussing various challenges in the treatment of clients diagnosed with bipolar disorder. Which student statement demonstrates an understanding of the most critical challenge in the care of these clients? A. "Treatment is compromised when clients can't sleep." B. "Treatment is compromised when irritability interferes with social interactions." C. "Treatment is compromised when clients have no insight into their problems." D. "Treatment is compromised when clients choose not to take their medications."

ANS: D The nursing student should understand that the most critical challenge in the care of clients diagnosed with bipolar disorder is that treatment is often compromised when clients choose to not take their medications. Symptoms of bipolar disorder will reemerge if medication is stopped. KEY: Cognitive Level: Application | Integrated Processes: Nursing Process: Evaluation | Client Need: Physiological Integrity: Pharmacological and Parenteral Therapies

A client who is in a severely abusive relationship is admitted to a psychiatric inpatient unit. The client fears for her life. A staff nurse asks, "Why doesn't she just leave him?" Which is the nursing supervisor's most appropriate reply? A. "These clients don't know life any other way, and change is not an option until they have improved insight." B. "These clients have limited skills and few vocational abilities to be able to make it on their own." C. "These clients often have a lack of financial independence to support themselves and their children, and most have religious beliefs prohibiting divorce and separation." D. "These clients are paralyzed into inaction by a combination of physical threats and a sense of powerlessness."

ANS: D The nursing supervisor is accurate when stating that clients in severely abusive relationships are paralyzed into inaction by a combination of physical threats and a sense of powerlessness. Women often choose to stay with an abusive partner for some of the following reasons: for the children, financial reasons, fear of retaliation, lack of a support network, religious reasons, and/or hopelessness. KEY: Cognitive Level: Application | Integrated Processes: Nursing Process: Implementation | Client Need: Psychosocial Integrity

19. Which role would be most appropriate for an undergraduate-level prepared nurse working with a mental health population? a. Prescribe medications and have hospital admission privileges. b. Work as a case manager for large groups of persons with mental illness. c. Assess clients in acute psychiatric hospital settings. d. Provide basic primary, secondary, and tertiary services.

ANS: D The undergraduate levelprepared nurse is prepared to implement primary, secondary, and tertiary services. Roles of the nurse may include clinician, educator, and coordinator.

23. The nurse plans to confront a client about secondary gains related to extreme dependency on her spouse. Which nursing statement would be most appropriate? A. "Do you believe dependency issues have been a lifelong concern for you?" B. "Have you noticed any anxiety during times when your husband makes decisions?" C. "What do you know about individuals who depend on others for direction?" D. "How have the specifics of your relationship with your spouse benefited you?"

ANS: D When a client goes to excessive lengths to obtain nurturance and support from others, the client is seeking secondary gains. Secondary gains provide clients the support and attention that they might not otherwise receive.

What personality disorder is described as a pervasive pattern of social inhibition, feelings of inadequacy, and hypersensitivity to negative evaluation, beginning in early adulthood and present in a variety of contexts?

Avoidant personality disorder

Janet has a diagnosis of generalized anxiety disorder. Her physician has prescribed buspirone 15 mg daily. Janet says to her nurse, "Why do I have to take this every day? My friend's doctor ordered Xanax for her, and she only takes it when she's feeling anxious." Which of the following would be an appropriate response by the nurse? A. "Xanax is not effective for generalized anxiety disorder." B. "Buspirone must be taken daily to be effective." C. "I will ask the doctor if he will change your dose of buspirone to prn so that you don't have to take it every day." D. "Your friend really should be taking the Xanax every day."

B. "Buspirone must be taken daily to be effective."

A new psychiatric technician mentions to the nurse, "Depression seems to be a disease of old people. All the depressed clients on the unit are older than 60 years." The reply by the nurse that clarifies the prevalence of this disease is A. "That is a good observation. Depression does mostly strike people older than 50 years." B. "Depression is seen in people of all ages, from childhood to old age." C. "Depression is most often seen among the middle adult age group." D. "The age of onset for most depressive episodes is given as 18 years."

B. "Depression is seen in people of all ages, from childhood to old age." Depression can occur at any age. Children, adolescents, adults, and the elderly may all experience depression.

The mental health nurse recognizes the new nurse requires more teaching when she makes this statement about panic disorder: A. " The panic attacks are manifested by intense apprehension, fear or terror, often associated with feelings of impending doom and accompanied by intense physical discomfort." B. "Episodes of panic attacks associated with panic disorder are predictable and often occur on exposure to an anxiety producing situation." C. "Some common symptoms of panic disorder are: palpitations, pounding heart, sweating and sensations of shortness of breath." D. "The average onset of panic disorder is in the late 20s."

B. "Episodes of panic attacks associated with panic disorder are predictable and often occur on exposure to an anxiety producing situation." Panic disorder is characterized by recurrent panic attacks, the onset of which is UNPREDICTABLE. The symptoms come on unexpectedly, not before or on exposure to a situation that usually causes anxiety. pg. 532

An statement that would show acceptance of a depressed, mute client would be A. "I will be spending time with you each day to try to improve your mood." B. "I would like to sit with you for 15 minutes now and again this afternoon." C. "Each day we will spend time together to talk about things that are bothering you." D. "It is important for you to share your thoughts with someone who can help you evaluate your thinking."

B. "I would like to sit with you for 15 minutes now and again this afternoon." Spending time with the client without making demands is a good way to show acceptance.

Which nursing diagnosis would be least useful for a depressed client who shows psychomotor retardation? A. Constipation B. Death anxiety C. Activity intolerance D. Self-care deficit: bathing/hygiene

B. Death anxiety A client with psychomotor retardation has vegetative signs of depression and is often constipated, too tired to engage in activities, and lacks the energy to attend to personal hygiene. Depressed clients usually do not have death anxiety. They are more likely to welcome the idea of dying.

What statement about the comorbidity of depression is accurate? A. Depression most often exists in an individual as a single entity. B. Depression is commonly seen in individuals with medical disorders. C. Substance abuse and depression are seldom seen as comorbid disorders. D. Depression may coexist with other disorders but is rarely seen with schizophrenia.

B. Depression is commonly seen in individuals with medical disorders. Depression commonly accompanies medical disorders. The other options are false statements.

Sasha has been having angry outbursts with staff and peers on the unit. You are talking with Sasha on her third day of admission. You ask whether she is having any thoughts of suicide. Sasha becomes angry and defensive, shouting, "I'm sick of you people! Are you ever do is ask me the same question over and over. Get out of here!" Your response is based on the knowledge that: A. Sasha is getting better because she is able to be assertive. B. Sasha may be at high risk for self-harm. C. Sasha is probably experiencing transference. D. Sasha may be angry at someone else and projecting that anger to staff.

B. Sasha may be at high risk for self-harm. Overt hostility is highly correlated with suicide; therefore the patient may be considered high risk, and appropriate precautions should be taken. The other responses are incorrect with no evidence to support them.

The initial care plan for a client with OCD who washes her hands obsessively would include which of the following nursing interventions? A. Keep the ct's bathroom locked so she can't wash her hands all the time. B. Structure the ct's schedule so that she has plenty of time for washing her hands. C. Place the ct in isolation until she promises to stop washing her hands so much. D. Explain the ct's behavior to her, since she's probably unaware that it's maladaptive.

B. Structure the ct's schedule so that she has plenty of time for washing her hands.

A client prescribed a monamine oxidase inhibitor (MOA) has a pass to go out to lunch. Given a choice of the following entrees, the client can safely eat A. avocado salad plate. B. fruit and cottage cheese plate. C. kielbasa and sauerkraut. D. liver and onion sandwich.

B. fruit and cottage cheese plate. Fruit and cottage cheese do not contain tyramine. Avocados, fermented food such as sauerkraut, processed meat, and organ meat contain tyramine. Monoamine oxidase inhibitors inhibit the breakdown of tyramine, which can lead to high blood pressure, a hypertensive crisis, and eventually a cerebrovascular accident.

What is a biochemical abnormality associated with panic disorder?

Blood elevations of lactate. Pg 533

Psychological assessment of a pt with anorexia should include what

Body distortion, fear of weight gain, unrealistic expectations or thinking, ritualistic behaviors, difficulty expressing neg feelings, inability to experience visceral cues/emotions

Physical assessment for a pt w/ anorexia should include what?

Body system eval, weight, BMI, menstrual hx,

Sasha is a 38-year-old patient admitted with major depression. Which of the following statements Sasha makes alerts you to a common accompaniment to depression? A."I still pray and read my Bible every day." B. "My mother wants to move in with me, but I want to independent." C. "I still feel bad about my sister dying of cancer. I should have done more for her!" D. "I've heard others say that depression is a sign of weakness."

C. "I still feel bad about my sister dying of cancer. I should have done more for her!" Guilt is a common accompaniment to depression. A person may ruminate over present or past failings. Praying and reading the Bible describes a coping mechanism; the other responses do not describe a common accompaniment to depression.

Sasha is started on fluoxetine. Which statement by Sasha indicates that she understands the medication teaching you have provided? A. "I will make sure to get plenty of sunshine and not use sunscreen to avoid a skin reaction." B. "I will not take any over-the-counter medication while on the fluoxetine." C. "I will report any symptoms of high fever, fast heartbeat, or abdominal pain to my provider right away." D. "I will report increased thirst and urination to my provider."

C. "I will report any symptoms of high fever, fast heartbeat, or abdominal pain to my provider right away." This describes symptoms of serotonin syndrome, a life-threatening complication of SRRI medication. The other options are incorrect because the patient should be wearing sunscreen to avoid sunburn, may take over-the-counter medications if sanctioned by the provider, and would not have been educated to report increased thirst and urination as a side effect of fluoxetine.

Joanie is a new pt at the mental health clinic. She has been diagnosed with body dysmorphic disorder. Which of the following medication is the psychiatric nurse practitioner most likely to prescribe for Joanie? A. Alprazolam (Xanax) B. Diazepam (Valium) C. Fluoxetine (Prozac) D. Olanzapine (Zyprexa)

C. Fluoxetine (Prozac)

Select the nursing diagnosis least likely to be chosen after analysis of data pertinent to a client with post-partum depression. A. Impaired parenting B. Ineffective role performance C. Health-seeking behaviors D. Risk for impaired parent/infant/child attachment

C. Health-seeking behaviors A client with severe depression of any etiology will not have the mental or physical energy to engage in health-seeking behaviors. Further, her negative view of self and the world would preclude such thinking.

A client with OCD says to the nurse, "I've been here 4 days now, and I'm feeling better. I feel comfortable on this unit, and I'm not at ill-at-ease with the staff or other pts anymore." In light of this change, which nursing intervention is most appropriate? A. Give attention the to ritualistic behaviors each time they occur and point out their inappropriateness. B. Ignore the ritualistic behaviors, and they will be eliminated for lack of reinforcement. C. Set limits on the amount of time Sandy may engage in the ritualistic behavior. D. Continue to allow Sandy all the time she wants to carry out the ritualistic behavior.

C. Set limits on the amount of time Sandy may engage in the ritualistic behavior.

Some obese individuals take amphetamines to suppress appetite and help them lose weight. Which of the following is an adverse effect associated with use of amphetamines that makes this practice undesirable? a. bradycardia b. amenorrhea c. tolerance d. convulsions

C. Tolerance

A depressed client tells the nurse, "There is no sense in trying. I am never able to do anything right!" The nurse can identify this cognitive distortion as an example of A. self-blame. B. catatonia. C. learned helplessness. D. discounting positive attributes.

C. learned helplessness. Learned helplessness results in depression when the client feels no control over the outcome of a situation.

Beck's cognitive theory suggests that the etiology of depression is related to A. sleep abnormalities. B. serotonin circuit dysfunction. C. negative processing of information. D. a belief that one has no control over outcomes.

C. negative processing of information. Beck is a cognitive theorist who developed the theory of the cognitive triad of three automatic thoughts responsible for people becoming depressed: (1) a negative, self-deprecating view of oneself; (2) a pessimistic view of the world; and (3) the belief that negative reinforcement will continue.

When the clinician mentions that a client has anhedonia, the nurse can expect that the client A. has poor retention of recent events. B. experienced a weight loss from anorexia. C. obtains no pleasure from previously enjoyed activities. D. has difficulty with tasks requiring fine motor skills.

C. obtains no pleasure from previously enjoyed activities. Anhedonia is the term for the lack of ability to experience pleasure.

A depressed client is noted to pace most of the time, pull at her clothes, and wring her hands. These behaviors are consistent with A. senile dementia. B. hypertensive crisis. C. psychomotor agitation. D. central serotonin syndrome.

C. psychomotor agitation. These behaviors describe the psychomotor agitation sometimes seen in clients with the agitated type of depression.

Assessment of the thought processes of a client diagnosed with depression is most likely to reveal A. good memory and concentration. B. delusions of persecution. C. self-deprecatory ideation. D. sexual preoccupation.

C. self-deprecatory ideation. Depressed clients never feel good about themselves. They have a negative, self-deprecating view of the world.

It is likely that a client diagnosed with seasonal affective disorder will begin to experience fewer symptoms in the A. fall. B. winter. C. spring. D. summer.

C. spring. Seasonal affective disorder occurs during the months when sunlight diminishes. Clients may begin to feel effects in the late fall and will be affected throughout the winter. They improve during the spring and feel well during the summer.

Interventions for the psychosocial domain of bulimia

CBT and IPT. Behavioral techniques such as cue elimination and self monitoring. Psychoeducation including boundary/limit setting, assertiveness, nutrition/healthy eating, and clarification of misconceptions about food. Group therapy.

What are some behaviors of someone with paranoid personality disorder?

Constantly on guard; hypervigilant; ready for any real/imagined threat; tense and irritable; hard exterior; insensitive to feelings of others; avoid interactions; feel others want to take advantage of them; oversensitive; misinterpret environmental cues; constantly testing honesty of others; attribute their shortcomings to others; intense desire for reprisal & vindication.

Biochemical influences

*Deficiency* of norepinephrine and dopamine = depression *Excess* of norepinephrine and dopamine = mania

Other nursing diagnoses

*Disturbed thought processes related to:* Biochemical alterations in the brain, evidenced by delusions of grandeur and persecution and inaccurate interpretation of the environment *Disturbed sensory perception related to:* Biochemical alterations in the brain and to possible sleep deprivation, evidenced by auditory and visual hallucinations *Impaired Social Interaction related to:* Egocentric and narcissistic behavior *Insomnia related to:* Excessive hyperactivity and agitation

Symptoms in children/adolescents

*Euphoric/expansive mood:* extremely happy, silly, or giddy *Irritable mood:* hostility and rage, often over trivial matters *Grandiosity:* Believes abilities to be better than everyone else's. *Decreased need for sleep:* May sleep for only 4 or 5 hours per night and wake up feeling rested. *Pressured speech:* loud, intrusive, difficult to interrupt *Racing thoughts:* Rapid change of topics. *Distractibility:* Unable to focus on school lessons. *Increase in goal-directed activity/psychomotor agitation:* Activities become obsessive. Increased psychomotor agitation. *Excessive involvement in pleasurable or risky activities:* Exhibits behavior that has an erotic, pleasure-seeking quality about it. *Psychosis:* May experience hallucinations and delusions. *Suicidality:* May exhibit suicidal behavior during a depressed or mixed episode or when psychotic.

Medications for Bipolar

*For mania* 1) Lithium carbonate 2) Anticonvulsants 3) Verapamil 4) Antipsychotics *For depressive pahse* Use antidepressants with care (may trigger mania).

types of abuse

- intimate partner abuse - child abuse - sexual assault

Antipsychotic Side effects

-Drowsiness, dizziness -Dry mouth, constipation -Increased appetite, weight gain -ECG changes -Extrapyramidal symptoms -Hyperglycemia and diabetes

Side effects of Lithium

-Drowsiness, dizziness, headache -Dry mouth, thirst, GI upset, nausea/vomiting -Fine hand tremors -Hypotension, arrhythmias, pulse irregularities -Polyuria, dehydration -Weight gain -Potential for toxicity

Anticonvulsant Side effects

-Nausea and vomiting -Drowsiness, dizziness -Blood dyscrasias -Prolonged bleeding time (with valproic acid) -Risk of severe rash (with lamotrigine) -Decreased efficacy of oral contraceptives (with topiramate) -Risk of suicide with all antiepileptic drugs

what are predisposing biological theories to abusive bahaviors?

-neurophysiological influences -biochemical influences -genetic influences -disorders of the brain

what are predisposing psychological theories to abusive behaviors?

-psychodynamic theory -learning theory

A DSM-5 diagnosis of schizoid personality disorder requires 4 or more of 7 criteria. What are they?

1. Neither desires nor enjoys close relationships, including being part of a family 2. Almost always chooses solitary activities 3. Has little, if any, interest in having sexual experiences with anther person 4. Take pleasure in few, if any, activities 5. Lacks close friends or confidants other than close family 6. Appears indifferent to praise or criticism of others 7.Shows emotional coldness, detachment, or flattened affectivity

DSM-5 criteria for histrionic personality disorder requires 5 or more of 8 traits. What are they?

1. Is uncomfortable in situations in which he or she is not the center of attention 2. Interaction with others is often characterized by inappropriate sexually seductive behavior 3. Displays rapidly shifting and shallow expression of emotions 4. consistently uses physical appearance to draw attention to self 5. Has a style of speech that is excessively impressionistic and lacking in detail 6. Shows self-dramatization, theatricality, and exaggerated expression of emotion 7. Is suggestible 8. Considers relationships to be more intimate that they actually are

What are the 5 functions of dialectictal behavior therapy:

1. To enhance behavioral capabilities 2. To improve motivation to change 3. To ensure that new capabilities generalize to the natural environment 4. To structure the treatment environment such that client and therapist capabilities are supported and effective behaviors are reinforced 5. To enhance therapist capabilities and motivation to treat clients effectively

A schizotypal personality disorder requires 5 or more of what 9 criteria?

1. ideas of reference 2. odd beliefs or magical thinking 3. Unusual perceptual experiences 4. odd thinking & speech 5. suspiciousness or paranoid ideation 6. inappropriate or constricted affect 7. behavior or appearance that is odd, eccentric, or peculiar 8. lack of close friends 9. excessive social anxiety

What are the three cognitive distortions that Beck believes serve as the basis for depression?

1. negative expectation of the environment 2. negative expectation of the self 3. negative expectation of the future

What is an acceptable BMI?

19-25

A patient arrives in the emergency department with impaired consciousness, nystagmus, and seizures. It is determined that he is suffering from lithium toxicity. With these symptoms, his lithium level would be expected to be above___________mEq/L.

3.5

What is splitting?

A primitive ego defense mechanism that is common people with borderline personality disorder. An inability to integrate and accept both positive and negative feelings. Something is all good or all bad.

What is the 5 year recovery rate for anorexia?

69%

9. According to the National Institutes of Mental Health (NIMH), which is the leading cause of disability among adults? a. Schizophrenia b. Major depression c. Obsessive-compulsive disorder d. Anxiety

ANS: B Major depression is pervasive and is the leading cause of disability among adults ages 15 to 44.

11. A nurse is working with a client who is concerned about discrimination in the workplace based on a current mental health diagnosis. Which legislation has prohibited discrimination for persons with mental disorders? a. National Mental Health Act b. American with Disabilities Act c. Developmental Disabilities Act d. Protection and Advocacy for Mentally Ill Individuals Act

ANS: B The Americans with Disabilities Act prohibited discrimination and promoted opportunities for persons with mental disorders.

20. A nurse is counseling a client following an unexpected loss. If given adequate support and adaptation, the client will most likely: a. Recover from the crisis and become mentally ill b. Avoid the loss and potential mental illness c. Be able to ignore the grief d. Resume previous lifestyle in spite of sadness

ANS: D When people do not have adequate resources, there is an increased risk of altered mental health. However, when given adequate support and adaptation, most persons will resume their lifestyles.

What are the medical/psychiatric parameters for someone w/ an eating d/o to be hospitalized?

Acute weight loss (<85% below ideal), HR near 40 bpm, temp <36.1, BP<80/50, hypokalemia, hypophophatemia, hypomagesemia, poor motivation to recover, risk for suicide, depression, failure to comply w/ tx

School intervention therapies for anorexia

Address the shame/guilt about having an eating d/o w/ the pt before returning to school. Involve school nurses/teachers to help in the reentry process and making new friends

Interventions for the psychological domain in anorexia

Addressing the interoceptive awareness (journaling), understanding their feelings, restructuring distortions, movement/dance therapy, imagery, relaxation, interpersonal therapy, and pt/family education

Bipolar Mania

Alteration of mood expressed by elation, self esteem, grandiosity, hyperactivity, agitation, accelerated thinking and speaking Causes cab be biological, psychological, substance induced, medical condition

In bipolar disorder, recovery is a continuous process.

Although there is no cure for bipolar disorder, recovery is possible in the sense of learning to prevent and minimize symptoms, and to successfully cope with the effects of the illness on mood, career, and social life.

What is a drive for thinness seen in anorexia nervosa?

An intense physical and emotional process that overrides all physiologic body cues such as hunger and weakness and focuses all efforts on controlling food intake

Pharmacologic therapy for bulimia

Antidepressants-->SSRIs

What personality disorder is more common among the lower socioeconomic classes, particularly so among highly mobile inhabitants of impoverished urban areas?

Antisocial personality disorder

Assessment of the psychosocial domain for bulimia

Assess for cognitive distortions, knowledge deficits, body dissatisfaction, and mood

Priority care issues w/ bulimia

Associated w/ a high risk of suicide. Risk for self mutilation. Display high levels of impulsivity, shoplifting, and overspending leading to financial and legal troubles

The major reason for hospitalization for depressed patients is: A. inability to go to work. B. suicidal ideation. C. loss of appetite. D. psychomotor agitation.

B. suicidal ideation. Suicidal thoughts are a major reason for hospitalization for patients with major depression. It is imperative to intervene with such patients to keep them safe from self-harm. The other options describe symptoms of major depression but aren't by themselves the major reason for hospitalization.

The nursing diagnosis Imbalanced nutrition: less than body requirements has been identified for a client diagnosed with severe depression. The most reliable evaluation of outcomes will be based on the client's A. energy level. B. weekly weights. C. observed eating patterns. D. statement of appetite.

B. weekly weights. The client's body weight is the most reliable and objective evaluation of success in treating this nursing diagnosis.

Why are intensive therapies for anorexia usually implemented for anorexia after refeeding?

Because concentration is usually impaired in those who are severely undernourished

The DSM-5 diagnostic system classifies personality disorders into 3 clusters depending on personality traits. What is Cluster C comprised of?

Behaviors described as anxious or fearful a. avoidant personality disorder b. dependent personality disorder c. obsessive-compulsive personality disorder

The DSM-5 diagnostic system classifies personality disorders into 3 clusters depending on personality traits. What is Cluster B comprised of?

Behaviors described as dramatic, emotional or erratic. a. antisocial personality disorder b. borderline personality disorder c. histrionic personality disorder d. narcissistic personality disorder

What is a binge eating d/o?

Binge eating similar to those w/ bulimia but w/o the purging and they don't compensate for binges thru other behaviors. Most are obese. Lower dietary restraint.

Criteria for binge eating d/o

Binge eating, distress about the binge, eating until uncomfortably full, feelings of guilt/depression after

Biologic theory for anorexia

Brain structure in the medial orbitofrontal cortex and striatum is altered in eating d/o's. Pleasantness in taste and sensitivity to reward in pt's w/ anorexia are also associated w/ alterations in brain structures.

How does bulimia nervosa r/t anorexia nervosa?

Bulimia used to be thought to be a type of anorexia but now it is seen as a distinct d/o. Bulimia is more prevalent than anorexia and is generally not as life threatening.

Evaluation is measured how?

By whether outcome criterion has been fulfilled (card #26)

From a biological theory perspective, which of the following predisposes individuals to be abusive? A- Unmet needs for security resulting in an underdeveloped ego and a weak super ego B- Imitation of individuals who have a predisposition toward aggressive behavior C- Various levels of norepinephrine, dopamine, and serotonin D- The influence of culture and social structure

C

Which of the following is the most appropriate therapy for a client with agoraphobia? A. 10 mg Valium qid B. Group therapy with other agoraphobics C. Facing her fear in gradual step progression D. Hypnosis

C. Facing her fear in gradual step progression

Clinical course of anorexia

Chronic condition w/ relapses that are characterized by significant weight loss

lithium toxicity reasoning

Clients taking lithium should consume a diet adequate in sodium and drink 2,500 to 3,000 mL of fluid per day. Lithium is a salt and competes in the body with sodium. If sodium is lost, the body will retain lithium, resulting in toxicity. Conversely, if sodium levels are significantly increased, it will reduce the level of lithium, resulting in decreased efficacy. Maintaining normal sodium and fluid levels is critical to maintaining therapeutic levels of lithium.

The DSM-5 diagnostic system classifies personality disorders into 3 clusters depending on personality traits. What is Cluster A comprised of?

Cluster A: behaviors described as odd or eccentric a. paranoid personality disorder b. schizoid personality disorder c. schizotypal personality disorder

Ms. T has been diagnosed with agoraphobia. Which behavior would be most characteristic of this disorder? A. Ms. T. experiences panic anxiety when she encounters snakes. B. Ms. T refuses to fly in an airplane. C. Ms. T. Will not eat in public places. D. Ms. T. stays in her home for fear of being in a place from which she cannot escape.

D. Ms. T. stays in her home for fear of being in a place from which she cannot escape.

What is anaclitic depression?

Depression caused by prolonged separation from the mother in the first year of life.

FIND

Diagnosis of bipolar in children/adolescents *Frequency:* symptoms occur most days in a week *Intensity:* symptoms are severe enough to cause extreme disturbance *Number:* symptoms occur 3-4x a day *Duration:* symptoms occur 4 or more hours a day

Biologic theory for bulimia

Dieting is believed to affect serotonergic regulation and binging affects the dopamine, ACh, and opioid reward-related systems but these changes are the result of eating dysregulation rather than the cause

What are some of the characteristics a person with anorexia may portray?

Drive for thinness, body image distortion, interoceptive awareness, perfectionism, and guilt/anger

Side effects of Verapamil

Drowsiness, dizziness Hypotension, bradycardia Nausea Constipation

When does the onset of anorexia nervosa usually occur?

Early adolescence

Prevention of bulimia

Effort on the part of teachers, school nurses, parents, & society as a whole. Education of elementary school nurses/teachers. Emphasis on protective factors that mediate b/w risk factors and the development of an eating d/o

Family factors theory for bulimia

Families are reported to be chaotic, with few rules and unclear boundaries. Often an overly close or enmeshed relationship b/w mother and daughter. Daughter's may r/t mother as best friend. Daughter feel guilty about separation & responsible for mother's happiness or emotional well-being.

Social assessment for anorexia includes...

Family interaction/influence, peer relationships, social contacts

What is the difference between fear and anxiety?

Fear involves cognition-the intellectual appraisal of a threatening stimulus while anxiety is the emotional response to that stimulus.

Characteristic of bulimia

Few outward signs. Individuals binge (eating an excessive amount, usually in one sitting) and purge (purposeful initiation of stomach or bowel evac thru vomiting or laxatives) in secret. Usually of normal weight. Overwhelmed & overly committed individuals. "social butterflies" who have difficulty setting limits and boundaries. Feel shame/guilt/disgust about binging/purging. Impulsivity in other aspects of life.

Genetic theory for anorexia

First degree relatives of people with anorexia have higher rates. Rates of partial-syndrome or subthreshold cases among female family members of those w/ anorexia are even higher. Genetic research shows a genetic vulnerability to anorexia esp in females. Genetic heritability accounts for 50-80% of the risk for developing an eating d/o

Psychoanalytic paradigm psychologic theory for anorexia

Focuses on conflicts of separation (individuation and autonomy). Thought to occur as a result of developmental arrest of normal adolescent struggles around identity and role, body image formation, & sexuality. Dieting/weight control viewed as a means to defend against these feelings of inadequacy and growing into adulthood.

What is interpersonal therapy for anorexia tx?

Focuses on uncovering and resolving the developmental and psychological issues underlying the d/o. Role transitions & negative social evaluations are the focus.

Nursing interventions aimed at

Fulfilling outcome criterion (card #26)

Child and Adolescent Bipolar Foundation (CABF)

Guidelines for diagnosis and treatment have been developed by CABF Recommends the use of FIND in making a diagnosis of bipolar disorder in children and adolescents

Protective factors against anorexia

Healthy eating attitudes, acceptance of body size, positive self evaluation, healthy parental relationships

Where does tx for bulimia usually take place?

In an outpt setting unless the pt is suicidal or when past outpt tx has failed

Genetic and familial predispositions for bulimia

In several twin studies, genetic influences have shown to outweigh environmental ones

When should SSRIs be taken?

In the morning b/c they can cause insomnia

Common symptoms of depression in adolescents

Inappropriately expressed anger, aggressiveness, running away, deliquency, social withdrawal, sexual acting out, apathy, loss of self-esteem, sleeping & eating disturbances and pyschosomatic complaints.

Risk factors for anorexia

Increased BMR, over exercising, low self esteem, body dissatisfaction, feelings of ineffectiveness. If the family is overprotective, enmeshed (extreme form of intensity in family interactions & low individual autonomy), has rigid boundaries, and inability to solve conflicts. Athletes involved in elite, lean sports are at higher risk.

The neurotransmitter most strongly associated with panic disorder is:

Increased levels of Norepinephrine. It's known to mediate arousal, and it causes hyperarousal and anxiety. Seratonin and GABA are believed to be decreased in panic disorder as well.

What is dialectical behavior therapy and what personality disorder was it developed for?

It was developed for the chronic self-injurious and parasuicidal behavior of clients with borderline personality disorder. It is a complex, eclectic treatment that combines concepts of cognitive, behavioral, and interpersonal therapies with eastern mindfulness.

The Recovery Model

Learning how to live a safe, dignified, full, and self-determined life in the face of the enduring disability which may, at times, be associated with serious mental illness.

Epidemiology of bulimia

Lifetime prevalence of 2% (less than 1/3 of cases have been detected). Onset b/w 15 and 24 y/o. Females to males 10:1. Higher rates in hispanics and white women. Comorbid conditions including substance abuse, depression, and OCD

What medications have been useful for preventing violent episodes in clients with antisocial personality disorder?

Lithium carbonate and propanolol. However, caution must be used when prescribing medications outside the structured setting because of the high risk for substance abuse in these individuals.

Bipolar I Disorder

Manic episodes with depressive episodes

What is one risk factor measurement instrument for eating d/o's?

McKnight risk factor survey

Hypomania

Milder, does not impair social/occupational functioning, no hospitalizations or psychotic features.

Clients with avoidance personality disorder and antisocial therapy disorder may find what therapy especially useful?

Milieu or group therapy helps those with antisocial personality disorder by peer feedback. It helps with social anxiety and developing interpersonal trust for those with avoidant personality disorder.

Feeling better in the morning and worse as the day goes on is seen in _______ related to the diurnal variation of neurotransmitter activity.

Moderate depression

What neurotransmitters are hypothesized to be related to depression?

Norepinepherine, dopamine, serotonin and acetylcholine

What does tx for anorexia focus on?

Nutritional rehab, conflict resolution r/t body image, effective coping, addressing underlying conflicts r/t maturity fears and role conflict, and assisting family with healthy functioning and communication

Cognitive-behavioral therapy is especially useful for those with:

Obsessive compulsive, antisocial, and avoidant personality disorders

Body dissatisfaction theory for anorexia

Occurs when the body becomes overvalued as a way of determining one's worth. Strongly r/t self-esteem. Pt compares body with others esp celebrities. Attempt to overcome dissatisfaction w/ dieting and exercise

What is intensive therapy for anorexia?

Outpatient programs that help pt's with their underlying issues (such as maturity fears and body distortion) and to help families w/ communication and enmeshment.

Psychosocial origin of people with schizoid type personality disorder includes what kind of parenting?

Parents that were cold, neglectful, lacking empathy, and did not have gratifying relationships with their children.

Psychosocial origin of people with paranoid personality disorder includes what kind of parenting?

Parents were antagonizing and harassing. Likely used their kids as scapegoats for displaced aggression.

Patients with borderline personality disorder (BPD) exhibit negative effect, which includes emotional _____________, described as rapidly moving from one emotional extreme to another.

Patients with borderline personality disorder (BPD) exhibit negative effect, which includes emotional ___lability__________, described as rapidly moving from one emotional extreme to another. One of pathological personality traits seen in persons with BPD is negative effect, which is characterized by emotional lability, that is, rapidly shifting emotions from one extreme to another. Patients exhibiting this trait are often documented as being "labile."

Social theory for development of anorexia

People interpret expectations of how they should look from societal influences, media, fashion industry, peer pressure. Preoccupation w/ body image & weight also influenced by obesity epidemic. Feminism focused on the role that the historical image of women has played in the development of eating d/o's

What is perfectionism?

Personal standards and concern over mistakes and their consequences for their self-worth and others' opinions. This is a significant personality risk factor in eating d/o's. It precedes the development of weight and shape concerns. The more severe the d/o, the more perfectionistic.

What is the difference between a personality trait and a personality disorder?

Personality traits influence the way a person acts, perceives their environment, are stable over time, and are generally developed early in life. Personality disorders are what happens when personality traits become rigid and inflexible, leading to maladaptive patters of behavior that affect functioning.

What all does tx for bulimia include?

Pharmacologic therapy, cognitive behavioral therapy (CBT), interpersonal therapy (IPT), nutrition counseling, group psychotherapy, & support groups

How can schools help decrease the prevalence of eating d/o's?

Prevention & early detection strategies. Monitor the media to remove unhealthy advertisements/articles. Change school curriculum to involve ED's. Implement anti-bullying policies.

Guilt and anger seen in those with anorexia

Pt tend to avoid conflict and have difficulty expressing negative emotions esp anger. Overwhelming sense of guilt/anger leading to conflict avoidance

Peer pressure internalization theory for anorexia

Pt's report that dieting, binge eating, and purging initially began as a result of peer pressure and a need to conform.

What is binge eating?

Rapid, episodic, impulsive, uncontrollable ingestion of a large amount of food during a short time, usually 1-2 hours

Diagnostic criteria for bulimia

Recurrent episodes of binge eating and compensatory behaviors in the form of purging (vomiting/laxatives/diuretics/emetics) or nonpurging (fasting/overexercising). Episodes occur at least once a week for 3 months. No severe weight loss or amenorrhea as w/ anorexia.

Interventions for the physical symptoms of anorexia include

Refeeding, sleep promotion, and fluoxetine

What medications are successful in decreasing impulsivity and self-destructive acts in borderline personality disorder?

SSRIs, MAOIs, atypical antipsychotics. However, MAOIs are not used commonly b/c of concerns about dietary restriction violations and the higher risk of fatality w/ overdose. SSRIs in combo with atypical antipsychotics have been successful in treating dysphoria, mood instability, and impulsivity.

Pharmacologic management of anorexia includes what?

SSRIs-->fluoxetine (Prozac)

what medications are used to modulate aggression?

SSRIs: decrease irrit. and aggressive bx (r/t aggressive bx d/t low serotonin) mood stabilizers: dampen limbic irrit. ie: carbamazapine phenytoin divalproex lithium antiadrenergic agents: beta blockers (propanolol) dampens excesive noradrenergic activity antipsychotics (typical and atypical) for aggressive bx r/t excess dopamine

Depressive disorders that occur as a consequence of a non-mood disorder or as an adverse effect of certain medications are called:

Secondary depression

Feeling worse in the morning and some what better as the day goes on is a physiological symptoms of _______ that is related to the diurnal variation of neurotransmitter activity.

Severe depression.

Risk factors for bulimia

Similar to those for anorexia. Society's influence, such as media and peer pressure. Comparing oneself to these ideal body types. These factors influence behaviors such as dietary restraint and over-exercising which leads to binge eating and purging for fear of becoming fat. Possible link to sexual abuse.

What factors influence personality development?

Some factors are: hereditary, temperament, experiential learning, social interaction

What do the goals for the tx of bulimia focus on?

Stabilizing and then normalizing eating (stopping the binge-purge cycle), restructuring dysfunctional thought patterns/attitudes esp about eating, weight, & shape, teaching healthy boundary setting, and resolving conflicts about separation-individuation.

Interventions for the biologic domain for bulimia

Strict monitoring of food intake, supervision of bathroom visits, sleep management, SSRIs, monitor the administration of meds

Criteria for meeting outcomes

The Client: 1) Exhibits no evidence of physical injury 2) Has not harmed self or others 3) Is no longer exhibiting signs of physical agitation 4) Eats a well-balanced diet with snacks to prevent weight loss and maintain nutritional status 5) Verbalizes an accurate interpretation of the environment 6) Verbalizes that hallucinatory activity has ceased and demonstrates no outward behavior indicating hallucinations 7) Accepts responsibility for own behaviors 8) Does not manipulate others for gratification of own needs 9) Interacts appropriately with others 10) Is able to fall asleep within 30 minutes of retiring 11) Is able to sleep 6 to 8 hours per night

Survivors of Abuse: Background Assessment Data: The Victim: What sx's are common for long term effects after the rape?

increased restlessness dreams and nightmares phobias (esp. those /c sexual undertones / sexual interaction)

Diagnostic criteria for anorexia

There is a restriction of intake leading to a significantly low body weight. BMI used as a measure of severity. Intense fear of gaining weight or becoming fat, body image issues including an undue influence of body weight and lack of recognition of seriousness of low body weight. Absence of 3 or more menstrual cycles. Specify the type.

List diagnostic criteria for antisocial personality disorder (7)

This disorder is a pervasive pattern of disregard for and violation of the rights of others occurring since age 15, as indicated by 3 or more of the following: a. failure to conform to social normal with respect to lawful behaviors by repeating acts that are grounds for arrest b. decieftfulness as indicated by repeated lying, use of aliases, conning others for personal pleasure or profit c. impulsivity d. reckless disregard for safety of self or others e. consistent irresponsibility f. lack of remorse *Must be at least 18 with evidence of conduct disorder before onset of 15 years. Behaviors must not occur exclusively during schizophrenic or bipolar disorder

Biochemical factors theory for bulimia

Those w/ bulimia are thought to have altered modulation of central serotonin neuronal systems so lowered brain serotonin levels. Depletion of tryptophan has also been shown to lead to a depressed mood, a desire to binge, and an increase in weight/shape concerns

What is the refeeding protocol for anorexia?

Typically starts w/ 1500 calories/day and is increased slowly until the pt is consuming at least 3500 calories/day in several meals. The higher the initial calories, the shorter the hospital stay and recovery. Want pts w/ very low weight to gain 1-2lbs per week.

When does family therapy for anorexia begin?

Usually while the pt is still in the hospital

Cognitive theory for bulimia

Views bulimia as a d/o of thinking in that distortions are the basis for behaviors like binging and purging. Psychological triggering mechanism models say that cues like stress, neg emotions, & environmental cues pay a role.

What is body image distortion?

When the individual perceives his or her body disparately from how the world or society views it

Which statement is least likely to be made by a client diagnosed with bulimia nervosa during the assessment interview? a. "I eat three meals each day and purge every evening." b. "I'm concerned about what others think about my binging and purging." c. "I feel as though my eating and purging are out of my control." d. "When I eat I feel calm, but then I realize I have to make myself vomit or gain weight."

a. "I eat three meals each day and purge every evening." Most clients with bulimia purge after each meal.

The physician orders lithium carbonate 600 mg tid for a newly diagnosed client with bipolar I disorder. There is a narrow margin between the therapeutic and toxic levels of lithium. Therapeutic range for acute mania is: a. 1.0 to 1.5 mEq/L b. 10 to 15 mEq/L c. 0.5 to 1.0 mEq/L d. 5 to 10 mEq/L

a. 1.0 to 1.5 mEq/L

The binging episode is thought to involve: a. A release of tension, followed by feelings of depression b. Feelings of fear, followed by feelings of grief c. unmet dependency needs and a way to gain attention d. feelings of euphoria, excitement, and self-gratification

a. A release of tension, followed by feelings of depression

Brandon, a client on the psychiatric unit, has been diagnosed with schizophrenia. He begins to tell the nurse about how the CIA is looking for him and will kill him if they find him. Brandon's belief is an example of a: a. Delusion of persecution. b. Delusion of reference. c. Delusion of control or influence. d. Delusion of grandeur.

a. Delusion of persecution.

Recent research on the RAISE approach to treatment of schizophrenia incorporates which of the following elements as important to improving outcomes? (Select all that apply.) a. Early intervention at the first episode of psychosis b. Support for employment and/or educational pursuits c. Rapid high-dose loading with antipsychotic medication d. Court-ordered sanctions for treatment e. Recovery-focused psychotherapy

a. Early intervention at the first episode of psychosis b. Support for employment and/or educational pursuits e. Recovery-focused psychotherapy

Jack is a new client on the psychiatric unit with a diagnosis of antisocial personality disorder. Which of the following characteristics would you expect to assess in Jack? a. Lack of guilt for wrongdoing b. Insight into his own behavior c. Ability to learn fro past experiences d. Compliance with authority

a. Lack of guilt for wrongdoing

What is the treatment of choice for histrionic personality disorder?

psychoanalytical psychotherapy

Although historically lithium has been the medication of choice for mania, several others have been used with good results. Which of the following are used in the treatment of bipolar disorder? (Select all that apply.) a. Olanzapine (Zyprexa) b. Oxycodone (OxyCotin) c. Carbamazepine (Tegretol) d. Gabapentin (Neuronrin) e. Tranylcypromine (Parnate)

a. Olanzapine (Zyprexa) c. Carbamazepine (Tegretol) d. Gabapentin (Neuronrin)

Which of the following behavioral patterns is characteristic of individuals with narcissistic personality disorder? a. Overly self-centered and exploitative of others b. Suspicious and mistrustful of others. c. Rule conscious and disapproving of change. d. Anxious and socially isolated

a. Overly self-centered and exploitative of others

Kim, a client diagnosed with borderline personality disorder, manipulates the staff in an effort to fulfill her own desires. All of the following may be examples of manipulative behaviors in the borderline client except: a. Refusal to stay in room alone, stating, "It's so lonely." b. Asking Nurse Jones for cigarettes after 30 minutes, knowing the assigned nurse has explained she must wait 1 hour. c. "Stating to Nurse Jones, "I really like having you for my nurse. You're the best one around here." d. Cutting arms with razor blade after discussing dismissal plans with physician.

a. Refusal to stay in room alone, stating, "It's so lonely."

A client arrested for an assault in which he savagely beat a classmate states, "The guy deserved everything he got." The behaviors described are most consistent with the clinical picture of a. antisocial personality disorder. b. borderline personality disorder. c. schizotypal personality disorder. d. narcissistic personality disorder.

a. antisocial personality disorder. Clients with antisocial personality act out feelings without consideration for the rights of others. They feel no remorse for their antisocial acts.

what is an act of aggression not passion?

rape

Characteristics the nurse will assess in the client diagnosed with antisocial personality disorder are a. deceitfulness, impulsiveness, and lack of empathy. b. perfectionism, preoccupation with detail, and verbosity. c. avoidance of interpersonal contact and preoccupation with being criticized. d. a need for others to assume responsibility for decision making and seeking nurture.

a. deceitfulness, impulsiveness, and lack of empathy. Antisocial clients have no conscience. Their sense of right and wrong is impaired, and they tend to do whatever serves them best without consideration for the rights or feelings of others.

A coping mechanism used excessively by clients with anorexia nervosa is a. denial. b. humor. c. altruism. d. projection.

a. denial. Denial of excessive thinness is the mainstay of the client with anorexia nervosa.

Jack is a new client on the psychiatric unit with a dx of antisocial personality disorder. Which of the following characteristics would you expect to assess in Jack? a. lack of guilt for wrongdoing b. insight into his own behavior c. ability to learn from past experiences d. compliance with authority

a. lack of guilt for wrongdoing

A focus for the acute phase of treatment for anorexia nervosa would be a. weight restoration. b. improving interpersonal skills. c. learning effective coping methods. d. changing family interaction patterns.

a. weight restoration. Weight restoration is the priority goal of treatment for the client with anorexia nervosa because health is seriously threatened by the underweight status.

Which assessment question should be asked of a client suspected of demonstrating characteristics of anorexia nervosa? a. "Do you find yourself feeling hungry?" b. "How would you describe your body?" c. "How often do you force yourself to vomit?" d. "Why do you choose to take laxatives?"

b. "How would you describe your body?" This question will reveal the cognitive distortion consistent with anorexia nervosa. Invariably the client will describe self as fat despite being excessively underweight.

Margaret, a 68-year-old widow experiencing a manic episode is admitted to the psychiatric unit after being brought to the emergency department by her sister-in-law. Margaret yells, "My sister-in-law is just jealous of me! She's trying to make it look like i'm insane!" This behavior is an example of: a. A delusion of grandeur b. A delusion of persecution c. A delusion of reference. d. A delusion of control of influence.

b. A delusion of persecution

Which of the following behavioral patterns is characteristic of individuals with schizotypal personality disorder? a. Belittling themselves and their abilities b. A lifelong pattern of social withdrawal c. Suspicious and mistrustful of others d. Overreacting inappropriately to minor stimuli

b. A lifelong pattern of social withdrawal

"Splitting" by the client with BPD denotes: a. Evidence of precocious development b. A primitive defense mechanism in which the client sees objects as all good or all bad. c. A brief psychotic episode in which the client loses contact with reality. d. Two distinct personalities within the borderline client.

b. A primitive defense mechanism in which the client sees objects as all good or all bad.

The primary goal in working with an actively psychotic, suspicious client would be to: a. Promote interaction with others. b. Decrease his anxiety and increase trust. c. Improve his relationship with his parents. d. Encourage participation in therapy activities.

b. Decrease his anxiety and increase trust.

Milieu therapy is a good choice for clients with antisocial personality disorder because it: a. Provides a system of punishment and rewards for behavior modification. b. Emulates a social community in which the client may learn to live harmoniously with others. c. Provides mostly one-to-one interaction between the client and therapist. d. Provides a very structured setting in which the clients have very little input into the planning of their care.

b. Emulates a social community in which the client may learn to live harmoniously with others.

Survivors of Abuse: Background Assessment Data: what is marital rape?

rape of a married partner

Josh, age 21, has been diagnosed with schizophrenia. He has been socially isolated and hearing voices telling him to kill his parents. He has been admitted to the psychiatric unit from the emergency department. The initial nursing intervention for Josh is to: a. Give him an injection of Thorazine b. Ensure a safe environment for him and others. c. Place him in restraints d. Order him a nutritious diet.

b. Ensure a safe environment for him and others.

Nurse Jones is caring for a client who has been hospitalized with anorexia nervosa and is severely malnourished. the client continues to refuse to eat. What is the most appropriate response by the nurse? a. You know that if you don't eat you'll die. b. If you continue to refuse to take food orally, you will be fed through a nasogastric tube c. You might as well leave if you are not going to follow your therapy regimen d. You don't have to eat if you don't want to. It is your choice.

b. If you continue to refuse to take food orally, you will be fed through a nasogastric tube

Margaret, a 68-year-old widow, is brought to the emergency department by her sister-in-law. Margaret has a history of bipolar disorder and has been maintained on medication for many years. Her sister-in-law reports that Margaret quit taking her medication a few months ago, thinking she no longer needed it. Margaret is agitated, pacing, demanding, and speaking very loudly. Her sister-in-law reports that Margaret eats very little, is losing weight, and almost never sleeps. "I'm afraid she's going to just collapse!" Margaret is admitted to the psychiatric unit. What is the priority nursing diagnosis for Margaret? a. Imbalanced nutrition: Less than body requirements related to not eating b. Risk for injury related to hyperactivity c. Disturbed sleep pattern related to agitation d. Ineffective coping related to denial of depression

b. Risk for injury related to hyperactivity

A nurse is educating a client about his lithium therapy and explaining signs and symptoms of lithium toxicity. Which of the following would she instruct the client to be on the alert for? a. Fever, sore throat, malaise b. Tinnitus, severe diarrhea, ataxia c. Occipital headache, palpitations, chest pain d. Skin rash, marked rise in blood pressure, bradycardia

b. Tinnitus, severe diarrhea, ataxia

Which of the following behavioral patterns is characteristic of individuals with schizotypal personality disorder? a. belitting themselves and their abilities b. a lifelong pattern of social withdrawal c. suspicious and mistrustful of others d. overreacting inappropriately to minor stimuli

b. a lifelong pattern of social withdrawal

Which of the following physical manifestations would you expect to assess in a client suffering from anorexia nervosa? a. tachycardia, hypertension, hyperthermia b. bradycardia, hypertension, hypethermia c. bradycardia, hypotension, hypothermia d. tachycardia, hypotension, hypothermia

c. bradycardia, hypotension, hypothermia

A client reveals that she induces vomiting as often as a dozen times a day. The nurse would expect assessment findings to reveal a. tachycardia. b. hypokalemia. c. hypercalcemia. d. hypolipidemia.

b. hypokalemia. Vomiting causes loss of potassium, leading to hypokalemia.

Which intervention would be least useful for accurate assessment of the weight of a client diagnosed with anorexia nervosa? a. Weigh two times daily, then three times weekly. b. Weigh fully clothed before breakfast. c. Do not reweigh client when client requests. d. Permit no oral intake before weighing.

b.Weigh fully clothed before breakfast. Clients should be weighed wearing only bra and panties before ingesting any food or fluids in the morning.

Survivors of Abuse: Background Assessment Data: Intimate Partner Violence: how is intimate partner violence defined?

battering: aka domestic violence CDC defines IPV as: physical violence, sexual violence, stalking and psychological aggression by intimate partner;

Survivors of Abuse: Background Assessment Data: Emotional Abuse: what is considered emotional abuse?

belittling or rejecting child ignoring the child blaming child for things out of their control isolating child from social experiences

Biological Theories: Disorders of the brain: what brain d/o's are involved /c aggressive / violent bx?

brain tumors in: limbic system and temporal lobes trauma to brain encephalitis secondary to cerebral trauma epilepsy temporal lobe epilepsy

A client recently admitted to the hospital reports to the nurse, "I don't understand why I was brought here. I was simply hanging out in my apartment and the police said I had to come with them." This is an example of what symptom of schizophrenia? a. Delusions of reference b. Loose association c. Anosognosia d. Auditory hallucinations

c. Anosognosia

Survivors of Abuse: Background Assessment Data: The Victim: what what are two response patterns that can occur hours after rape?

expressed response pattern: expresses feelings of: fear anger anxiety ie: crying, sobbing, restlessness, tension controlled response pattern: feelings are: masked or hidden ie: calm, composed, subdued affect

The nurse is interviewing a client on the psychiatric unit. The client tilts his head to the side, stops talking in midsentence, and listens intently. The nurse recognizes from these signs that the client is likely experiencing: a. Somatic delusions. b. Catatonic stupor. c. Auditory hallucinations. d. Pseudoparkinsonism.

c. Auditory hallucinations.

A hospitalized client with bulimia nervosa has stopped vomiting in the hospital and tells the nurse she is afraid she is going to gain weight. Which is the most appropriate response by the nurse? a. Don't worry. The dietitian will ensure you don't get too many calories in your diet. b. Don't worry about your weight. We are going to work on other problems while you are in the hospital. c. I understand that you are concerned about your weight, and we will talk about the importance of good nutrition; but for now I want you to tell me more about your recent invitation to join the National Honor Society. That's quite an accomplishment." d. You are not fat, and the staff will ensure that you do not gain weight while you are in the hospital because we know that is important to you."

c. I understand that you are concerned about your weight, and we will talk about the importance of good nutrition; but for now I want you to tell me more about your recent invitation to join the National Honor Society. That's quite an accomplishment."

which type of aggression is more common in people that are abused?

reactive aggression

In evaluating the progress of Jack, a client diagnosed with antisocial personality disorder, which of the following behaviors would be considered the most significant indication of positive change? a. Jack got angry only once in group this week b. Jack was able to wait a whole hour for a cigarette without verbally abusing the staff c. On his own initiative, Jack sent a note of apology to a man he had injured in a recent fight d. Jack stated that he would no longer start any more fights

c. On his own initiative, Jack sent a note of apology to a man he had injured in a recent fight

In evaluation the progress of Jack, a client diagnosed with antisocial personality disorder, which of the following behaviors would be considered the most significant indication of positive change? a. Jack got angry only once in group this week. b. Jack was able to wait a whole hour for a cigarette without verbally abusing the staff. c. On his own initiative, Jack sent a note of apology to a man he had injured in a recent fight. d. Jack stated that he would no longer start any more fights.

c. On his own initiative, Jack sent a note of apology to a man he had injured in a recent fight.

Margaret, age 68, is diagnosed with bipolar I disorder, current episode manic. She is extremely hyperactive and has lost weight. One way to promote adequate nutritional intake for Margaret is to: a. Sit with her during meals to ensure that she eats everything on her tray. b. Have her sister-in-law bring all her food from home because she knows Margaret's likes and dislikes. c. Provide high-calorie, nutritious finger foods and snacks that Margaret can eat "on the run." d. Tell Margaret that she will be on room restriction until she starts gaining weight.

c. Provide high-calorie, nutritious finger foods and snacks that Margaret can eat "on the run."

A client experiencing a manic episode enters the milieu area dressed in a provocative and physically revealing outfit. Which of the following is the most appropriate intervention by the nurse? a. Tell the client she cannot wear this outfit while she is in the hospital b. Do nothing, and allow her to learn from the responses of her peers. c. Quietly walk with her back to her room and help her change into something more appropriate. d. Explain to her that if she wears this outfit, she must remain in her room

c. Quietly walk with her back to her room and help her change into something more appropriate.

Which diagnosis from the list below would be given priority for a client diagnosed with bulimia nervosa ? A. Disturbed body image B. Chronic low self-esteem C. Risk for injury: electrolyte imbalance D. Ineffective coping: impulsive responses to problems

c. Risk for injury: electrolyte imbalance The client who engages in purging and excessive use of laxatives and enemas is at risk for metabolic acidosis from bicarbonate loss.

Which intervention would be removed from the plan of care for a client diagnosed with bulimia nervosa? a. Teach that fasting sets one up to binge eat. b. Assist client to identify trigger foods. c. Support importance of avoiding forbidden foods. d. Teach client to plan and eat regularly scheduled meals.

c. Support importance of avoiding forbidden foods. No foods should be considered forbidden foods. This issue may be a focus of cognitive behavioral therapy.

The nurse is caring for a client with schizophrenia. Orders from the physician include 100 mg chlorpromazine IM STAT and then 50 mg PO bid; 2 mg benztropine PO bid prn. Why is chlorpromazine ordered? a. To reduce extrapyramidal symptoms b. To prevent neuroleptic malignant syndrome c. To decrease psychotic symptoms d. To induce sleep

c. To decrease psychotic symptoms

Characteristic behaviors the nurse will assess in the narcissistic client are a. dramatic expression of emotion, being easily led. b. perfectionism and preoccupation with detail. c. grandiose, exploitive, and rage-filled behavior. d. angry, highly suspicious, aloof, withdrawn behavior.

c. angry, highly suspicious, aloof, withdrawn behavior. Narcissistic clients give the impression of being invulnerable and superior to others to protect their fragile self-esteem.

In contrast to the client diagnosed with anorexia nervosa, the client diagnosed with bulimia usually a. uses greater denial. b. is aware of the eating problem. c. fits more easily into the family. d. appraises his or her body more realistically.

c. fits more easily into the family. There is less family concern about the client with bulimia because these clients appear physically normal, the weight is at or near normal, they eat with the family, and the purging is done in secret. The anorexic client is noticed by the family for painful thinness and poor food intake.

During assessment of a client with anorexia nervosa, it is not likely that the nurse would note indications of a. introversion. b. social isolation. c. high self-esteem. d. obsessive-compulsive tendencies.

c. high self-esteem. Most clients with eating disorders have low self-esteem.

Survivors of Abuse: Background Assessment Data: Sexual Abuse of a child: what are indicators of sexual abuse?

child has: difficulty walking refuses to change for gym reports nightmares sudden chagne in appetite unsual knowledge / sexual bx contracts a veneral dz when younger than 14 runs away reports sexual abuse attaches quickly to strangers suspect parent when: unduly protective of child limits child's contact /c other children secretive and isolated jealous and controlling /c other family members

Survivors of Abuse: Background Assessment Data: Emotional abuse: what are indicators of neglect?

child is: frequently absent from school begs or steals food or money lacks needed medical or dental care consistently dirty lacks suffic. clothing abuses alcohol states no one is home suspect parent commits neglect when the parent: appears indifferent to the child apathetic / depressed behaves irrationally abuses alcohol

Psychological Theories: Learning Theory: how is learning theory r/t aggressive bx?

children imitate role models ie parents PTs abused as a child / saw domestic violence => more likely to have reactive aggression PTs /c biolog. predisp. to aggressive bx => more susceptible to negative role modeling

Survivors of Abuse: Background Assessment Data: Sexual Abuse of a child: what is sexual abuse?

coercion of a child => engage in sexual conduct /c another person sexual exploitation of a child: child is induced or coerced into engaging in sexual conduct to promote pleasure of an adult

Brandon, a client on the psychiatric unit, has been diagnosed with schizophrenia. He begins to tell the nurse about how the CIA is looking for him and will kill him if they find him. The most appropriate response by the nurse is: a. "That's ridiculous, Brandon. No one is going to hurt you." b. "The CIA isn't interested in people like you, Brandon." c. "Why do you think the CIA wants to kill you?" d. "I know you believe that, Brandon, but it's really hard for me to believe."

d. "I know you believe that, Brandon, but it's really hard for me to believe."

Biological Theories: Biochemical Influences: what neurotransmitters are r/t aggression?

increased dopamine release low levels of striatal serotonin high plasma concentration of 5-HIAA serotonin complex interaction b/t tesosterone and corisol

What is the most common comorbid condition in children with bipolar disorder? a. Schizophrenia b. Substance disorders c. Oppositional defiant disorder d. Attention-deficit/hyperactivity disorder

d. Attention-deficit/hyperactivity disorder

Your patient, Erin, is a 16-year-old patient newly diagnosed with anorexia. Her provider is starting her on medication to reduce compulsive behaviors regarding food and resistance to weight gain. You prepare teaching on which class of medication that may help these specific symptoms in eating disorders? a. Mood stabilizers b. Antidepressants Incorrect c. Anxiolytics d. Atypical antipsychotics

d. Atypical antipsychotics Atypical antipsychotic agents may be helpful in improving mood and decreasing obsessional behaviors and resistance to weight gain. Mood stabilizers are not specifically used in treatment of eating disorders. The antidepressant fluoxetine (Prozac, an SSRI) has proven useful in reducing obsessive-compulsive behavior after the patient has reached a maintenance weight. Anxiolytics would be prescribed for anxiety.

Survivors of Abuse: Background Assessment Data: Adult survivors of incest: what physiological changes may occur r/t childhood maltreatment in adult survivors?

increased levels of: CRP fibrinogen pro inflammatory cytokines common comorbid's: depression anxiety substance use eating d/o's SI bx

According to Margaret Mahler, predisposition to BPD occurs when developmental tasks go unfulfilled in which of the following phases? a. Autistic phase, during which the child's needs for security and comfort go unfulfilled. b. Symbiotic phase, during which the child fails to bond with the mother. c. Differentiation phase, during which the child fails to recognize a separateness between self and mother. d. Rapprochement phase, during which the mother withdraws emotional support in response to the child's increasing independence

d. Rapprochement phase, during which the mother withdraws emotional support in response to the child's increasing independence

Jessica is a nurse who was floated to the psychiatric unit to cover for a staff nurse who called out sick. She encounters a patient diagnosed with BPD, and the patient states, "Thank goodness they sent you to the unit. No one else here has taken the time to listen to my concerns." This may be an example of which symptom of BPD? a. Impulsivity b. Self-harming behaviors c. Dissociation d. Splitting

d. Splitting

The nurse is caring for a client with schizophrenia. Orders from the physician include 100 mg chlorpromazine IM STAT and then 50 mg PO bid; 2 mg benztropine PO bid prn. Because benztropine was ordered on a prn basis, which of the following assessments by the nurse would convey a need for this medication? a. The client's level of agitation increases. b. The client complains of a sore throat. c. The client's skin has a yellowish cast. d. The client develops muscle spasms

d. The client develops muscle spasms

A client who is 16 years old, 5 foot, 3 inches tall, and weighs 80 pounds eats one tiny meal daily and engages in a rigorous exercise program. The nursing diagnosis for this client would be a. death anxiety. b. ineffective denial. c. disturbed sensory perception. d. imbalanced nutrition: less than body requirements.

d. imbalanced nutrition: less than body requirements. A body weight of 80 pounds for a 16-year-old who is 5 foot, 3 inches tall is ample evidence of this diagnosis.

Survivors of Abuse: Background Assessment Data: Emotional Abuse: What are sx's of emotional abuse?

extremes in bx (ie extreme passivity, aggression) inappropiately adult inappropriately infantile delay in emotional / phys. development suicide attempts lack of attachment to parent suspect emotional abuse when parent: constantly blames, belittles, unconcerned about the child overtly rejects the child

what are indicators of abuse in a child?

extremes of behavior, delayed physical or emotional development, lack of attachment to parent

Survivors of Abuse: Background Assessment Data: Why do victims of abuse stay?

fear of their lives / children's lives 1. fear of: retaliation: may have been threatened /c murder ie: blackmail murdering a pet fear of being "outed" in LGBTQ relationships losing custody of children: threatening to take kids away convince victim she is unfit mother 2. physical / financial independence: victims /c disabilities may depend on batterer 3. lacking social support: may be isolated from family or friends may be pressured by family to stay in marriage 4. cultural / religious reasons: religious convictions against divorce 5. hopefulness: denial in that batterer will change 6. lack of attention to the danger: disassociaton r/t PTSD may make victim oblivious to staying in dangerous relationship

what are behavioral indicators of neglect?

frequent absences from school, begs or steals food or money, lack needed medical dental immunizations or glasses, consistently dirty or serious B.O, abuses alcohol or drugs,

Survivors of Abuse: Background Assessment Data: The Victim: what is the character profile of the victim?

highest risk age group: younger than 34 lower income living in rural areas single attacker is in own neighborhood

Survivors of Abuse: Background Assessment Data: Cycle of Battering: what char's phase 3?

honey moon phase aka calm, loving, respite phase (can last very short, almost undetected, OR as long as phase I) batterer becomes loving, kind promises abuse won't happen again victim: wants to believe batterer will change redreams ideal life /c batterer (believes this is what her partner is really like) uses this phase as the reason to stay in relationship and not leave

hypomania vs mania length?

hypomania: 4 days mania: 1 week

Survivors of Abuse: Background Assessment Data: Adult survivor of incest: what char's the adult survivor of incest?

lack of trust low self esteem poor sense of identity when reporting incest, mother often doesn't believe them some victims: either avoid sex all together; or become very promiscuous sx's of child abuse include: in males: ED premature ejaculation exhibitionistic d/o compulsive sexual conquests

what areas are associated /c aggressive bx?

limbic: emotional alter's prefrontal and frontal cortices: social judgement amygdala: anxiety, rage, fear hypothalamus: stimulates SNS (fight/flight) hippocampus: learning and memory

Survivors of Abuse: Background Assessment Data: Profile of the Victimizer: what are char's of the victimizer aka batterer?

limited ability to cope possessive views spouse as possession

Survivors of Abuse: Background Assessment Data: Profile of the Victim: what are the char's of the victim of abuse?

low self esteem typically adhere to feminine sex role stereotypes accept blame for batterer's actions develops learned helplessness (no matter what PT does, outcome is the same; what's that remind you of?? pfffft)

Biological Theories: Neurophysiological Influences:what parts of the brain are involved in aggression?

lower amygdala volume (resp. for impulse control and affective processing) limbic prefrontal cortex: smaller volumes of left sided gray matter greater right sided volume lowered connectivity b/t amygdala and prefrontal cortex dysfunctional striatum

Survivors of Abuse: Background Assessment Data: Adult survivors of incest: what ironically occurs /c certain relationships /c survivors of incest?

may enter: relationships /c men that abuse them aka "repetition compulsion" which is: reliving a trauma rooted in subconscious in order to => fix situations they couldn't fix in childhood

what are examples of disorders of the brain that influence abusive behaviors?

neurocognitive disorders, brain tumors/trauma, encephalitis, temporal lobe epilepsy (TLE)

what are examples of biochemical influences for abusive behaviors?

norepinephrine, serotonin, dopamine

Interpersonal psychotherapy is suggested for what personality disorders?

paranoid, schizoid, shizotypal, borderline, dependent, narcissistic, and obsessive-compulsive personality disorders

what are characteristics of a mother who sexually abuses their child?

passive, submissive, denigrates her role of wife/mother; uses denial of incest, keeps quite for fear of abuse from husband

what are the types of child abuse?

physical abuse, emotional abuse, physical neglect, emotional neglect.

Treatment Modalities: Crisis Intervention: what are two other treatment methods other than crisis intervention?

safe house or shelter: individual or group counseling is provided at the shelter / safe house family therapy: help families develop democratic ways of problem solving (instead of using violence) ie: time out techniques for children positive reinforcement for acceptable bx

Survivors of Abuse: Background Assessment Data: Profile of the victimizer: what are the four classifications of victimizers?

sexual sadists: aroused by inflicting pain exploitative predators: use victim for dominance and power inadequate men who are obsessed /c sex fantasies (that can't achieve them /cout force) men that displace anger and rage

what are predisposing sociocultural theories to abusive behaviors?

societal influences= Aggressive behavior is primarily a product of one's culture and social structure. The American culture was founded on a general acceptance of violence as a means of solving problems.

Genetics

strong evidence that genes play a major role in the etiology of bipolar disorders.

what are examples of nuerophysiological influences for abusive behaviors?

temporal lobe, limbic system, amygdaloid nucleus

Survivors of Abuse: Background Assessment Data: Cycle of Battering: what is char / phase 1?

tension building phase (lasts few weeks to many months or years) batterer: his tolerance for frustration declines may lash out, but quickly apologize minor battering incidents victim: denies anger and rationalizes batterer's bx ie: "I need to do better." "it's the alcohol, if only he didn't drink." may even convince herself she deserves the abuse battering becomes more intense, victim withdraws from batterer => batterer thinks victim is rejecting him

what is the psychodynamic theory?

the abuser has unmet needs for satisfaction and security and aggression and violence supply the individual with power and prestige that increases self-esteem

what is the learning theory?

the abuser learned their abusive behavior most likely from childhood

Survivors of Abuse: Background Assessment Data: Profile of the victimizer: what are char's of premeditated rapes?

victim that is vulnerable violating victim's rights victimizer often has no empathy **most rapists are b/t age of: 25 - 44 yo most are white; offender was known to victim

Biological Theories: Genetic influences: what genetic influences are r/t aggression?

x linked monoamine oxidase A gene r/t antisocial bx *more research needed though strong 5HT (hydroxytryptamine) transporter genes


Set pelajaran terkait

2204 Schizophrenia and Other Psychotic Disorders

View Set

Pre-Ch 2-1, 2-2, & W2 : Discovering the Universe for Yourself

View Set

Microbiology Exam 4 (chapter 14, 15, and 16)

View Set

SOCI 1101H - Chapter 11 Review Questions

View Set